Combined CRACKcast through 7-5

Ace your homework & exams now with Quizwiz!

List the 5 stages of wound healing (Ch 59):

1. Coagulation (immediate) 2. Inflammation (immediate - 48 hours) 3. Collagen metabolism (>48 hours, peaks 7d, greatest mass 3wks) 4. Wound contraction 5. Epithelialization (48 hours - days) Epithelial Cell migrate across wound, soon resembling uninjured skin

List 4 complications of tibial shaft fractures (Ch 57):

1. Compartment syndrome (highest risk at 24-48 hrs) 2. Infection - shallow easily exposed bone (look for subtle open #!) 3. Nerve injury - "foot off the brake, to the right, and on the gas" 4. DVT, pseudoaneurysm 5. Fat embolism 6. Delayed healing - average union times is 20-30 weeks 7. Malrotation 8. CRPS

What are two ethical considerations in intimate partner violence (Ch 68)

1. Confidentiality 2. Informed consent and autonomy

Describe the management of testicular torsion (Ch 99)

1. Consult urology ASAP (even if U/S is negative if you have a high pre-test probability for TT) 2. Attempt manual detorsion if no immediate urologist available 3. Analgesia and antiemetics

List the 5 most common disease processes resulting in HF and briefly describe the contribution of each (Ch 81):

1. Coronary artery disease #1 cause. Myocardial necrosis, fibrosis and scarring → dyskinesis. Aneurysmal dilation of infarcted areas. Cardiogenic shock -- occurs when 40% of the LV muscle is infarcted. Chronic coronary insufficiency: Diffuse ischemic cardiomyopathy Worsened in sickle cell disease, diabetes mellitus Ventricular remodelling - dilation, hypertrophy, fibrosis 2. Cardiomyopathy / myocarditis Dilated, hypertrophic, restrictive 3. Valvular heart disease Mitral or aortic valve regurg/stenosis 4. Pericardial diseases Look for the pericardial effusion 5. Pulmonary disease COPD - 30% prevalence in people with CHF Hypoxia also leads to pulmonary arteriolar vasoconstriction which leads to cor pulmonale PE

List 3 types of drug-induced liver disease (Ch 90)

1. Cytotoxic ("ALT") Acetaminophen Lovastatin Tetracycline 2. Cholestatic ("ALP") Azathioprine haLdoL Phenobarbital 3. Veno-occlusive ("AAA") Anabolic steroids Azathioprine Anti-pregnancy (OCP's)

4 Contraindications to CPR: (Ch. 9)

1. DNR ordered 2. Unwitnessed arrest in pt who is cold, in rigor mortis 3. Traumatic arrest 4. Unsafe to perform CPR

List 6 physical findings in compartment syndrome: (Ch. 49)

1. Deep, burning, unrelenting, difficult to localize pain 2. Increasing need for analgesics 3. Pain on passive stretching of muscle 4. Pain with active flexion of muscle 5. Hypoesthesias or paresthesias in the distribution of nerves crossing the compartment 6. Tenderness / tenseness of the compartment *NOTE: the 5 P's (pulseless, pallor, poikilothermic, etc) relate to acute disruption of arterial flow and are NOT reliable indicators of compartment syndrome

List 5 late complications of vascular injuries: (Ch. 48)

1. Delayed Thrombosis (e.g post operative anastomoses or shunt placement) 2. Intermittent claudication 3. Chronic pain (nerve compression / ischemia) 4. Edema (venous compression / elevated compartment pressure) 5. Aneurysm / Pseudoaneurysm formation

List 6 low-risk criteria that should be met if discharging an UGIB: (Ch. 30)

1. No comorbid disease 2. normal vital signs 3. normal of trace positive result of stool guaiac test 4. normal hgb and hematocrit 5. patient understanding of sx of significant bleeding 6. immediate access to care

First line agents to treat status epilepticus: (Ch. 18)

--Diazepam 0.15mg/kg IV --Lorazepam 0.1 mg/kg IV --Midazolam 0.1 mg/kg try two doeses then switch to second line...

What lab tests would you order for a jaundiced patient? (Ch. 28)

--GGT- if up, confirms alkphos elevation is hepatic in origin --Retic count - if up, suggests hemolysis --Acetaminophen level - AST first to rise --Ascitic fluid - for culture --Blood culture - if febrile --Glucose level --INR, PTT --AST, ALT

Define massive hemoptysis: (Ch. 24)

--Greater than 500cc expectorated blood /24hr or --Bleeding rate >100ml/hr, regardless of gas exchange

When is a stool culture indicated? (Ch. 31)

--Ill-appearing --Immunocompromised (including young and old people) --Non-responders to treatment --Chronic course

List some risk factors of GI bleeding: (Ch. 30)

--NSAIDS, steroids, anticoagulants, ASA --hx of PUD --known liver cirrhosis --advanced age >60 --alcoholism --current smoker --hx of AAA of graft

Second line agents to treat status epilepticus: (Ch. 18)

--Phenytoin 20 mg/kg IV infusion over 10 minute --Phenobarb 20-30 mg/kg IV " " --Propofol

What are the causes of enophthalmos? (Ch. 22)

--contralateral proptosis --penetrating globe injury causing vitreous extrusion

What are some common upper GI bleed MIMICS? (Ch. 30)

--epistaxis --hemoptysis --dental bleeds --red food colouring --iron/bismuth supplements

What are some common lower GI bleed MIMICS? (Ch. 30)

--gross hematuria --vaginal bleeding --red foods (beets!)

What are the causes of exophthalmos? (Ch. 22)

--hyperthyroidism (Grave's disease) --orbital cellulitis --retrobulbar hematoma (most common) --orbital emphysema or inflammation

RFs for a *central* cause of vertigo: (Ch. 19)

--older age --male --HTN --CAD --DM --AFib

What are the causes of anisocoria (pupil asymmetry)? (Ch. 22)

--previous eye trauma --previous eye surgery (iridotomy) --synechiae from previous iritis --physiologic (up to 10% of gen pop) --medication related --serious causes: uveitis, AACG, Horner's

What kind of cervical spinal injuries can you get from EXTENSION mechanisms? And are they stable/unstable? (Ch. 43)

1. Posterior neural arch # UNSTABLE 2. Hangman's # UNSTABLE 3. Extension Teardrop UNSTABLE

List the top 3 risk factors for contrast induced ATN (Ch 97)

1. Pre-existing renal disease (especially Cr >120) 2. Diabetes mellitus 3. Age >75yrs

Pathophysiology of fever: (Ch. 12)

1. Pre-optic neurons in hypothalamus directly sense blood temperature 2. Vasomotor changes activated by neurons: shivering, metabolic changes, behavioural changes 3. Cytokines/pyrogens signal hypothalamus to release prostaglandins, resetting temp set-point

List 12 causes of vaginal bleeding (3 per age group)? (Ch. 34)

1. Pre-pubertal: vaginitis, genital trauma/foreign body 2. Adolescent: anovluation, *coagulopathy*, pregnancy, hormone use 3. Reproductive: pregnancy, anovluation, hormone use, 4. Peri-menopausal: anovulation, uterine leiomyomas, polyps 5. Post-menopausal: endometrial cancer, hormone use, vulvar/cervical cancer, vaginitis

Quality CPR involves the following: (Ch. 9)

1. Two hands over lower half of sternum 2. Compression rate: 100-120 bpm 3. Compression depth: 2 inches (5 cm) 4. Allow full recoil of chest 5. Minimize pauses! 6. Good ventilation: 2 breaths every 30 compressions, breath delivered over 1 sec.

What information can be useful in determining location and likelihood of injury in sexual assault (Ch 67)

1. Type of sexual assault 2. Sexual dysfunction history in suspect 3. Use of foreign body 4. Victim assistance with insertion of penis 5. History of victims sexual experience 6. Gravity/ parity 7. Mental health history 8. Substance use 9. Method of controlling the victim 10. Multiple assailants

List 5 brain herniation syndromes: (Ch. 41)

1. Uncal herniation 2. Central transtentorial herniation 3. Upward transtentorial herniation 4. Downward cerebellar-tonsillar herniation 5. Cingulate-subfalcine herniation

List 2 CXR findings consistent with PE (Ch 88)

1. Unilateral basilar atelectasis 2. Hampton's hump 3. Westermark's sign (unilateral lung oligemia)

What are the six indications for intubating a burn patient?(Ch. 63)

1. Upper airway obstruction 2. Inability to handle secretions 3. Hypoxemia despite 100% O2 4. Patient obtundation 5. Muscle fatigue suggested by high or low resp rate 6. Hypoventilation(PC)2>50, ph <7.2)

What % of the population will have one or more seizures in their life? (Ch. 18)

10%

Describe the ED management of a pt with aortic dissection (Ch 85)

1st: Pain control to counter sympathetic contribution to elevated heart rate and blood pressure Fentanyl 25-50 mcg boluses. This is KEY! Because it helps reduce BP and reduce shearing forces on the artery 2nd: HR control to a goal of 60 bpm Esmolol 0.5mg/kg bolus then 50-300 mcg/kg/min or Labetalol 10-20mg bolus then 0.5-2mg/min Warning: Giving a vasodilator without concomitant reduction in inotropy may cause progression of dissection Don't be afraid to keep going up on Labetalol - keep doubling until a total of 300 mg of Labetalol is given! Even if their BP is ok - give a BB to drop the HR to 60. 3rd: BP control to a goal of SBP = 110 Nitroprusside 0.25-0.5 mcg/kg/min then titrate or Nicardipine 5mg/hr Warning: Beware of pseudohypotension! of the bilateral BPs use the higher BP reading.

Define SIRS

2 or more of: -Temp: <36 or >38.3 -HR: >90 -RR: >20 or CO2 <32 (resp. alk.) -WBC: <4k or >12k

Two ways to choose size of chest tube in kids: (Ch. 38)

2 x ETT size 4 x ETT size

Dose and indication for core peds resuscitation med: *Dobutamine* (Ch. 10)

2-20 mcg/kg/min ; for cardiogenic shock (B1 agonist)

Dose and indication for core peds resuscitation med: *Dopamine* (Ch. 10)

2-20 mcg/kg/min ; for shock and hypotension -Low dose: dopamine receptor -vasodilation (coronary) -Med dose: beta 1 agonist -inotrope -High dose: alpha 1 agonist -vasocontriction

What is Dressler's Syndrome? Outline the management of this for bonus points (Ch 82):

2-4 weeks (although can be up to 10-weeks) post MI pericardial chest-pain. 20% of post of MI pts experience a "different quality of chest pain" +/- low grade fever and rub ECG changes are often masked by the ACS ECG changes At risk for dysrhythmias and CHF Mgmt: 1-3 days of ASA 325 mg daily

Base deficit less than what makes us worried? (Ch. 6)

-4 (bigger negative # = BAD)

Indications for post-mortem C-Section: (Ch. 37)

-Must start within 4 min of maternal cardiac arrest -Present fetal heart tones -Greater than 24 wks gestation

Proposed indications for mechanical ventilation (i.e. intubation) (Rosen's box)

-Respiratory arrest -Declining LOC despite maximal interventions -Unstable (shock or CHF decompensation) -Failure of NIPPV or inability to use NIPPV -severe dyspnea with significant WOB (accessory muscles and paradoxical abdo motion) - Severe tachypnea - Life threatening hypoxia -severe acidosis and hypercapnia - other complications (metabolic, sepsis, PE, barotrauma, massive pleural effusion)

Common Sites for Tendinitis: (Ch. 49)

-Rotator cuff of the shoulder -Achilles tendon -Radial aspect of the wrist (de Quervain's tenosynovitis), -Insertion of the hand extensors on the lateral humeral epicondyle (tennis elbow). -Patellar tendon -Biceps femoris, semitendinosus, and semimembranosus (hamstring syndrome); -Posterior tibial tendon (shin splint syndrome) -Iliotibial band -Common wrist extensors (medial epicondylitis) (little league pitchers and golfers)

Complications arising after traumatic head injury? (Ch. 41)

-Seizures -CNS infection - meningitis, brain abscesses, osteomyelitis -DIC -Cardiac dysfunction

PHYSIOLOGIC indications for trauma team activation: (Ch. 36)

-Systolic BP <90 -RR <10 or >30 -GCS <12 or focal neurological signs

Management of local anaesthetic toxicity: (Ch. 4)

-Therapy for extravasation (warm compress, phentolamine, nitro cream) -Time

Key considerations with spinal injury in pediatric trauma: (Ch. 38)

-They are rare -Higher cord injuries are more common than lower cord -SCIWORA (Spinal Cord Injury without Radiography Abnormality) -found in 25-50% of spinal injuries

Describe basic approach to STABLE pts with acute neck trauma: (Ch. 44)

-Transport to trauma centre -Basic wound compression and occlusion (to prevent air embolus) -If neurologic deficits → should prompt consideration for cervical collar

Describe the management of suspected pharyngoesophageal trauma" (Ch. 44)

-W/U with X-ray of chest and neck - r/o pneumomediastinum -Combo endoscope with contrast swallow - 100% sensitive -CT scan with contrast -Abx/NPO/NGT if not contraindicated

ED management of *moderate TBI* (GCS 9-13): (Ch. 41)

-Watch for deterioration in first 48hrs! Serial neuro checks -Admit for close observation, even if CT head normal

Differential of CRITICAL etiologies of cyanosis: (Ch. 14)

-acute heart failure -ACS -hypovolemic or cardiogenic shock -acute rspiratory failure -massiv PE -congenital heart disease

ANATOMIC indications for trauma team activation: (Ch. 36)

-amputation proximal to elbow or knee -2 or more long bone #s -flail chest -tension pneumo or hemothorax -suspected spinal injury with deficit -suspected penetrating injury -unstable pelvis

6 techniques for reducing pain of injection of local anaesthetic: (Ch. 4)

-buffer 1 mL bicarbonate to 10 ml lidocaine (1:10) -inject slowly -inject through laceration itself -warm anaesthetic -apply topical anaesthetic first -mitigate irritation with distraction

Warning signs of impending RESPIRATORY FAILURE in pt with neuromuscular weakness: (Ch. 13)

-climbing pCO2 -decreasing LOC -decreasing peak flows -tachypnea -shallow breaths -work of breathing (accessory muscle use)

List 5 or more risk factors for FALLS in the elderly: (Ch. 39)

-cognitive impariment -vision/hearing loss -impaired thirst mechanism - dehydration - orthostatic drops -reduced respiratory reserve -cardiac disease - poor CO, arrythmias -arthritis/osteophytes - poor mobility -loss of fine motor skills -decreased balance -drugs/polypharmacy

Situations where capnography is useful: (Ch. 5)

-confirming ventilation and tube placement -a rise may indicate ROSC -most sensitive detectn of apnea in proc. sedation -confirming adequate ventilation in postictal, overdosed, or intoxicated pts -in acidotic pts developing resp alkalosis (where EtCO2 drops) -close correlation btw alveolar and arterial pCO2 -shape of capnograph tells about obstruction, spont resp effort, or ET leaks

Risk factors of GI bleeding... (Ch. 3)

-dehydration, hypovolemia -liver disease, renal disease -on ACE-I or ARBs -elderly -asthmatics -hx GI bleeding or PUD -anticoagulated (warfarin)

MECHANISTIC indications for trauma team activation: (Ch. 36)

-ejection from vehicle -ped-struck >30km/hr -high speed MVC or roll-over -fall > 20ft or 6m -severe deceleration injury -bicycle or motorcycle crash -2nd or 3rd degree burns >10% BSA -inhalation burns *special considerations: >60, <16 years, pregnancy

"Ventilator trouble shooting": Potential *MECHANICAL* causes of acute resp distress *without* hemodynamic compromise... (Ch. 2)

-endotrach tube migration (into bronchus) -endotrach tube obstruction -endotrach tube cuff leak -inadvertent extubation -ventilator circuit discontinuity

How do you manage an avulsed tooth? (Ch. 42)

-tooth placed in saline or ToothSaver solution -rinse but DO NOT wipe (may damage tennuous ligaments) -reimplant tooth with firm pressure -secure tooth with splint -referral to dentist -66% viability if tooth replaced in <1hr

How do you reverse arterial spasms in context of vascular trauma? (Ch. 48)

-warm saline compress -topical nitrates -IV nitrates -CCBs -alpha blockers -prostaglandins

LOGISTICAL indications for trauma team activation: (Ch. 36)

-whenever the emergency physician needs more help or resources

"Ventilator trouble shooting": Potential *PHYSIOLOGIC* causes of acute resp distress *without* hemodynamic compromise... (Ch. 2)

-worsening lung compliance -worsening airway obstruction -abdo distension -PE -pain or inadequate sedation

Dose and indication for core peds resuscitation med: *Epinephrine* (Ch. 10)

0.01 *mg*/kg ; for persistent unstable brady, PEA, or VT/VF despite defib, or shock 0.01 *mcg*/kg/min ; for shock

Dose and indication for core peds resuscitation med: *Naloxone* (Ch. 10)

0.01-0.1 mg/kg ; for opiate overdose

Dose and indication for core peds resuscitation med: *Atropine* (Ch. 10)

0.02 mg/kg ; for persistent unstable bradycardia

Dose and indication for core peds resuscitation med: *Norepinephrine* (Ch. 10)

0.05-2 mcg/kg/min ; for shock (septic)

Dose and indication for core peds resuscitation med: *Adenosine* (Ch. 10)

0.1 mg/kg (max 6 mg first dose)-rapid IV push ; for SVT 0.2 mg/kg (max 12 mg second dose) Each followed by saline push (20cc)

Dose and indication for core peds resuscitation med: *Dextrose* (Ch. 10)

0.5-1 g/kg IV/IO ; for hypoglycemia (adults D50, kids D25)

List Sgarbossa criteria for AMI in pre-existing LBBB (Ch 78):

(1) ST segment elevation of at least 1 mm that is concordant with the QRS complex; (2) ST segment depression of at least 1 mm in lead V1, V2, or V3; and (3) ST segment elevation of at least 5 mm that is discordant with the QRS complex.

What 2 factors affect the progression of an aortic dissection? (Ch 85)

(1) The degree of elevation of blood pressure (2) The steepness (slope) of the pulse wave (upstroke pattern on apex cardiogram, dP/dt) *Remember dP/dt is the shear force from the left ventricle, your change in pressure over a time interval seen by the proximal aorta

List 3 causes of pacemaker malfunction (Ch 80):

(1) failure to capture (no pacemaker spikes or spikes not followed by an atrial or ventricular complex) (2) inappropriate sensing (oversensing or undersensing spikes occur prematurely or do not occur even though the programmed interval is exceeded) (3) inappropriate pacemaker rate.

Differentiate between depolarizing and non-depolarizing paralytic agents: (Ch. 1)

*Depolarizing: Succinylcholine* -non-competitively binds Ach receptors -onset 45s, lasts 6-10 mins -decreased plasma-pseudocholinesterase increases amount of sux available -there are contraindications to its use *Non-Depolarizing: Rocuronium* -competitively binds Ach receptors -onset 60s, lasts 50 mins -reversed competitively by cholinesterase -no contraindications

Define conjugated, unconjugated, and total bilirubin: (Ch. 28)

*Conjugated bilirubin* = direct bilirubin *Unconjugated bilirubin* = indirect bilirubin (can cross blood-brain barrier if not bound to albumin) *Total bilirubin* = direct + indirect bilirubin

Conditions with increased risk of hyperkalemia from succinylcholine administration? (Ch. 1)

*Duration of >5 days*: -burn of >10% BSA -crush injury -intra-abdominal sepsis *Duration of 6 mo to years*: -denervation (stroke, cord injury) -neuromusc disease (ALS, MS, MD)

Two types of local anaesthetics: (Ch. 4)

*Esters* -procaine, tetracaine -unstable in solution -metabolized by plasma esterases *Amides* -lidocaine, mepivacaine, bupivacaine -stable in solution -metabolized by liver enzymes

Describe the classification system for open fractures: (Ch. 49)

*Gustilo Classification*: I = clean wound, < 1cm, "bone-poked through" II = wound 1-10 cm length, no contamination/crush III = >10 cm laceration, with extensive soft tissue damage/contamination/crush (farm wound) III.A = soft tissue stripped III.B = periosteal stripping and bone damage III.C = severe vascular injury

What signs do you see for the various ranges of fire of a bullet?(0, 0-6 inch, <48 inch, distant or indeterminate) (Ch 65)

0: Soot, seared skin, triangular tears 0-6: Soot, abrasion collar <48: Tattooing, abrasion collar Distant or ind: Abrasion collar

What are some clinical signs of liver disease? (Ch. 28)

*Head: encephalopathy (asterixis), temporal wasting, fetor hepatis, conjunctival icterus *Torso: spider angiomata, gynecomastia (in males), caput medusae, ascites *Hands: palmar erythema, Dupuytren's contractures, Terry's nails, asterixis

What spinal cord injuries warrant immediate surgical intervention? (Ch. 43)

*Impingement on spinal cord* by foreign bodies, herniated disks, bony fracture fragments, or epidural hematomas

Describe different classifications of heart failure (Ch 81):

1. Acute vs. Chronic 2. Systolic vs. Diastolic 3. Right vs. Left-sided 4. High-output vs. Low-output

List 4 signs of flexor tenosynovitis: (Ch. 50)

*Kanavel's Signs:* 1. Tenderness along course of the flexor tendon 2. Symmetrical swelling of the finger (sausage finger or fusiform swelling) 3. Pain on passive extension 4. Semi-flexed posture of the finger

NSAIDS related to increased risk of GI side-effects: (Ch. 3)

*Ketorolac* (24x) Indomethacin (3x) Naproxen (2x) Ibuprofen (1-2x)

What is the most common cause of organic anxiety, anxiety that results from a physiologic origin? (Ch 112)

1. Adrenal disorders 2. Alcohol and drug use 3. Cardiac disease 4. Hyperthyroidism 5. Pulmonary embolus

Predictors of difficult intubation? (Ch. 1)

*LEMON* test Look -gestalt (MOANS criteria helpful) Evaluate 3-2-1 rule: 3 fingers for thyromental distance, 2 for mouth opening, 1 for TMJ Mallampati Obstruction or Obesity Neck mobility - ank. spond or RA

List pediatric-specific cardiovascular and abdominal injury patterns associated with classic mechanisms of trauma: (Ch. 38)

*Lap belt use:* -small bowel injury/hematoma -pancreatitis -chance fractures *Bike handlebar injuries:* -duodenal hematoma -pancreatic transection/trauma *Sports related:* -spleen, kidney, intestinal tract

Predictors of difficult BVM? (Ch. 1)

*MOANS* Mask seal (beard) Obesity/Obstruction Age >55 No teeth Stiffness to ventilate (COPD, pregnancy)

Differentiate myocardial concussion, contusion, and rupture: (Ch. 45)

*Myocardial concussion*: "agitation of heart" result of direct blow to anterior chest. Result in non-perfusing rhythm, can be fatal (asystole or VF). *Contusion*: direct injury to cardiac muscle cells; hemorrhage and edema can lead to immediate ventricular dysfunction or delayed pericarditis / scar formation and possible spontaneous rupture. *Rupture*: acute traumatic perforation of atria or ventricles and their surrounding structures (intra-atrial or ventricular septum, chordae, valves, coronary arteries).

List causes of vomiting specific to age groups: infant, newborn, child, and teen. (Ch. 29)

*Newborn*: Obstructive disease, malrotation volvulus, inborn errors of metabolism, serious infection, etc. *Infant*: pyloric stenosis, consider all the infectious causes from head to toe, Hirschsprung's disease, intussusception *Child*: Infections, metabolic diseases, cow's milk intolerance, FTT, abuse, diabetes, appendicitis, gastroenteritis *Teen*: All the common adult causes; PUD, PID, diabetes Porphyria, SMA syndrome, achalasia

How do you distinguish between orbital and peri-orbital cellulitis? (Ch. 22)

*Orbital cellulitis*-specific features:

In a healthy brain, describe how PO2, PCO2, MAP, and ICP all relate to the amount of blood getting to brain tissue (CBF): (Ch. 41)

*PO2* - inverse - if oxygen content decreases, vasodilation and CBF increases *PCO2* - direct - if CO2 content decreases , vasoconstriction and CBF decreases *MAP* - direct - if systemic BP increases, CBF goes up *ICP* - inverse - if ICP increases, CBF declines.

The diagnostic criteria of HSP includes 2 or more of the following (Ch 118)

1. Age < 20 2. Palpable Purpura 3. Abdominal pain 4. Vessel wall granulocytes on biopsy

Risk factors of SIDS: (Ch. 10)

1. Age between 1-12 mo (peak 2-4 mo) 2. Low socioeconomic status 3. Multiple gestation pregnancy 4. Male 5. Prone sleeping 6. Second hand smoke expoure

What are 8 risk factors for central retinal artery occlusion?(Ch. 71)

1. Age: 50-70 2. Hypertension 3. Cardiac disease 4. CAROTID ARTERY DISEASE! 5. Diabetes 6. Collagen disease 7. Vasculitis 8. Valvular heart disease 9. Sickle cell 10. Glaucoma 11. Retrobulbar hematoma 12. Endocrin exopthalmos

Describe basic approach to UNSTABLE pts with acute neck trauma: (Ch. 44)

1. Airway - RSI; r/o hemo/pneumothorax if injury in zone 1 2. Vascular concerns - occlusive pressure and trendelenburg lowers risk of air embolus 3. Cervical spine immobilization if indicated 4. Thoracotomy if cardiac arrest - think venous air embolism 5. NG tube? - not usually if still awake 6. CT head/neck for virtually everyone 7. Consult surgery/ICU **If platysma injury --> admission

List 3 reduction techniques for posterior dislocations (Ch 56):

1. Allis Technique 2. Stimson's Technique 3. Whistler Method 4. Rochester Method 5. Traction counter-traction Method 6. And many others (See: https://www.ncbi.nlm.nih.gov/pmc/articles/PMC4821229/ for an exceptional list of 12 different techniques with figures)

Describe 3 types of incomplete cord lesions: (Ch. 43)

1. Anterior cord syndrome --loss of pain/temp, preserved dorsal columns 2. Central cord syndrome --loss of motor function (legs relatively spared) 3. Brown-sequard lesion --ipsilateral dorsal column and motor loss, contralateral pain/temp loss

Common meds that cause syncope: (Ch. 15)

1. Anti-HTN meds 2. QT-prolonging meds (TCA, amiodaron, sotolol, haldol) 3. Digitalis 4. Insulin/oral hypoglycemics 5. Rec drugs

Describe a general approach to management for suspected infectious diarrhea (Ch 94)

1. Anti-emetics (serotonin, dopamine, histamine and muscarinics all great choices) and oral rehydration (solution with glucose AND electrolytes) - for all! 2. Encourage food intake - fasting will worsen the bowels capacity to absorb food! 3. Antimotility drugs prn (loperamide 4 mg PO prn) Antibiotics (see below) 4. Volume resuscitation prn, electrolyte replacement prn

Three main patho pathways of cardiac arrest in kids: (Ch. 10)

1. Asphyxia (#1 way) 2. Ischemia (hypovolemia, sepsis shock, MI) 3. Arrythmogenic

Describe *relative afferent pupillary defect* (RAPD): (Ch. 21)

*Positive RAPD*- when a pupil dilates with direct beam of light but constricts with consensual response (when performing swinging light test)

What are the 3 key aspects to injury control? (Ch. 40)

*Prevention*: education, public law enforcement *Acute Care*: trauma systems, EMS, guidelines for care *Rehabilitation*: physio, OT, mental health

When to intubate? (Ch. 1)

*Protection* (from aspiration or impending deterioration) *Positive pressure* (failure of ventilation/oxygenation) *Pulmonary toilet* (maintain drainage) *Pharmacy* (administer drugs thru tube)

Differentiate between pulmonary contusion and ARDS: (Ch. 45)

*Pulmonary Contusion*: Often unilateral, appears within minutes to 6hrs post-injury, lasts 24-48hrs *ARDS*: Diffuse and bilateral, slow onset, appears within 24-72hrs

Critical causes of acute dyspnea: (Ch. 25)

*Pulmonary*: --airway obstruction --PE --anaphylaxis --tension pneumothorax --non-cardiogenic pulm edema *Cardiac*: --cardiogenic pulm edema --MI --cardic tamponade *Other*: --toxins (ASA), DKA, epiglottitis, CO poisoning, acute chest syndrome (sickle cell), CaA catastrophe

Predictors of difficult LMA insertion? (Ch. 1)

*RODS* Restricted mouth opening Obstruction/Obesity Distorted anatomy Stiffness to ventilate

List the 5 layers of the scalp and describe 3 types of extra cranial bleeding in pediatrics: (Ch. 38)

*S*-Skin *C*-Connective Tissue *A*-Aponeurosis *L*-Loose Connective Tissue *P*-Periosteum Caput succedaneum: -hematoma, freely mobile and crosses suture lines Cephalohematoma -blood UNDER periosteum = does NOT cross suture lines -scalp bleeding can be profuse and lead to shock in infants Cerebral contusions -clearly seen on CT as a brain parenchymal injury -neuro features with altered MS

Predictors of difficult cricothyrotomy? (Ch. 1)

*SMART* Surgery Mass (abscess, hematoma) Access/Anatomy problems (obesity, edema) Radiation Tumour

Specific considerations for elderly trauma patients: (Ch. 39)

*Vitals:* normal vitals should NOT be reassuring *Airway:* often difficult airway (dentures, stiff neck, obese) faster apneic desat, higher risk of hypotension less RSI meds needed (try to avoid succ) *Breathing:* low FRV, consider high flow Quick to tire with WOB *Circulation:* blunted stress response, less CO reserve previous HTN may mask hypotention as normotension, Pt on anticoagulation? *Disability:* elderly at incr risk of spine # (C, T, L, S) or SCIWORA, watch out for TBI (CT every head) *Exposure:* increased risk of hypothermia (thin skin, less fat/muscle,impaired thermoregulation), remove from backboards ASAP, tetanus UTD?

What extensor zones of the hand are reasonable for ER repair? (Ch. 50)

*Zone VI*- ER repair is reasonable as tendons do not retract significantly due to the synechia. *Zone V*- should be discussed with a hand surgeon - possible in ED but often complicated. R/O human bite. *Zone I/II*- only repair in ED if partial and no extensor lag *Zone I-IV*- otherwise need meticulous repair by hand surgeon.

Appropriate discharge instructions for post-procedural sedation: (Ch. 4)

*must be back to baseline *should NOT drive for 12-24 hrs *should be supervised for 4-8 hrs post

Describe the NYHA function HF Classes (Ch 81, Box 81-2):

1. Asymptomatic on ordinary physical activity. 2. Symptomatic on ordinary physical activity 3. Symptomatic on less than ordinary physical activity. 4. Symptomatic at rest.

What kind of cervical spinal injuries can you get from a FLEXION mechanism? And are they stable/unstable? (Ch. 43)

1. Atlanto-occipital or atlantoaxial joint dislocation (+/-odontoid #) UNSTABLE 2. Simple wedge injury STABLE provided post. column intact 3. Flexion Teardrop UNSTABLE - involveds ant/post. ligament disruption 4. Clay-Shoveler's # STABLE 5. Spinal sublux potentially UNSTABLE -depends on lig. 6. Bilateral facet dislocation UNSTABLE

List ED management goals for patients with anxiety (Ch 112)

1. Avoid escalating behaviour leading to self-harm or harm to others 2. Prevent end-organ dysfunction 3. Improve patient comfort and satisfaction

List 6 lifestyle modifications (Ch 89)

1. Avoidance of high risk foods (caffeine, chocolate, alcohol, fatty, spicy foods) 2. Weight loss* 3. Smoking cessation 4. Elevation of the head of bed, and avoidance of lying down after meals* 5. Exercise 6. Alcohol cessation

Describe the analysis of pleural fluid (Ch 76):

1. Blood - post-op, trauma, malignancy, pulmonary infarct 2. Chyle - trauma or rupture thoracic duct (eg strongyloides) 3. Transudate - increased hydrostatic pressure (heart and liver failure, fluid overload), decreased oncotic pressure (nephrotic syndrome, loosing protein), negative pleural pressure (atelectasis) 4. Exudate - parapneumonic effusion, empyema, subphrenic abscess, pancreatitis

What questions would you ask in a partner violence screen? (Ch 68)

- Have you been hit, kicked, punched or otherwise hurt by someone within the past year? If yes, by whom? - Do you feel safe in your current relationship - Is there a partner from a previous relationship who is making you feel unsafe now?

What questions would you ask in an abuse assessment screen? (Ch 68)

- Have you ever been emotionally or physically abused by your partner or someone important to you? - Have you been hit, kicked, punched or otherwise hurt by someone within the past year? If yes, by whom? - Within the last year has anyone forced you to have sexual activities, if yes who? - Are you afraid of your partner or anyone mentioned above?

Describe the method for reducing a jaw dislocation (Ch. 70)

- Sedation and analgesia - Place thumbs in the buccal sulcus bilaterally, provide downward traction on the mandible which rotating the chin upwards and backwards

List 3 complications of supracondylar fractures (Ch. 52)

1. Brachial artery injury 2. Compartment syndrome 3. Loss of the normal carrying angle. 4. Injury to nerves 5. Stiffness

What are the injury patters in thermal force injuries? (Ch 65)

- Sharp/clear lines of demarcation(Dipping or immersion burns) - Irregular borders(Splash burns) - Round or oval shapes(Hot liquid or cigarettes)

What are the AHA exclusion criteria for fibrinolytic therapy in stroke?

- Significant head trauma or prior stroke in previous 3 months - Symptoms suggest SAH - Arterial puncture at noncompressible site in previous 7 days - History of previous intracranial hemorrhage - Intracranial neoplasm, AVM or aneurysm - Recent intracranial or intraspinal surgery - Elevated blood pressure - Active internal bleeding - Acute bleeding diathesis, including but not limited to: Platelet count <100,000, heparin in last 48 hours, current anticoagulant with INR > 1.7 or PT >15s, current use of direct thrombin inhibitor - Glucose <50mg/dl - CT demonstrates multilobar infarction Relative - Improving stroke symptoms - Pregnancy - Seizure at onset with postictal residual neuro impairments - Major surgery or serious trauma in last 14 days - Recent GI or urinary tract hemorrhage - Recent acute myocardial infarction

Which patients with AECOPD require admission (Rosen's box)

- Significant worsening of symptoms from baseline - Inadequate response to ED management - Significant comorbid condition (pneumonia, CHF) - Worsening hypoxia or hypercarbia from baseline - inability to cope at home or lack of home resources

Describe the commonly inflicted injury patterns in blunt force trauma, and what might cause these injuries (Ch 65)

- Slap marks with digits delineated - Looped or flat contusions(Belts or cords) - Circular contusions(Fingertip pressure) - Parallel contusions with central clearing from linear objects(Bat) - Contusions from shoe heels and soles - Semicircular contusions and abrasions from bite marks - Lacerations

List Indicators of upper airway burns(Ch. 63)

- Soot in and around nose/mouth - Charring - Mucosal inflammation or necrosis - Edema

What risk factors make domestic violence more common?(Ch 68)

- Stalking and harassment - Estrangement (physical or legal separation) within a family unit - Access to firearms - History of forced sex - History of violence during pregnancy

List 4 components of treatment of Bell's palsy and with whom should we consider lyme disease?(Ch. 105)

- Steroids!(Ex. prednisolone 50-60mg x 10 days) - Antivirals for severe disease(Evidence poor)(Valacyclovir 100mg TID x 7 days) - Eye care -> need artificial tears - Physiotherapy( evidence poor) Consider lyme disease if - Systemic symptoms - bilateral facial paralysis

Describe facial weakness associated with bell's palsy and list 3 other associated symptoms(Ch. 105)

- Sudden onset - Eyebrow sagging - Inability to close eye - Disappearance of nasolabial fold - Drooping at the affected corner of the mouth, which is drawn to the unaffected side - Decreased tearing - Hyperacusis(Increased sensitivity to certain sounds and frequencies) - Loss of taste on anterior two thirds of tongue

Describe the symptoms of vestibular schwannoma. What is the diagnostic test of choice?(Ch. 105)

- Sudden onset sensorineural hearing loss is classic, may have tinnitus ENT --> MRI w/ gad

List the "One pill can kill" drugs(Ch. 147)

- TCA - CCB - Methadone and other opiates - Oral hypoglycemics - Chloroquine - Clozapine

What is your toxicologic differential for wide complex QRS(Ch. 147)

- TCA's - Type 1a, 1c dysrhythmics: Quinidine, procanamide, flecainide, propafenone - Cocaine - Diphenhydramine

What is the management of an avulsed tooth?(Ch. 70)

- Tooth should be placed in saline or Tooth Saver solution - Tooth should be replanted in the socket ith firm pressure until it clicks into place

List 4 causes of superficial punctate keratitis(Ch. 71)

- UV burns, conjunctivitis, topical eye drop toxicity, minor trauma, contact lens use, blepharitis, mild chemical injury

What are the most common affected parts of the body in burn injuries?(Ch. 63)

- Upper extremities(41%) - Lower extremities(26%) - Head and neck(17%)

List the important diagnostic studies for the workup of delirium(Ch. 104)

- Vital signs! - Glucose - CBC - Electrolytes, extended lytes, Crt, BUN, osmolality - UA - XR - CT head - Maybe: ABG, drug levels, trop, LFT, TSH, B12, folate, heavy metals, toxic alcohols, MRI brain, LP, EEG

What are common reactions to NAC? How are these managed?

- Vomiting and mild anaphylactic - Reduce rate of transfusion and antihistamines

What is the tx for bacterial conjunctivits?(Ch. 71) What are some complications?

- Warm compress - Topical opthalmic abx - No steroids!!!! Complications: Corneal ulcers, keratitis, corneal perforation

List indicators of lower airway burns(Ch. 63)

- Wheezing - Crepitations - Hypoxemia - Abnormalities on CXR - VQ mismatch, decreased lung compliance, microatelectasis --> ARDS

Which type of anal fissure is suspicious for underlying disease? (Ch 96)

A fissure anywhere other than posterior midline (or anterior midline in women) should make you suspicious for: Leukemia Crohn's disease HIV infection TB Syphilis

What is a supracondylar fracture in a child? (Ch. 52)

A fracture of the distal humerus, proximal to the epicondyles

What the pathophysiology of sinusitis? What are the typical pathogens? (Ch 75)

A healthy sinus - is sterile; with free air exchange and mucous drainage Ostial obstruction leads to sinusitis: Due to: Viral / allergic / ciliary paralysis (smokers) Leads to bacteria introduction by coughing, nose blowing → then overgrowth and infection Pathogens: Strep. Pneumoniae Nontypable H.influenzae M. catarrhalis Pseudamonas Strep. Staph. Fungi

Define status epilepticus and refractory status epilepticus(CH. 102)

A seizure lasting longer then 5 minutes OR recurrent seizure activity without intervening return to baseline mental status Refractory: A seizure that does not terminate after treatment with a benzo PLUS a second antiepileptic.

List 8 DDx for appendicitis (Ch 93)

A simplified table summarizing categories: Note the list can be extensive!

How do you identify a herpes simplex infection in the eye? What is the Treatment? (Ch. 71)

Dendritic pattern, ulcer or superficial puctate keratitis on slit lamp exam FB sensation, tearing, photophobia Tx: Trifluridine 1% q2hrs x 14 days. Erythromycin ointment q4h. Cyclopentolate No steroid! Optho follow up

Carpal-metacarpal & MCP dislocation... describe management. (Ch. 50)

Dense specifics - best to look up the shownotes, Ch. 50.

Gamekeeper's / Skiier's thumb... describe management. (Ch. 50)

Dense specifics - best to look up the shownotes, Ch. 50.

Grade III DIP/PIP ligament disruption + dislocation... describe management for each. (Ch. 50)

Dense specifics - best to look up the shownotes, Ch. 50.

Volar plate injuries... describe management. (Ch. 50)

Dense specifics - best to look up the shownotes, Ch. 50.

Describe the ER management of poorly controlled HTN (Ch 84)

Depends on patient's follow-up: If they have a GP or internist = nothing to be done in ED If they don't have follow-up, Rosen's suggest initiating antihypertensive agent and arranging follow-up is possible However, quote from Rosen's : "Acute BP reduction in the ED provides absolutely no benefit to patients with chronic HTN and exposes them to unnecessary risk of potential hypoperfusion in regions where blood flow has been governed by long-standing autoregulation."

Describe 1st line medical therapy for depression and bipolar disorder (Ch 111)

Depression: SNRI / SSRI (plus psychotherapy!) e.g. citalopram 20 mg daily Bipolar disorder: Antipsychotic (2nd gen) + Mood stabilizer (either Lithium, Valproic acid, Carbamazepine).

When do you start NAC in chronic ingestion? (Ch. 148)

Chronic: Any ingestion > 8 hour period - Toxic level(>7.5g in an adult) - Symptomatic regardless of APAP level - Elevated AST(>2x upper limit of normal) - Elevated APAP > 30mc/ml - CONTROVERSIAL: All high risk patients with elevated AST(ETOH/ liver disease)

What are the two types of arteriovenous (AV) fistulae used for dialysis? (Ch 87)

Cimino-Brescia fistula Preferred fistula for long term hemodialysis. Usually radial artery and cephalic forearm vein anastomosed Longest functional AV fistula Prosthetic bridge fistula "H"-shaped, with a prosthetic conduit connecting vein and artery.

List two antibiotic options for uncomplicated pyelonephritis (Ch 99)

Ciprofloxacin 500 mg bid 7 days GI disturbance, headache, dizziness, tremors, restlessness, confusion, rash, Candida infections Levofloxacin 750 mg once daily 5 days Same as for ciprofloxacin Trimethoprim-sulfamethoxazole 160/800 mg bid 10-14 days Nausea, vomiting, anorexia, hypersensitivity reactions In places with >10% fluoroquinolone resistance: give 1 g ceftriaxone followed by 10-14 days of an oral cephalosporin

What is the typical dose for empiric abx coverage in diarrhea? (Ch. 31)

Ciprofloxacin 500mg PO BID x 3-5 d Not recommended in pregnant women and age <18

What are the indications for an escharotomy(Ch. 63)

Circumferential chest or neck burns with: - Increased airway pressures Hypoxemia - Difficulty with ventilation Circumferential extremity burn with: - Decreased doppler signal distally - Pulse oximetry of less than 90% distally in the limb (one study) - Pain, loss of sensation, delayed cap refill (early signs)

What are some signs and symptoms of Central Retinal Artery Occlusion? (Ch. 71)

Classic is PAINLESS vision loss over SECONDS Signs: Reduced VA, prominent RAPD, pale edematous fundus with a cherry red spot

What are the clinical features used to diagnose AECOPD?

Classic patterns: -blue bloater (polycythemia, cor pulmonale from chronic obstructive bronchitis) - pink puffer (V/Q mismatch from emphysema ...increased RR) But just look for: -Cough with/without expectoration -Increased WOB or air hunger -Wheeze -ALOC "Irritable Somnolence" (hypercapnea: often has asterixis with it) -Right heart failure: JVD and peripheral edema

What is Beck's Triad? (Ch 82)

Classical Triad of Cardiac Tamponade: JVD; hypotension, muffled heart sounds (Pro tip: Also look for electrical alternans or pulsus paradoxus).

What is a concussion? (Ch. 41)

Complicated minor traumatic brain injury, most commonly from collision sports. Leads to short-lived distortion of axons with neurotransmitters in an elevated/hypermetabolic state for WEEKS post-injury. This can cause secondary damage if not allowed to heal.

What are the diagnostic criteria for endocarditis, and how are they used? (Ch 83)

Duke Criteria (Refer to Box 83-1) Mnemonic: BE FIVE PM Blood culture positive for IE Endocardial involvement Fever Immunologic phenomena Vascular phenomena Echocardioraphy minor criteria (eliminated) Predisposition Microbiologic evidence

Difference between fever and hyperthermia: (Ch. 12)

Fever= caused by increase in body's SET-POINT, rarely goes beyond 41 degrees C (body autoregulates) Hyperthermia= caused by inability to dissipate sufficient heat. Temp can increase indiscriminately until there is multi-system failure and death.

Describe depth classifications for burn injuries(Ch. 63)

First degree(Superficial): Limited to epidermis. Painful, erythematous, dry surfeace Second Degree(Superficial) - Papillary dermis - Painful, erythematous, blisters, blanches with pressure, moist surface Second Degree(Deep) - Reticular dermis - Painful, white or mild erythema, hemorrhagic blisters, minimal blanching, dry or moist surface Third degree - Full thickness - entire dermis - Stiff, white or tan color, dry, leathery and insensate Fourth degree - Involves muscle or bone - Stiff, charred skin

Describe the decontamination of an individual who is exposed to a chemical agent(Ch. 64)

First responder uses a scene survey approach Attempt to determine which substance(s) may be involved Don appropriate PPE (personal protective equipment) Safely remove the victim from the hostile environment ALL the victim's clothing should be removed and placed in plastic bags Dry agents should be brushed off; Wet agents should be sprayed off copiously Priority: eyes, mucous membranes, skin, hair Clean until a normal skin pH

List 5 burn prevention strategies. (Ch. 63)

Flame burn prevention - Secure matches and lighters where they are inaccessible to children - Safety device around fireplace - Create home escape plan and practice with family - Check that smoke detectors are functional Scald prevention - Use splash guards on stove - Use thermometer on bathwater - Lower hot water heater maximum temperature to 49-54 C

Define Todd's paralysis(Ch. 102)

Focal motor deficit after seizure, can last up to 24 hours, caused by cerebral hypoperfusion. HIGH liklihood of structural abnormality

List different types of Ovarian Cysts (Ch 100)

Follicular cyst 1st half of menstrual cycle MOST COMMON type of cyst Pathologic when > 3.0 cm Thin-walled & filled with clear fluid Corpeus luteum cyst 2nd half of menstrual cycle Often filled w/ hemorrhagic fluid Benign Cystic Teratoma (dermoid cyst) Endometriomas (chocolate cysts), Fibroma Cystadenoma various types of malignant neoplasms

List 4 etiologic agents causing anaphylaxis by immunologic mechanisms (Ch 119)

Foods: Egg, peanut, tree nut, milk, fruits, shellfish, soybean, sesame Medications: Antibiotics, NSAIDs, chemotherapeutic agents, immunomodulators Insect stings: Hymenoptera venoms, fire ant stings Natural rubber latex Hormones: Insulin, methylprednisolone, parathormone, estradiol, progesterone, corticotropin Local anesthetics: Mostly ester family (procaine, tetracaine, benzocaine) RCM (radiocontrast) Occupational allergens: Enzymes, animal protein, plant protein Aeroallergens: Pollen, dust, spores, pet dander

Canadian C-Spine Rule: (Ch. 43)

For all alert (GCS=15) and stable trauma patients where cervical spinal injury is a concern, the following can be applied:

Describe EtCO2 curve... (Ch. 5)

Four phases of typical capnograph correlate to: 1) rest state 2) expiration 3) end expiration 4) inspiration

List 8 DDx for DVT (Ch 88)

Fracture (stress / occult / pathologic) Popliteal cyst / rupture of Cellulitis Superficial thrombophlebitis Vasculitis Proximal venous compression (tumour, gravid uterus) CHF related bilateral leg swelling Hypoalbuminemia Lymphedema Muscle strain / calf strain Hematoma Chronic venous insufficiency

What is an alveolar ridge fracture? (Ch. 42)

Fracture through the ridge of bone that forms the sockets for the teeth (the dental alveoli) Requires maxillofacial surgeon

When do the sinuses typically develop? (Ch 75)

Fully develop by age 10.

What does one need to have consciousness? (Ch. 41)

Functioning RAS (in brainstem) and cerebral cortices.

Define Generalized Anxiety Disorder (GAD) (Ch 112)

GAD is defined as excessive worry that occurs most days over a 6-month period involving several events or activities. The anxiety must cause significant distress or impairment in functioning.

What are poor prognostic factors for ICH? What is a short formula for calculating ICH volume? (CH. 101)

GCS 3-4(2 points) 5-12(1 point) 13-15(0 points) ICH volume >30(1 point) <30(0 points) IVH yes(1 point) no(0 points) Infratentorial origin of ICH yes(1 point) no(0 points) Age >80(1 point) <80(0 points) Total 0-6 Score - mortality rate 0-0 1-13% 2-26% 3-72% 4-94% 5/6- 100% "The computed tomography (CT) slice with the largest area of hemorrhage is identified. The largest diameter of the hemorrhage on this slice is measured in centimeters (line A). The largest diameter 90 degrees to A on the same slice is measured (line B). C is the approximate number of 10-mm slices on which the intracerebral hemorrhage (ICH) was seen. (Many centers use 5-mm slices, in which case an adjustment can be made by dividing by 2.) The volume of the hemorrhage = A × B × C ÷ 2 (ABC/2)."

Most acurate way of measuring core body temperature? (Ch. 12)

GOLD STANDARD: temp probe on pulm artery catheter NEXT BEST: esophageal, bladder, or rectal probes Bottom rung: axillary and tympanic often under report and oral temps are highly biased by smoking, hyperventilation, or consumed liquids.

Describe a black widow spider and where you might find one. (Ch. 55)

Glossy black color and bright red marking on the abdomen - either and hour glass or two red spots. These species frequently take cover in protected placed such as rocks, woodpiles, stables and... outhouses

How are vascular injuries diagnosed in the ED? (Ch. 48)

Good history and clinical suspicion BP/pulse palpation comparison between sides Examine for expanding hematomas and bruits ABI/API Hand-held Doppler Duplex Ultrasound CT with contrast enhancement

List 8 acute complications of seizures(Ch. 102)

Hypoglycemia Neurogenic pulm edema Skeletal muscle damage Rhabdomyolysis Autonomic discharge: incontinence, vomiting, tongue biting, airway impairment Shoulder dislocation or fractures

List 5 DDx of aniscoria. Which is greater in dark and which is greater in light?(Ch. 71)

In lighted setting: Problem pupil is the larger one In dark setting: Problem pupil is the smaller one Ddx: ADIE tonic pupil syndrome, pharm induced mydriasis, third nerve palsy, physiologic aniscoria, Horner's syndrome

In approximately what percentage of anaphylactic cases for adults and children respectively is an inciting agent identified? (Ch 119)

In up to 60% of adults and 10% of children - no inciting agent is identified!

List 15 causes of delirium(Ch. 102)

Drugs - ETOH - Rec drugs - sedatives/ hypnotics - Dig/ lithium/ quinidine/ salicylate/ antiepileptics - Anticholingergics!!! Infection Metabolic - Hypo/hypernatremia - Hyperosmolarity - Hypercapnia - Hypoxia - Hyperglycemia - uremia - liver failure Envirmental - Hyper/hypothermia Structural

List the 3 neurotransmitters implicated in depression (Ch 111)

Low levels of these in the synapse: Serotonin Norepinephrine Dopamine Also implicated: Glutamate, GABA

How do you treat central retinal vein occlusion? (Ch. 71)

Lower IOP(Timolol, acetazolamide) Topical steroid Intraocular anti vascular growth factor LMWH

How is the rule of 9s modified in pediatrics?(Ch. 63)

Lund-Browder chart

What is a luxed tooth? How are these managed? (Ch. 42)

Luxation = loosening of or displacement of the tooth in the socket, without avulsion Management = repositioning + splint and dental referral

What is a luxed tooth?(Ch. 70) What is the management?

Luxation of a tooth is loosening of or displacement of the tooth in the socket, without avulsion Repositioning, splinting and follow up with a dentist

List 10 causes of non-traumatic spinal cord dysfunction(Ch. 106)

Intrinsic - MS - Transverse myelitis - Spinal arteriovenous malformation, SAH - Syringomyelia - HIV myelopathy - Spinal cord infarction Extrinsic(Compression) - Spinal epidural hematoma - SEA - Diskitis - Neoplasm - Metastatic - Primary CNS

What is Wallenburg's syndrome? (Ch. 43)

Loss of PICA circulation to the brain PICA supplied by vertebral artery. Results in ischemia to lateral medulla = swallowing difficulty, or dysphagia, slurred speech, ataxia, facial pain, vertigo, nystagmus, Horner's syndrome, diplopia, ataxia

Define neurogenic shock (hypotension): (Ch. 43)

Loss of vasomotor tone and lack of reflex tachycardia from disruption of autonomic ganglia. Only consider if: --Flaccid and areflexic --Reflex tachycardia and peripheral vasoconstriction absent --Exclusion of hemorrhage, tamponade, and tension pneumothorax

What is the pathophysiology of vasculitis? (Ch 118)

Rosen's, "The cause of most vasculitis syndromes is unknown". Most cases are believed to result from immune complex deposition in blood vessel walls, prompting a complement-mediated inflammatory reaction. This results in vessel wall damage and necrosis, leading to stenosis, occlusion, and subsequent end-organ ischemia. The clinical manifestations are determined predominantly by the size and distribution of blood vessels involved along with the histologic subtype of inflammation."

What kind of cervical spinal injuries can you get from FLEXION/ROTATION forces? And are they stable/unstable? (Ch. 43)

Rotary atlanto-axial dislocation - UNSTABLE Unilateral facet dislocation - STABLE

Describe the signs of traumatic iridocyclitis (uveitis). What is the treatment? (Ch. 71)

Signs: Perilimbal conjunctival injection, cells in the anterior chember, flare, non-dilating pupil, direct and consensual photophobia Tx: Long acting cycloplegic, prednisolone

Describe the physical findings of globe rupture and describe management. (Ch. 71)

Signs: bloody chemosis / severe subconjunctival hemorrhage / tear drop pupil RAPD / poor VA / no red light reflex Treatment: Eye shield, head of bed >45 degrees, NPO, antiemetics, analgesics, antitusives, broad spectrum IV abx

What is the presentation of a posterior vitreous detachment? What treatment is needed? (Ch. 71)

Similar symptoms to retinal detachment and vitreous hemorrhage. Floaters, flashing lights, vision loss Tx: No specific treatment, but needs optho consult to differentiate between other conditions!

What are symptoms and signs of retinal detachment? What is the treatment?(Ch. 71)

Sx: Flashes of light, floaters, visual loss, PAINLESS, visual field cuts with varying visual loss Signs: CANNOT be ruled out with fundoscopy --> can BEDSIDE U/S! Tx: Optho call! needs surgical management ideally in 24-48 hours

What are some signs and symptoms of primary angle closure glaucoma?(Ch. 71)

Sx: Sudden severe eye pain, blurred vision, headache, N/V, abd pain. HALO AROUND LIGHTS Signs: Hazy cornea, mid- dilated pupil, conjunctival injection, steamy cornea, increased IOP

Describe your management for cracked tooth/ split root syndrome (Ch. 70)

Symptomatic analgesia (Tylenol, NSAIDS) Refer to Dentist

List clinical presentations of diverticular disease (Ch 95)

Symptoms: Bloating Crampy pain Excessive gas Change in bowel habits 10-30% of people progress to diverticulitis (West = left side, Japan = right side)

What are some signs and symptoms of macular disease? What causes this disease? When does opthamology need and urgent referral?(CH. 71)

Symptoms: Central vision loss with preserved peripheral vision. Gradual to rapid visual loss Signs: Scattered drusen leading to car and detachments Causes: Degenerative(MOST COMMON!) radiation, vascular, infectious, idiopathic Urgent referral: Infectious or inflammatory causes

What is a common symptom associated with CN III, IV, VI nerve palsies? What is the differential for monocular EOM disorder? Binocular? (Ch. 71)

Symptoms: Diplopia Monocular: Refractive error, dislocated lens, iridodialysis, feigned Binocular: Disappears with either eye covered. Hematoma, orbital floor fracture, abscess, thyroid disease, progressive opthalmoplegia, EOM muscle fibrosis, MS, myasthenia gravis. Nerve palsy!(Most common)

What are some signs and symptoms of temporal arteritis? What is the workup? What is the treatment? (Ch. 71)

Symptoms: Weight loss, fevers, jaw pain, scalp tenderness, malaise, headache, visual loss, painless visual loss, age >50, hx of PMR SIgns: RAPD, visual loss, visual field defect, pale optic disc Tests: Elevated ESR(Corrected for age age/2; age+10/2 for women). TA biopsy Tx: High dose steroids AS SOON AS SUSPECTED. Biopsy in one week

List 6 ddx of somatic symptom disorder (Ch 113)

The list is innumerable (see Rosen's Box 103.2). A few to list: Differential Diagnosis of Somatic Symptom Disorder Major depressive disorder Anxiety disorders Multiple sclerosis Porphyria Hyperparathyroidism Systemic lupus erythematosus Thyroid disease Wilson's disease Substance abuse disorder Personality disorder Malingering

What is the dosing strategy for rt-TPA(CH. 101)

The recommended dose for rtPA is 0.9 mg/kg IV to a maximum of 90 mg (10% of the dose given as a bolus followed by an infusion lasting 60 minutes).

Indication for CPR in a neonate: (Ch. 11)

When HR <60 despite adequate ventilation for 30 sec

How long will an acute gout attack last without treatment? (Ch 116)

Without treatment, the attack is self-limited, peaking during 24 to 48 hours and lasting about a week.

What are two concerning presentations for intimate partner violence? (Ch 68)

Woman with injuries to head face or neck (MC injury type) Female patient who has attempted suicide (90% of hospitalized suicide attempts in women report current severe IPV)

What imaging do you do for scaphoid fracture w/u? (Ch. 51)

X-ray (15% DON'T SHOW FRACTURE immediately after injury)... get scaphoid views CT scan 93% sensitive for occult fracture in acute setting. MRI 100% sensitive

Are tetanus shots safe for mom in prenancy? (Ch. 37)

Yes.

Can you safely electircally cardiovert a pregnant woman? (Ch. 37)

Yes. No harm to fetus shown with shocks up to 300J.

Should you give steroids in the acute treatment of gout? (Ch 116)

Yes. Once Septic Arthritis is ruled out! Intra-articular injection: Triamcinolone acetonide 20 to 40 mg for the knee 5 to 10 mg for smaller joints is

Classification for finger-tip amputations: (Ch. 50)

Zone I: proximal ⅔ of the nail bed is preserved, no bony involvement Zone II: exposed bone Zone III: entire nailbed is lost

Define massive transfusion: (Ch. 7)

infusion >10 units of blood in 24hrs

For activated charcoal what is your dosing strategy? When can it be given? For what substances is it useful? What substances is it not useful? (Ch. 147)

historically 0.5-1.0 gram/kg Think about potentially 10:1 ratio of charcoal to drug Rule of thumb: 1g/kg typically 50grams/ patient Needs to be less then 1-2 hours!, large ingestion, be absorped by charcoal and the patient but be willing to take it!(Think GCS and patient willingness) Lethal toxins when activated charcoal may be indicated(Killer Cs) - Cyanide - Colchicine - CCB - Cyclic antidepressants - Cardio glycoside - Cyclopeptide mushrooms - Cocaine - Cicutoxin - Salicylates Overdoses when charcoal is NOT recommended PHAILS Pesticides Hydrocarbons/ heavy metals Acids/alkali/alcohols Iron Lithium Solvents Of note: Rosen's does not recommend charcoal in routine use, but does recommend it in certain scenarios

Most lethal sequelae of massive hemoptysis?? (Ch. 24)

hypoxia (V/Q mismatch)

Good NIPPV (CPAP) settings... (Ch. 2)

iPAP = 10 cm H2O ePAP = 5 cm H2O increase each by 1 cm at a time max IPAP is 20 H2O

List meds amenable to multi dose charcoal(Ch. 147)

ie: Gut hemodialysis, draws the drug from circulation, decreases the amount of unbound drug in the gut lumen ABCDQ Aminophylline/ theophylline Barbituates Concretion forming drugs; Salicylates or carbamazepine Dapsone Quinine DOse: 0.5-1g/kg then 50% of initial dose every 4 hours

List 5 indications for optho consult for eyelid lacerations. (Ch. 71)

lac of the lid margin lac. of the canalicular system (medial eyelid) lac. involving the levator or canthal tendons lac. through orbital septum presence of orbital fat*** = no subcutaneous fat in the eyelids so the fat is likely from a globe injury lac. with tissue loss

Describe the management of dental caries (Ch. 70)

locate tooth by percussion or having patient bite down on tongue depressor. NSAIDS are mainstay of Tx. Consider local/regional blocks for rescue therapy

What are indications of surgical repair for an orbital floor fracture? (Ch. 71)

persistent diplopia +/- loss of visual acuity cosmetic concerns that persist after 7-10 days when swelling has subsided. Don't need "in ER" consultation. can be seen in f/u in 1-2 weeks Consider admission and quicker consultation if the fracture extends through an infected sinus

Mechanisms of anterior urethral injuries: (Ch. 47)

straddle injuries falls gunshots amputations self-instrumentation

*Red flags* on history for headaches: (Ch. 20)

sudden onset worst headache of my life (esp in those w/o hx of h/a) refractory headaches headache onset during exertion HIV or immunocompromised headache upon waking altered mental status + headache meningismus unexplained fever focal neurological signs

List indications for thrombolysis in PE, what is the risk of ICH? (Ch 88)

tPa: use is controversial Usually reserved for massive PE (diagnosed on CT-scan): Systolic BP < 90 mmHg for > 15 mins Or a reduction of ~> 60 mmHg from baseline Case by case use: in cardiac arrest Extensive clot burden RV dysfunction Hypoxemia Clot in transit Alteplase: 15 mg bolus, then 2 hr infusion of 85 mg. (then start heparin)

What are the soft signs of peripheral vascular injury? (Ch. 48)

pallor/distal cyanosis soft pulses in comparison to other side superficial vein congestions poor temperature

Wounds at high-risk for vascular trauma based on mechanism? (Ch. 48)

penetrating injuries (esp near neurovasc bundle) animal bites fractures crush injuries/dislocations

Clinical progression of local anaesthetic toxicity: (Ch. 4)

tingly lips --> dysarthria --> lightheaded --> H/A --> tinnitus --> dec LOC --> confusion --> seizure --> coma --> cardiac collapse

Describe your management for Neuropathic pain (Ch. 70)

Often Tic doulourex / trigeminal neuralgia - consider Carbamazepine, referral to pain specialist / GP Migraines & Cluster headaches - standard Tx +/- High flow O2 Don't miss Myocardial Ischemia, Giant cell arteritis (Temporal arteritis), and Polymalgia Rheumatica

What are Pousseils Law and LaPlaces Law (Ch 81):

Poiseuilles law: Flow is directly proportional to the fourth power of the vessel radius. Bigger is better Laplace's law: The larger the ventricular cavity the more mural tension (myocardial work) is needed during contraction

What is the most common site for arterial aneurysm in the leg? (Ch 87)

Popliteal aneurysms Often bilateral! And associated with an AAA Second most common = femoral aneurysms Diagnosed with CT / ultrasound

List 4 causes of spinal epidural hematoma

Post trauma - LP - Epidual anesthesia - Spinal surgery Coagulopathy - Anticoagulated - Thrombocytopenia - Liver disease or alcoholism Spontaneous - SPinal or dural AVM - Vertebral hemangioma

Define 1° heart block. List three causes. (Ch 79)

Prolonged conduction, but all impulses conducted PR > 0.2 sec, normal QRS Etiologies: Can be normal in up to 2% of ppl Ischemic heart disease Athletes

How are prostaglandins used the setting of GI disorders? (Ch 89)

Prostaglandins protect the gastric mucosa! (Inhibit acid secretion, increase mucous/bicarbonate/stimulate blood flow). Misoprostol (an analogue of prostaglandin) - used for high risk patients - to prevent progression of NSAID-induced ulcers.

How do you determine between true subluxation and pseudosubluxation at C2-3 in kids? (Ch. 38)

Pseudosubluxation of the c-Spine at C2-3 is common in kids *Swischuk line*: spinolaminar line of C1-3, If the line crosses C2's anterior cortical margin by less than 2 mm, (and no cervical soft tissue swelling and no fracture is seen) the image demonstrates pseudosubluxation. If not... true sublux

What is a pterygium? What are some risks of developing?What is a pinguecula?(Ch. 71)

Pterygium - Wedge shaped area of conjunctival tissue on the nasal side of the sclera extending onto the cornea - Exposure to UV light, surfing Pinguecula White/ yellow flat/ raised tissue next to cornea --> NO EXTENSION INTO CORNEA

Special consideration regarding fevers in the elderly, chronically ill, or immunosuppressed: (Ch. 12)

Pts response to prostaglandins can be BLUNTED. They may not mount the expected febrile response. Often are COLD.

What is the most common significant injury in blunt chest trauma? (Ch. 45)

Pulmonary contusions (30-70% of cases)

List cardiovascular clinical features of AKI (Ch 97)

Pulmonary edema Arrhythmia Hypertension Pericarditis Pericardial effusion Myocardial effusion Pulmonary embolism

List 5 indications for emergency dialysis (Ch 97)

Pulmonary edema Severe uncontrollable hypertension Hyperkalemia Other severe electrolyte or acid-base disturbances Some overdoses Pericarditis (possibly)

What are 6 objective findings of severe asthma

Pulse rate: >120(In adults) Resp rate: >40 Pulsus paradoxus: >10, but may be absent in 50% of severe asthma Combination of pulse >120, RR >20, PP >10 Use of accessory muscle ABG: PsO2 <60 or PaCO2 >42 Pulm function studies: Best test to define severity

List markers of poor prognosis in patients with PE (Ch 88)

Pulseless electrical activity (fully occluded pulmonary artery) Arterial hypotension Worsening respiratory distress Clot in transit Syncope or seizure in the ED Hypoxemia Evolving RBBB Multiple comorbidities RV dusyunfction (on Echo, troponin, or BNP) Rosen's states that 8% of patients with a PE in the ED will die within 30 days.

What are the acute complications of aortic dissection? (Ch 85)

Put into the simplest way, as only LITFL can (see here): Remember: there are essentially 3 ways blood can get into the media Atherosclerotic ulcer leading to intimal tear Disruption of vasa vasorum causing intramural haematoma De novo intimal tear So, the complications make sense then: Extension up or down Rupture Vessel branch occlusion Aortic regurgitation Pericardial effusion / tamponade 80% of aortic dissections are in non-aneurysmal vessels"

What are risk factors for lens subluxation and dislocation? Symptoms? Signs? (Ch. 71)

RF: Marfan, homocystinuria, tertiary syphilis, connective tissue disease Symptoms: Monocular diplopia, visual distortion, blurry vision Signs: dec VA, lubluxed lens after dilation, shimmering iris

List 5 risk factors for spinal epidural abscesses and list 4 common bacterial causes(Ch. 106)

RFs - Diabetes - IV drug use - Chronic renal failure - Alcoholism - Immunosuppression - Recent infarction Common bacteria - Staph aureus(>50%) - Aerobic and anaerobic streptococci - E. Coli - Pseudomonas aeruginosa

Describe the three different mechanisms of retinal detachment (Ch. 71)

Rhegmatogenous: Tear or hole in the neuronal layer leading to fluid from vitreous cavity leaking between layers Exudative retinal detachment: Fluid or blood leakage from vessels within the retina to cause tear. Caused by HTN, toxemia of pregnancy, CRVO, glomerulonephritis, papilledema, vasculitis, choroidal tumor Traction retinal detachment: Fibrous band formation in the vitreous and contraction of these bands from prior vitreous hemorrhage

List the most common viral causes of sore throat: (Ch. 23)

Rhinovirus Adenovirus HSV 1&2 EBV Influenza

List the reversible causes of CKD (Ch 97)

Reversible Factors Hypovolemia CHF Pericardial tamponade Severe hypertension Catabolic state, protein loads Nephrotoxic agents Obstructive disease Reflux disease

What are 4 risk factors for the develop of symptomatic haemorrhoids? (Ch 96)

Risk Factors: 1. Family hx of hemorrhoids 2. Straining and constipation 3. Pregnancy & traumatic deliveries 4. Prolonged sitting 5. Heavy lifting 6. Portal hypertension (technically these are rectal varices though!)

Red flags in sore throat: (Ch. 23)

rapid worsening toxic appearing, dehydrated drooling hypoxia muffled voice dehydration trismus uvular deviation exudates

What types of NSAIDs are more toxic?(CH. 149)

- Pyrozolone(Rarely used -> withdrawn from market) and fenamate

What is the tension pneumothorax clinical TRIAD? (Ch. 45)

1. JVD 2. Absent breath sounds unilaterally 3. Tachycardia

What is the triad of normal pressure hydrocephalus(Ch. 104)

- Progressive dementia - Ataxia - urinary incontinence

What is the pathophysiology of diverticular disease? (Ch 95)

"An affliction of middle age and Western civilization" - unknown before 20th century, etiology related to ?removal of the outer fiber on wheat kernels by the milling process. Supposedly 80% of people over 85 have diverticula. The colon wall has weak points where the vasa recta run, it is thought that chronic low fiber stools (causing the colon to stressfully process stool), chronic inflammation, and altered bowel flora lead to diverticula. These weak points are prone to herniation of the colonic mucosa - usually small, but can be > 1cm.

What is food poisoning? (Ch 94)

"Classic food poisoning manifests usually 1 to 6 hours after the ingestion of preformed toxins from bacterial organisms such Staphylococcus, B. cereus, or C. perfringens...Food poisoning is generally short-lived (24 hours), and its treatment is generally supportive care only." - Rosen's 9th edition Pearl: Scombroid poisoning IS NOT AN ALLERGIC reaction. It results from eating spoiled dark meat from fish with toxin complexes that work via histidine decarboxylase activity; your patient just consumed straight histamine!

Define the cholinergic toxidrome. what physical exam findings would you see. What substances give you this presentation? What is the treatment? (Ch. 147)

"Fluids....EVERYWHERE" Signs - No change in vitals - Pinpoint pupils - ++++ diaphoresis - Diarrhea - Urination - Emesis - Lacrimation - Salivation - Lethargic! Substances - Organophosphates and carbamate insecticides, nerve agents, nicotine, pilocarpine, physostigmine, edrophonium, bethanechol, urecholine, liquid form e-cigarettes Tx - Atropine - 2-pam(2-pyridine aldoxime methyl chloride) - Benzos - Supportive

List at least 6 hard and 6 soft signs of penetrating neck trauma: (Ch. 44)

"HARD SIGNS" -very high likelihood of vascular injury

List 8 unstable cervical neck fractures: (Ch. 43)

"Jumping Off Awnings Nearly Warrants Frank Spinal Trauma" 1. Jefferson 2. Odontoid 3. Atlantoaxial dislocation 4. Neural arch (posterior) 5. Wedge 6. Facet dislocation (large/bilateral) 7. Subluxation 8. Teardrop

List 8 causes of 2° spontaneous PTX (Ch 77):

"LUNG" Lung disease COPD - this is the #1 risk factor, and accounts for >70% of cases Asthma, CF Unusual stuff Sarcoidosis Marfans Endometriosis Neoplasms Growing infections TB, PCP, necrotizing pnuemonia

Please define *Sprain*: (Ch. 49)

"Ligamentous injury resulting from an abnormal motion of a joint" 1st degree -minor tearing of ligamentous fibers w/ mild hemorrhage and swelling. 2nd degree -partial tear of ligament with moderate hemorrhage / swelling 3rd degree -complete tearing of ligament

List 4 features of physical exam suggestive of aortic dissection (Ch 85)

"Look. The patient doesn't always know they have Marphan's so you need to look for the signs: (Arachnodactyly / Pectus excavatum / sternal excavation / Lanky limbs ) Listen. A new aortic regurgitation murmur has a surprisingly high +LR = 5. Feel. Feel for a pulse deficit which has a +LR = 2.7." Watch out for pseudo hypotension: if the dissection knocks off a subclavian it will artificially drop the BP in that arm BP Pitfall: Do not assume that the patient with a normal or low BP does not have an aortic dissection. We know from the IRAD data that only about half of patients are hypertensive at initial presentation. Dissection that progress into the pericardium and end up with tamponade are often hypotensive.

Describe the dosing, timeline, indications, and contraindications for emergency contraception (Ch 100)

"Morning after pill" 3 forms: 1. Ulipristal acetate progesterone receptor modulator. 2. Levonorgestrel Progestogen- progesterone receptor agonist 3. OCP Combination of progestin and estrogen "Yuzpe method" Fallen out of favour

Define the anticholinergic toxidrome. what physical exam findings would you see. What substances give you this presentation? What is the treatment? (Ch. 147)

"Red as a beet Dry as a bone Blind as a bat Mad as a hatter Hot as hell The bladder keeps its tone and the heart runs alone" Signs - Increased HR, no change in RR, increased temp - Dilated pupils - Decresed bowel sounds - DRY! - Delirum Substances - Antihistamines - TCA - Cyclobenzaprine - Orphenadrine - Antiparkinsons agents - Antispasmodics - Phenothiazones - Atropine - Scopolamine - Belladonna alkaloids Tx - Supportive, based on specific agent

What is Takotsubo cardiomyopathy and how does it present? (Ch 78):

"Takotsubo cardiomyopathy is referred to as left apical ballooning or "broken heart" syndrome. Takotsubo cardiomyopathy features ST segment elevation (or deep T wave inversions) without evidence of obstructive CAD.

What is Uhtoff's phenomenom? (Ch. 105)

"Uhthoff's phenomenon (also known as Uhthoff's syndrome, Uhthoff's sign, and Uhthoff's symptom) is the worsening of neurologic symptoms in multiple sclerosis (MS) and other neurological, demyelinating conditions when the body gets overheated from hot weather, exercise, fever, or saunas and hot tubs. It is possibly due to the effect of increased temperature on nerve conduction. With an increased body temperature, nerve impulses are either blocked or slowed down in a damaged nerve but once the body temperature is normalized, signs and symptoms may disappear or improve."

What are the Wells criteria for DVT? Describe how to use this score. (Ch 88)

"Wells 10 minus 2" Wise gestalt (including past hx of DVT!) Edema (swelling/veins) Leg casted Leukemia / lesions / lump (active cancer) Surgery in 12 10 - for ten criteria -2 - for one of them is minus two points.

Please define *Strain*: (Ch. 49)

"injury to musculotendinous unit resulting from violent contraction or excessive forcible stretch" 1st degree -minor tearing of muscle and/or tendon fibers w/ mild hemorrhage and swelling. 2nd degree -partial tear of muscle and/or tendon fibers with moderate hemorrhage / swelling 3rd degree -complete tearing of muscle and/or tendon fibers with possible avulsion fracture

What's the most common cause of acute pancreatitis in adults? (Ch 91)

#1 = Gallstones (40%-70%) Followed by: Chronic alcohol consumption (25%- 35%). Endoscopic retrograde cholangiopancreatography (ERCP) Drugs Trauma Idiopathic (up to 10-30% of the time; may be due to microlithiasis?)

What's the most common cause of acute pancreatitis in children? (Ch 91)

#1 = Trauma Followed by: Infection (viral: Mumps and other viruses (EBV, CMV) Congenital (aberrant ductal structure, stone forming diseases - sickle cell disease, hereditary spherocytosis) Genetic lipid disorders

What is the Rule of 9s?(Ch. 63)

% TBSA is allocated as: 18% for front of trunk (9% torso, 9% abdomen) 18% for back of trunk (9% upper back, 9% lower back) 18% for an entire lower extremity 9% for the entire head 9% for each upper extremity 1% for the perineal area

Describe the Ottawa Knee Rule (Ch 57):

(Easy mnemonic: Don't WAIT: 1 of 5 criteria: W Inability to bear weight (4 steps) both immediately and in ED A Age greater than or equal to 55 I Inability to Flex Knee to 90 degrees T Tenderness isolated to patella or head of fibula

Describe a NAC protocal for IV administration(Ch. 148)

**Always check with your local institutions and local tox Typical is 21 hour protocal Adult dosing - Give 150mg/kg dilauted in 200ml of 5% detrose IV over 15-60 minutes - Then: 50mg/kg NAC dilauted in 500ml of 5% dextrose IV over 4 hours - THEN: 100mg/kg NAC diluted in 1000ml of 5% dextrose over 16 hours For later presentations or hepatotoxic - Repeat third bag until transaminases begin to fall Peds dosing: Paediatric Dosing: Children < 20kg body weight: 150 mg/kg in 3 ml/kg of 5% dextrose over 15 minutes. Followed by: 50 mg/kg in 7 ml/kg of 5% dextrose over 4 hours. Followed by: 50 mg/kg in 7 ml/kg of 5% dextrose over 8 hours. Followed by: 50 mg/kg in 7 ml/kg of 5% dextrose over 8 hours. Paediatric Dosing: Children >20kg body weight: 150 mg/kg in 100 ml of 5% dextrose over 15 minutes. Followed by: 50 mg/kg in 250 ml of 5% dextrose over 4 hours. Followed by: 50 mg/kg in 250 ml of 5% dextrose over 8 hours. Followed by: 50 mg/kg in 250 ml of 5% dextrose over 8 hours

List 5 predictors of anxiety caused by an underlying medical issue (Ch 112)

**Many medical conditions mimic anxiety disorders, and up to 42% of patients initially thought to have anxiety disorders are later found to have organic disease**. Predictors of Anxiety Caused by an Underlying Medical Issue: > onset after 35 > no previous personal/family hx > lack of avoidance behavior > no significant life/traumatizing events > poor response to anxiolytics

What are some mainstays of managing abdominal gunshot wounds? (Ch. 46)

**high incidence of peritoneal violation and viscus damage -almost absolute need for operative exploration

What is the dose of sucrose for neonate anaesthesia? (Ch. 4)

*2-10 ml* of 24% solution

How do you measure an arterial-pressure index (API)? (Ch. 48)

*API*: comparison of systolic BP (measured ideally with doppler) in the injured vs. uninjured limb. Think of the injured limb as the "top priority" so its value is the numerator (on "top") divided by the uninjured. Ratio <0.9 suggests badness.

List some acute and delayed complications of fractures: (Ch. 49)

*Acute*: hemorrhage, vascular/nerve injury, compartment syndrome, fat embolism, #blisters, *Delayed*: AVN, osteomyelitis, CRPS, immobilization illnesses

ED management of *severe TBI* (GCS <8 or intracranial contusion/hemorrhage on CT): (Ch. 41)

*Airwary* (etomidate?) *Hypotension* (shock etiology? Keep SBP >90 mmHg) *Hyperventilate* (Goal PaCO2 in acute setting=30-35 mmHg) *Osmotic agents* (controversial) - mannitol/hypertonic saline *Cool the patient?* (weak evidence, maybe helps brain) *Steroids?* NO EVIDENCE *Seizure prophylaxis* - if indicated...benzos/barbituates *Antibiotic prophylaxis* - if open or depressed skull #

What are Bennett's and Rolando's fractures and how are they managed?

*Bennett's*: Intra-articular fracture of the base of the thumb, extending into CMC joint *Rolando's*: Y-shaped 3-part fracture of the base of the thumb Manage both with ED reduction, thumb spica splint and ref. to hand surgeon. VERY UNSTABLE.

CENTOR criteria for dx of sore throat as strep: (Ch. 23)

*C*ervical lymphadenopathy (anterior chain) +1 *E*xudates +1 *N*o cough +1 *T*emperature (hx of fever) +1 *A*ge considerations ( +1 if <15yrs, -1 if >44yrs) **if > or = 4 then ~50% chance of strep infection and consider abx.

Difference between choking, hanging, and strangulation: (Ch. 44)

*Choking*: something inside your mouth/pharynx/trachea blocking air flow *Hanging*: various terms (i/e complete/incomplete ; typical/atypical) based on whether or not the victim's feet were totally suspended and the location of the knot *Strangulation*: may be either manual or ligature due to hands or a device compressing the neck independent from the weight of the patient

Chronic pain vs acute pain: (Ch. 3)

*Chronic Pain=* Prolonged pain that lasts beyond ordinary time needed for an injury/insult to be healed. >3-6 mo persistent pain (not malignancy)

List 10 therapies for an acute severe asthma exacerbation

- 02 --> Sats >90%(95% in pregnant women) - Short acting B2 agonist - Anticholinergics - Corticosteroids - Magnesium sulfate(Calcium channel blocking properties, inhibition of cholinergic neurotransmission, stabilization of mast cells, stimulates NO and prostacyclin) Evidence questionable - Inhaled anesthetic agents - Intravenous ketamine(Bolus 0.5-1mg/kg, followed by infusion of 0.5/3mg/kg/hour) - Parenteral beta agonists - High dose inhaled glucocorticoids - Helium-oxygen - Leukotriene receptor antagonists No evidence - Methyxanthines -Antibiotics - Nebulized furosemide

What is the optimum replantation time of an avulsed tooth?(Ch. 70)

- < 1 hour(66% success) - Viability falls rapidly after three hours(20% success)

List the differential diagnosis for elevated liver enzymes/ abnormal liver function with/without renal failure(Ch. 148)

- Acute tubular necrosis - Rhabdomyolysis - Ischemic hepatitis - Alcoholic hepatic disease - Cyclopeptide containing mushroom toxicity - Viral hepatitis - Wilsons disease - Other hepatic toxicities: valproic acid, statins, isoniazid, herbal medications, vinyl chloride, Biphenyls

List seven ddx for binocular diplopia (Ch. 71)

- Basilar artery thombosis / aneurysm - Botulism - Basilar meningitis - Vertebral dissection - Myasthenia Gravis/ MS/ Graves disease/ Miller-fisher syndrome - Wernicke's encephalopathy - Orbital myositis - Opthalmoplegic migraine - Orbital apex syndrome - Cavernous sinus process - Brainstem tumor

List 6 ddx for facial (CN VII) paralysis. (Ch. 105)

- Bell's palsy - Ramsey hunt syndrome(herpes zoster oticus) - Lyme disease - Bacterial infections of middle ear, mastoid, or external auditory canal - Guillain- barre - HIV infection - Tumor compressing nerve - Stroke(RARE, pons stroke) - Sjogren's syndrome also rare - Sarcoidosis also rare - Melkersson- rosenthal syndrome (rare rare) - Recurrent facial paralysis, episodic facial swelling and a fissured tongue

What is your plan for intubation in a ASA toxicity patient?(CH. 149)

- Bicarb boluses - Maintain vent setting at pre-intubation resp status(High minute ventilation and PCO2 <20mmHg --> Be careful, you can kill a salicylate toxicity patient with incorrect vent settings!

What is ludwig's angina? How do you manage it? (Ch. 70)

- Cariogenic bacteria form plaque, produce acid to decalcify enamel. Necrosis of the tooth, forms periapical abscess. Once this extends into the submaxillary, sublingual and submental space -- Ludwig's angina - Tx: Consider early airway management and treat as difficult airway. Abx like other soft tissue infections of the head and neck(broad spectrum abx). CT. OR

What is your toxicologic differential for Tachycardia + Hypertension(Ch. 147)

- Cocaine - Anticholinergics - Antihistamines - ANtipsychotics - Alcohol withdrawl - Amphetamines - Sympathomimetics - TCA, thyroid hormone, Theophyllines

Risks of STIs after sexual assault (Ch 67)

- Consider local prevalence - Gonorrhea: 6-18% - Chlymadia: 4-17% - Syphilis 0.5-3% - HIV <1% - Pregnancy 2-4%, but increased in women aged 19-26 midcycle

What is the management of an acute MS exacerbation? What is the usual management of a pt with relapsing-remitting MS who presents with a flare?(Ch 105)

- Consult primary neurologist - IV methylpred 250-500mg every 12 hours for 3-7 days Relapsing remitting - Initiate treatment with interferon B or glatiramer acetate - Newer disease modifying agents Facial pain: Carbamazepine Tremor: Propranolol or benzos Spasticity: Baclofen

What is the treatment for ocular chemical injury?(Ch. 64)

- Copius water irrigation - Submerge eyes in running tap water In the ER - Continued tap water irrigation until morgan lens set up - Repeated application of tetracaine/ alcaine local anesthesitc

List 5 mechanisms of action of N-Acetylcysteine(Ch. 148)

- Decrease toxic/ free NAPQI - Direct reduction of NAPQI-APAP - Increase sulfation route - Free radical scavenger - Hepatocyte stabilizing effect

What are some physical signs of neglect? (Ch 69)

- Dehydration - Evidence that the patient has been lying in urine or stool - Clothing inappropriate for climate, dirty or worn out - Poor hygeine - Untreated injuries and medical problems - Poor oral hygeine - Skin breakdown - Elongated toenails

What is the treatment for Central Retinal Artery Occlusion? (Ch. 71)

- Digital global eye massage in the ED (10-15 sec. of pressure, then release -- to dislodge the clot) - timolol 0.5% eye drops - Acetazolamide to lower intraocular eye pressure and increase retinal blood flow treat underlying thrombotic disease Currently debated, used depending on clinical situation: ?inhaled carbogen (95%o2, 5% Co2) to cause retinal artery vasodilation ? role for IV tPa - variable but trendingly positive evidence ? local intra-arterial thrombolytic therapy in less than 6 hrs of onset Intraocular paracentesis of the anterior chamber by optho

What chemicals should you NOT irrigate with water?(Ch. 64)

- Dry lime: Turns into strong alkali when exposed to water= bad - Elemental metal/ reactive metal compounds - Phenol: Use PEG instead!

List 4 removal techniques of esophageal FB (Ch 60)

- Endoscopy -Contrast filled balloon catheter and fluoroscopy -Bougienage (pushing object into stomach) -Expectant Management

List 5 reasons why urine toxicilogy are not typically useful(Ch. 147)

- False negatives - False positives - Are qualitative(Dont predict acute intoxication) - Costly and do not change patient care - Have a time lag (Exposure may be days or weeks ago)

What factors predict abnormal CT findings in seizure patients?(Ch. 102)

- Focal abnormality on exam - Malignancy - Closed head injury - Neurocutaneous disorder - Focal onset of seizure - Absence of a history of alcohol abuse - History of cystircosis - Altered mental status - Older then 65 - Seizure duration of > 15 minutes

What is an alveolar ridge fracture? (Ch. 70)

- Fracture through the ridge of bone that forms the sockets for the teeth(Dental alveoli)

Describe the clinical presentation of ASA toxicity. Describe the acid base disturbance and progression of toxicity in acute salicylate overdose(Ch. 149)

- Hyperventilation, N/V/Dyspepsia, tinnitus Then: Severe hyperpnea, lethargy, hyperthermia End organ dysfunction: Coma, pulmonary/cerebral edema, seizures. Renal failure. Cardiovascular collapse Acid base disturbance - Respiratory alkalosis --> Bicarb losses - Met acidosis-->AG!

When can you stop NAC? (Ch. 148)

- If APAP is negative - If treatment is started in 8 hours, and it is immediate release then NAC can be stopped at the 20 hour mark when the infusion is done. UNLESS the overdose was >500mg/kg - For other ODs NAC is continued until AST/ALT is stabilized or decreaseing, the patient is asymptomatic and the paracetamol concentration is decreasing/ cleared - Anaphlactoid reactions

What are some indications from a medical history that there may be elder abuse? (Ch 69)

- Implausible history of injury mechanism - Inconsistent history between patient and caregiver of injury - Delay between injury and presentation to care - Unexplained injuries - Referred to as "accident prone" - Past history of frequent injuries - Noncompliance with medications, appointments or physician directions - Caregiver not able to give details about medical care - Caregiver answers the question regarding patient - Patient or caregiver reluctant to answer questions - Strained patient-caregiver interaction - Poor living conditions according to paramedics or others

How do you tell physiologic aniscoria from other causes?(Ch. 71)

- Increases in darkness - NO dilation lag - NO loss of vision - NO diplopia

What is your ventilation strategy for a critically ill asthma patient

- Induction: Ketamine/propofol/etomidate. Par: Succs or roc - Vent: VC!, volume over pressure - Tidal volume: 6-8cc/ideal body weight(Lung protective) - Minute ventilation: Less than 115 ml/kg/min - Allow increased expiratory time by decreaing I:E ratio(1:3 or 1:4 up to 1:5) - Low PEEP - Start FIO2 at 100%, titrate downwards >90%

What is the treatment for an acid burn of the eye? What kind of necrosis does it cause?(Ch. 71)

- Irrigation for atleast 30 minutes --> until pH normal - May need topical anesthetic - Remove particle - After irrigation: Cycloplegic, 0.5% erythromycin ointment QID, pain management, check IOP Necrosis: Coagulation

What is the treatment for an alkali burn of the eye? What are some examples of substances that induce these burns? What kind of necrosis does it cause?(Ch. 71)

- Irrigation for atleast 30 minutes --> until pH normal - May need topical anesthetic - Remove particle - After irrigation: Cycloplegic, 0.5% erythromycin ointment QID, pain management, check IOP Substances: Drain cleaners, chemical detergents, solvents, lime Necrosis: Liquefactive

List 5 dialyzable drug properties(Ch. 147)

- Low molecular weight - Low protien binding - Low volume of distribution - Low plasma clearence - Low dialysate drug concentrations - high water solubility

List 4 possible investigations for spinal epidural abscess and diskitis. What are the expected findings?(Ch. 106)

- MRI with contrast: ENhancement - CBC: WBC may be elevated but NONSPECIFIC - ESR/CRP: 100% sensitive if elevated - Xray: Disc space narrowing, loss of vertebral body endplates

How does NSAID toxicity manifest? (CH. 149)

- Most only cause minor symptoms - Less common: Metabolic acidosis, fasiculations, mydriasis, diaphoresis, hyperventilation, bradycardia, hypotension, dyspnea, tinnitus, and rash. Rare cases: coma, seizure, hypotension and met acidosis have been reported in massive ODs Renal dysfunction is usually only seen after large overdose and usually responds to supportive measures

What differences should be considered with male victims of sexual assault? (Ch 67)

- Much less common(About 8%) - Increased risk amongst: Gay and bisexual men, veterans, inmates, MH treatment centers - Increase non genital trauma

What are the characteristics of self inflicted knife wounds? (Ch 65)

- Multiple superficial incisions located on the anterior trunk, arms and face - Multiple superficial stab wounds located on the anterior trunk, arms and face - Parallel incisions, in close proximity to each other, on non dominant side of body - Sparing of sensitive body areas - Linear or curved incisions toward the hand inflicting the wound - Intact clothing covering the wound - Evidence of prior wounds

What are the law enforcement exemptions to the HIPPA act? (Ch 65)

- Name and address - Date and palce of birth - Social security number - ABO blood type and R factor - Type of injury - Date and time of treatment - Date and time of death - A description of distinguishing physical characteristics

List differential diagnosis for facial pain/ Trigeminal neuralgia(Ch. 105)

- Odontogenic infections - Sinus disease - otitis media - Acute glaucoma - TMJ disease - Herpes zoster Important to remember that the craniofacial pain can come from any of these nerves: CN V, VII, IX, X, and upper cervical spinal roots

When should you consider thombolysis in pts presenting with stroke?(Ch. 101)

- Onset of symptoms <4.5 hours before beginning treatment(or when patient last known to be normal) - Age >18 year

What is your toxicologic differential for Hypoglycemia(Ch. 147)

- Oral hypoglycemic agents - Beta blockers - Isoniazid - Salicylates - SUlfonylurea - Insulin - Ethanol

List 3 ECG Findings of COPD

- P-pulmonale (right atrial enlargement, peaked P-waves in II,III, aVF) - low QRS voltage due to large hyperinflated chest - clockwise rotation & poor R wave progression in the precordial lead (classically described from RV hypertrophy or dilatation, but this is non specific) -Tachydysrythmias: AFIB/Multifocal atrial tachy

What are common red flags to alert you to the possibility of domestic violence? (Ch 68)

- Patient is reluctant to describe or hesitant when questioned about how the injury occured - History of medical, drug or alcohol abuse, especially for sleep - Injuries during pregnancy - Multiple vague complaints - Wounds at various stages of healing - Household members that are angry, rushed, indifferent, controlling or minimizing complaints - Homelessness

List 10 toxins that cause delirium(Ch. 147)

- Prescription medications: Opioids, sedative- hypnotics, antipsychotics, lithium, skeletal muscle relaxers, polypharmacy - Non presciption: Anti histamines - Drugs of abuse: Ethanol, heroin, hallucinogens - Withdrawl: Ethanol/ benzos - Ethylene glycol, methanol - Carbon monoxide, cyanise, hydrogen sulfide - Jimson weed, salvia

4 indications for invasive BP monitoring: (Ch. 5)

- anticipated hemodynamic instability - monitoring condition or Tx in real time - need for frequent arterial sampling - inaccurate BP due to obesity or dysrythmias

Which patients with AECOPD should be treated with antibiotics?

- need for intubation or NIPPV -Increased sputum purulence with: --Increased dyspnea, OR --Increased sputum volume - clinical pneumonia +/- radiographic findings - Cater to local bug and drug patterns - 5 days course of resp fluoroquinolones > 7-10 days with beta lactams and tetracycline

What are contraindications to using the Rumack-Matthew normogram(Ch. 148)

- unknown time of ingestion - <4 hours since ingestion - Time of ingestion >24 hours - Chronic ingestion - Co-ingestions with other drugs that effect gut motility(anticholinergics/ opioids) Note: Extended release tablets can still use normogram, just make sure to do another level at 8 hours

How long does a warm (not actively cooled) limb with occlusive vascular injury have until irreversible ischemia sets in? (Ch. 48)

-6hrs until at least 10% have complete muscle and nerve damage -12hrs until 90% have irreversible damage

Types of ACUTE complications of transfusions: (Ch. 7)

-ABO incompatibility (intravascular hemolysis) -TRALI -TACO -Bacterial contamination -Allergic reaction -Febrile transfusion reaction

4 side-effects of Propofol: (Ch. 4)

-BP decrease -resp depression -injection pain -loss of airway reflexes

Management of systemic toxicity of anaesthetic: (Ch. 4)

-Benzos for seizures -ACLS guidelines (high risk of MI) -Maybe lipid emulsion??

Indications for antibiotics in patient with facial trauma: (Ch. 42)

-Bite wounds -Evidence of devascularization -Damage through the buccal mucosa -Involvement of cartilage of ear or nose -Highly contaminated wounds

Tell-tale clinical features of basial skull fracture? (Ch. 41)

-Blood in ear canal (Hemotympanum ) -Rhinorrhea -Otorrhea -Battle's sign (retro auricular hematoma) -Racoon's sign (periorbital ecchymosis) -Cranial nerve deficits: ■ Facial paralysis ■Decreased auditory acuity ■Dizziness ■Tinnitus ■Nystagmus

List indications for removal of esophageal FBs (Ch 60):

-Button battery -Magnet -Sharp Object -Displaced esophageal stent -Duration is place > 24 hours

Describe the management of venous air embolism: (Ch. 44)

-Can occur from blunt/penetrating/iatrogenic mechanisms -Problem vessels: carotid > subclav > vert a. > int/ext jug vein -Direct occlusive pressure over site -Trendelenburg -Left lat decubitus (so air rises to R atrium) -aspiration of air bubble from R atrium via U/S guided or direct (thoracotomy)

List the complications of an ear foreign body (Ch 60)

-Damage to Ear canal -Otitis Externa -TM Perforation -Expansion of vegetable / porous material with irrigation Critical Complications: -Eustachian tube dysfunction -para-pharyngeal abscess -mastoiditis with progression to fatal brain abscess and meningitis.

List 4 side-effects of NSAIDS: (Ch. 3)

-GI bleeding -renal failure -anaphylaxis/TENS/SJS -platelet dysfunction

Types of DELAYED complications of transfusions: (Ch. 7)

-Graft vs host disease -Extravascular hemolysis -Viral transmission

Main Sxs of Post-Concussive Syndrome? (Ch. 41)

-Headache -Nausea -Sensory hypersensitivity (photophobia, phonophobia) -Memory/concentration problems -Sleep changes -Mood changes, irritability

List 4 CXR findings of COPD

-Hyperinflated lungs -Decreased vascular markings -Small cardiac silhouette OR late stage cardiomegaly -Increased vascular markings. -Bullae may be many or may be large, mimicking pneumothorax.

Describe Felon and its Tx: (Ch. 50)

-Infection of the pulp of any digit, made worse due to the fibrous septa of the pulp. -S. aureus or gram negs Tx: I+D with a deep lateral incision posterior to digital artery and nerve ■Ulnar side of index, middle, ring fingers ■Radial side of thumb and little finger

Describe Paronychia and its Tx: (Ch. 50)

-Infection or abscess of lateral nail fold -Staph/strep Tx: If fluctuant or discharge, lift lateral nail fold with scalpel and drain + Abx. Risk of osteomyelitis.

What are three complications of maxillary tooth infection? (Ch. 70)

-Intracranial extension of abscess -Periorbital cellulitis -Cavernous Sinus Thrombosis

When should a specialist repair an eyelid lac? (Ch. 42)

-Involvement of deep structures (e.g. tarsal plate) -Avulsion or loss of tissue -Involvement of lid margin -Violation of any lacrimal apparatus (check by instilling fluoroscein in eye and assessing for uptake in the wound)

Describe the management of compartment syndrome? (Ch. 49)

-Measure tissue compartment pressure (serial) -When tissue pressure is within 30mmHg of MAP, -consider fasciotomy (STAT or within 12hrs) -Don't elevate limb -Manage rhabdo, hyperKa, lactic acidosis

Clinical findings of lower motor neuron (LMN) lesion: (Ch. 13)

-flaccidity -*hypo*-reflexia -fasciculation -decreased muscle bulk

8 side-effects/complications of Ketamine: (Ch. 4)

-hypersalivation -emesis -laryngospasm -emergence reactions -transient apnea (if given quickly) -increased IOP -worsening psychosis -catecholamine surge

"Ventilator trouble shooting": Potential causes of acute resp distress *WITH* hemodynamic compromise... (Ch. 2)

-increased iPEEP -tension pneumothorax (surprisingly common) -massive PE

What are some mainstays of managing blunt abdominal trauma? (Ch. 46)

-is patient hemodynamically unstable? (occult bleeding!) -if unstable: laparotomy (+/-FAST u/s) -if stable: FAST/CT, OR vs watchful waiting

Contraindications to succinylcholine? (Ch. 1)

-known allergy -hx/fmhx malignant hyperthermia -increased hyperkalemia risk -plasma cholinesterase deficiency

Less-critical etiologies of cyanosis: (Ch. 14)

-methemoglobinemia -sulfhemoglobinemia -polycythemia -Raynaud's phenomenon

Clinical findings of neuromuscular junction lesion: (Ch. 13)

-mix of UMN and LMN signs -slowly progressive manifestations -bilat proximal muscle weakness

Contraindications to do thoracotomy in ED: (Ch. 36)

-no signs of life on scene and in the ED -CPR (despite signs of life initially) > 10 mins -system or department reasons

What is the importance of peri-oral electrical burns? (Ch. 42)

-often full thickness (saliva = poor impedence to current) -eschar separation after ~2wks --> labial artery hemorrhage -poor cosmetic outcomes

Besides COPD, list 4 other differentials that could present with progressive shortness of breath +/- wheeze

-pneumonia -congestive heart failure (CHF) (may wheeze / cardiac asthma) -pneumothorax -pulmonary embolism (PE) -lobar atelectasis (plugging / mass) -pleural effusion -dysrhythmias

Name some potential complications of rib #s: (Ch. 45)

-pneumothorax -hemothorax -pulmonary contusion -post-traumatic pneumonia or neuroma -delayed hemorrhage (rebleeding intercostal artery) -liver/spleen injury

List 3 ideal IO sites in kids: (Ch. 38)

-proximal medial tibia -proximal humerus -anterior distal femur

ED management of mild TBI (GCS 14-15, a clinical diagnosis): (Ch. 41)

-reassurance -F/U if worsening sx

What are some mainstays of managing abdominal stab wounds? (Ch. 46)

-selective non-operative management (lap or not) -is peritoneum violated or not? -is diaphragm involved? -"implement in situ"

Clinical findings of upper motor neuron (UMN) lesion: (Ch. 13)

-spasticity to extension in extremities -spasticity to flexion in lower extremities -*hyper*-reflexia -pronator drift -Hoffmann's sign -Upgoing Babinski -normal muscle bulk

What are some potential fluid therapies for hemorrhagic shock in kids and their doses? (Ch. 38)

-standard trauma/resus room protocol : MOVIE -10-20ml/kg of crystalloid IV bolus repeated up to 3x then: ○ PRBCs: 10 ml/kg ○ FFP: 25 ml/kg ○ Platelets: 10 ml/kg

Dose and indication for core peds resuscitation med: *Furosemide* (Ch. 10)

1 mg/kg ; for pulmonary edema

Dose and indication for core peds resuscitation med: *Lidocaine* (Ch. 10)

1 mg/kg ; pretreatment agent for intubation in elevated ICP

List 5 techniques for initiating therapeutic hypothermia

1) Cold saline infusion 2) Misting and fans 3) Ice packs in the groin and axilla 4) Cooling blankets 5) Internal cooling (bladder irrigation, chest tubes, ECMO)

How is are chronic cirrhosis and ascites managed in the ER? (Ch 90)

1) Correct fluid/electrolyte abnormalities 2) Complications that need ER treatment: a) Encephalopathy b) GI bleeding Plts > 50k Fibrinogen > 100 mg/dL (use Cryoprecipitate) Vitamin K for bleeding (and elevated INR) Ceftriaxone 1 g IV daily until patient is taking food orally - this is for SBP prophylaxis! c) Spontaneous bacterial peritonitis Paracentesis* Consider antibiotic prophylaxis if high risk for SBP recurrence Stoppage of PPI's and BB's d) Hepatorenal syndrome Renal failure in the setting of cirrhosis, with no other renal pathology May need albumin infusions, midodrine, octreotide, IV norepinephrine e) Consider replacing vitamin / nutrient deficits Thiamine Mg K f) Ascites

List 4 mechanisms by which therapeutic hypothermia may improve neurologic outcome: (Ch. 8)

1) Decreases metabolic demands 2) Decreases free radical formation 3) Decreases production of inflammatory cytokines 4) Prevents programmed neuron death

What are the three pathophysiologic mechanisms for dysrhythmias? (Ch 79)

1) Enhanced automaticity: ectopic focus in non-pacemaker cells OR lower threshold in pacemaker cells: MI, digitalis toxicity, drugs, electrolyte disturbances 2) Triggered activity >early or delayed afterdepolarizations Torsades de pointes (TdP); intracell Ca+ overload, post MI reperfusion 3) Re-entry >due to abnormal conduction: creating circus movement two pathways, one with a longer refractory period (so conduction goes down the faster pathway. Usually the cause of VT and SVT

List 6 clinical signs that would point to genitourinary trauma: (Ch. 47)

1) Flank/pelvic/genital pain (esp with pelvis #) 2) Urinary retention 3) Penile or scrotal ecchymosis 4) Gross hematuria 5) Blood at urethral meatus 6) High-riding prostate (ever seen it? nope)

List 5 PEX findings in appendicitis. (Ch 93)

1) Iliopsoas (psoas) sign 2) Rovsing's sign 3) Obturator sign 4) McBurney's point tenderness 5) A genitourinary examination Here are the Top three: "RLQ tenderness; Abdominal wall rigidity; Pain focused at McBurney's point"

Describe the management of elevated ICP in kids: (Ch. 38)

1) Non-pharmacologic -HOB @30deg -head midline -hyperventilate for temporary fix -avoid hyperthermia -keep euvolemic -craniotomy/burr holes if necessary 2) Pharmacologic -Mannitol (0.5 g/kg IV) -rapid osmotic diuresis -Hypertonic saline (0.1-1 mL/kg of 3% saline) -Pentobarbital/phenobarbital-lowers cerebral metabolism -Neuromuscular blockade -Sedation -Treat seizures aggressively

What features of skull fractures in kids are associated with BAD outcomes? (Ch. 38)

1) fracture over a vascular channel 2) a depressed fracture 3) a diastatic fracture-- one that crosses through/along suture lines. Leptomeningeal cysts (growing fractures) may develop (>2mm of separation) 4) fracture extending over the area of the medial meningeal artery (risk of epidural bleed) 5) rhinorrhea, otorrhea

Four main parts of male urethra: (Ch. 47)

1) pendulous urethra 2) bulbous urethra 3) membranous urethra 4) prostatic urethra

What are the most common causes of status epilepticus? (Ch. 18)

1) seizure med non-compliance (25%) 2) EtOH-related (15-20%) 3) drug toxicity/post-stroke/CVA/lytes/hypoxia (35%) 4) other: infection, tumour, trauma (15-20%)

List four key diagnostic criteria for delirium(Ch. 104)

1. Acute onset with fluctuating course 2. Disturbs attention and awareness 3. Disturbed perception 4. Not better explained by another neurocognitive disorder

Dose and indication for core peds resuscitation med: *Sodium bicarbonate* (Ch. 10)

1-2 mEq/kg ; for profound metabolic acidosis or Na channel blocker overdose (e.g. TCA)

Clinical presentation of placental abruption (it is a clinical diagnosis): (Ch. 37)

1. **Fetal distress** 2. Painful, vaginal bleeding 3. abdominal cramping 4. uterine tenderness, contractions 5. Maternal shock

List the 5 main causes of peripheral vertigo: (Ch. 19)

1. *BPPV* --no tinnitus or hearing loss, acute onset 2. *Vestibular neuronitis* --no auditory Sx, recent infection or toxicity 3. *Labyrinthitis* --hearing loss + active infection 4. *Acousitc neuroma* --gradual hearing loss, unilateral, ataxia in later stages 5. *Meniere's Disease* --vertigo for HOURS + tinnitus + hearing loss

Major etiologies of non-traumatic cardiac arrest: (Ch. 9)

1. *Cardiac* (VF, VT, MI, HCM, valvular disease, PEA) 2. *Respiratory* (asthma, foreign body) 3. *Circulat./Obstr.* (tension pneumo, PE, tamponade) 4. *Metabolic* (hyperkalemia!) 5. *Toxic* (Digoxin, CCB, TCA, cocaine, heroin, CO) 6. *Environmental* (electrocution, hypothermia, drowning)

What are the 3 most common C-spine injuries in the elderly? (Ch. 39)

1. *Central cord syndrome* (hyperextenion) weakness in arms > legs often no fracture but have preexisting stinal stenosis painful hyperesthesias usually stable, but need early mobilization 2. *Cervical distraction and extension injuries* from forehead/face trauma 3. *Odontoid fractues*

List 4 general mechanisms that causes compartment syndrome: (Ch. 49)

1. *Increased compartment contents*: Bleeding, reperfusion after ischemia, trauma, exercise, burns, DVT 2. *Decreased compartment volume*: closure of fascia, excessive traction of # 3. *External pressure*: Casts, air splints, dressings, Lying on a limb 4. *Miscellaneous*: Muscle hypertrophy, popliteal cysts, leaky cannulae, interstitial infusions/pressure infusions

List 4 main steps of pain perception... (and their associated Tx strategies) (Ch. 3)

1. *Pain detection* -local anaesthetic, nerve blocks, topical NSAIDS 2. *Pain transmission* -opiates, tylenol, gabapentin, TCA 3. *Pain expression* -narcotics, benzos, antipsychotics 4. *Pain modulation* -TENS, massage, acupuncture, hypnosis

In "confusion", what are the 3 major categories that can impact and disrupt normal cortical function? (Ch. 17)

1. *Substrate deficiency* -hypoxia, hypoglycemia 2. *Neurotransmitter dysfunction* -endocrine d/o, hepatic failure, CNS sedatives, infections, EtOH, poisons 3. *Circulatory dysfunction* -shock

List the 5 main causes of central vertigo: (Ch. 19)

1. *Vascular disorder* --vertebrobasilar stroke (vasculopath pt with vertigo) --cerebellar hemorrhage (ataxia, dysmetria, N/V) --Post. inf. cerebellar art. occl. (vertigo + neuro deficits) 2. *Head trauma* 3. *Vertebrobasilar migraine* --vertigo leads to headache, no residual neuro deficits 4. *Multiple sclerosis* --7-10% have vertigo as presenting sx, look for bilat INO 5. *Temporal lobe epilepsy* --may have memory impairment + vertigo episodes

Describe 4 types of hip dislocation (Ch 56):

1. 80-90% = Posteriorly - internally, flexed, adducted hip (knee rests on the opposite thigh) MVC (dashboard forces on the flexed abducted hip) 2. 10--15% = Anteriorly - externally rotated, abducted, slightly flexed hip (leg appears longer) Forceful extension, abduction, and external rotation of the femoral head MVC's or hyperextension during a fall May also be: Obturator dislocation - towards the obturator foramen Pubic dislocation - towards the pubis 3. 2-4% = central dislocation Femoral head is pushed through a decimated acetabulum 4. Luxatio erecta femoris

4 Contraindications to neonate resuscitation: (Ch. 11)

1. <23 weeks gestational age 2. <400 g birth weight 3. confirmed anencephaly 4. trisomy 13 or 18

List 3 differential diagnoses for kinin-mediated angioedema (Ch 119)

1. ACE inhibitor-induced 2. HAE with or without C1 esterase inhibitor deficiency 3. Acquired C1 esterase inhibitor deficiency (AC1D) 4. Idiopathic angioedema

Six differential diagnoses for physical abuse (Ch 66)

1. ALTE(BRUE now) 2. Seizure 3. Intestinal injuries 4. Pancreatitis 5. Liver/ spleen injury 6. Underlying coagulopathy(Familial, acquired or leukemia) 7. Osteogenesis imperfecta

List 3 other studies that can be performed (Ch 89)

1. Abdominal radiography 2. Gastrograffin swallow (water soluble agent) 3. CT chest 4. Flexible EGD - 100% sensitivity (although has a risk of enlarging a minimal transmural opening due to insufflation of air)

Six differential diagnoses for sexual abuse (Ch 66)

1. Accidental trauma(Straddle injury) 2. Dermal melanocytosis(formerly mongolian spot) 3. Lichen sclerosus 4. Impetigo 5. Urethral prolapse 6. Anal fissures 7. Infectious causes(STI, shigella, Group a beta hemolytic strep, candida, pinworm, chigger) 8. Vaginal foreign bodies 9. Priaprism

List 4 criteria for hospitalization of an acute psychiatric episode (Ch 111)

1. Actively suicidal 2. Dangerous to others 3. Possess severe mental debilitation precluding self-care 4. Patient with a first psychotic episode

List 4 indications for hospitalization of psychotic patient (Ch 110)

1. Actively suicidal 2. Dangerous to others 3. Possess severe mental debilitation precluding self-care 4. Patient with a first psychotic episode

List the clinical criteria for the diagnosis of anaphylaxis (Ch 119)

1. Acute onset - minutes/hours of skin and/or mucosal tissue involvement pressure drop. 2. Exposure to a likely allergen, with acute symptomatology, with two of the four signs and symptoms: Skin-mucosa Respiratory system Blood pressure drop GI symptoms 3. Acutely reduced BP after exposure to a known allergen for that patient

List at least 5 early complications of AMI and briefly describe the management of each one (Ch 78).

1. Bradydysrhythmias / AV conduction block (Treatment: Atropine +/- pacing, ACLS care) 2. Tachydysrhythmias (Sinus tach, a.flutter, VF, VT, etc) (Treatment: ACLS care!) 3. Cardiogenic shock (Treatment: Investigate for any other contributing causes (pericardial effusion, hypovolemia), i.e. vasopressor and inotropic support, intra-aortic balloon counterpulsation, and early revascularization) 4. LV freewall rupture (Treatment: Resuscitate. Surgery! (If survived) 5. Septal rupture (Treatment: Resuscitate, Surgery 6. Papillary muscle rupture (leading to mitral valve regurg.) (Treatment: Resuscitate, Surgery) 7. Infarct Pericarditis (Treatment: Anti-inflammatories) 8. Embolic or hemorrhagic stroke (Treatment: stroke care) 9. Iatrogenic complications: From anticoagulation/antiplatelets/fibrinolysis

List 6 treatment options for constipation: (Ch. 32)

1. Bulking agents: indigestable fibre (figs, psyllium) 2. Osmotic agents: milk of magnesia 3. Sugars: lactulose, PEG 3350 4. Stool softeners: mineral oil, colace 5. Stimulant laxatives: senokot 6. Suppositories and enemas: glycerin for impacted stool

What kind of cervical spinal injuries can you get from VERTICAL COMPRESSION forces? And are they stable/unstable? (Ch. 43)

1. Burst # STABLE -may impinge on ant. cord 2. Jefferson # UNSTABLE**

What are the indications for removal of an esophageal foreign body? (Ch 89)

1. Button batteries (can cause severe tissue damage in 2 hrs!) 2. Large objects (e.g. dentures, tooth brush) 3. Sharp objects 4. Coins lodged in the proximal esophagus 5. Complete esophageal obstructions (preventing liquid past) Note: Incomplete obstructions must be removed in 24 hrs

For what C-spine injuries is CT-A indicated to rule out vascular injury? (Ch. 43)

1. C1-C3 # 2. Any vertebral body fracture 3. Transverse foramen fracture 4. Facet sub/dislocation 5. Ligamentous injury

Three indications to NIPPV: (Ch. 2)

1. COPD 2. Cardiogenic pulm edema w. fatigue 3. Hypercapnic acidosis (in asthma, pneumonia)

What are the best tests of choice for imaging chronic pancreatitis? (Ch 91)

1. CT Dilated pancreatic ducts Atrophy Microcalcifications Pseudocysts 2. MRCP 3. Endoscopic ultrasound = most sensitive tool

List 6 RFs for bacterial endocarditis (Ch 83):

1. Calcific/degenerative aortic/mitral valves 2. Rheumatic heart disease 3. Congenital cardiac lesions either repaired or unrepaired (VSD, pulm stenosis, TEt. of fallot, palliative conduits, ) 4. Cardiac valve disease in a TRANSPLANTED heart 5. Mitral valve prolapse 6. IVDU (right sided endocarditis) 7. Prosthetic valve 8. History of endocarditis Others: indwelling IV catheters, immunosuppression, recent dental/surg. Procedure.

Name the areas of narrowing in the ureter (Ch 99)

1. Calyx of the kidney 2. Ureteropelvic junction (UPJ) 3. Uretero-pelvic brim (ureter arches over iliac vessels posteriorly into the true pelvis) 4. Ureterovesicular junction (UVJ) this is the most common site of obstruction. This is where the ureter enters the muscular coat of the bladder 5. Vescial orefice

List any four complications of IE (Ch 83):

1. Cardiac Heart failure Perivalvular abscess 2. Embolic CVA Blindness - due to CRAO Pain syndromes from intestinal / cardiac infarctions Metastatic infections (showered lung) 3. Neuro: StroKe Acute encephalopathy Meningo-ecphaliTis Seizures 4. Renal Infarction GNephritis 5. MSK: Vertebral osteomyelitis

List 10 treatment options for chronic HF (Ch 81):

1. Cardiovascular exercise and strength training / conditioning program 2. Obesity reduction 3. Healthy diet 4. Beta-blockers 5. ACE Inhibitors 6. ARBs 7. Diuretics 8. Spironolactone 9. Digoxin 10. Cardiac resynchro therapy (biventricular pacemaker)

When should antibiotic prophylaxis for a wound be considered (Ch 59):

1. Cat bites - all. (Staphylococcus, streptococcus and Pasturella multocida). Amox clav (875mg x 7d) 2. Dog bites - controversial. Guidelines say limit to hand, very dirty, older patients, deep puncture and immunocompromised. Amox clav x7d 3. Fight bites - human bites or assumed to the hand. First thoroughly look for tendon or joint damage. Streptococcus, staphylococcus, eikenella corrodens and bacteroides. Amox Clav , plastic surgery consultant opinion 4. Puncture wound of foot - no data supporting but should be considered especially in puncture through subber show (pseudomonas). Ciproflox for pseudomonas, keflex for staph/strep. ?MRSA Septra or Doxy 5. Delayed primary closure in high risk patient 6. Open fractures 7. High velocity missile wounds

Besides Light's Criteria, list 5 other tests to perform on pleural fluid. What does a low pH indicate? (Ch 77)

1. Cell count (High counts = empyema) 2. Glucose (Pleurisy, malignancy or esophageal rupture) 3. Amylase (pancreatitis, esophageal rupture or malignancy) 4. Gram stain and culture 5. Cytology A pH <7.0 is strongly suggestive of empyema or esophageal rupture: these patients need a tube thoracostomy

6 important neuromuscular sites of weakness (with examples): (Ch. 13)

1. Cerebral cortex (CVA, tumour, abscess, demyelination) 2. Brainstem (CVA, demyelination) 3. Spinal cord (ischemia, compress by abscess, disk) 4. Peripheral nerve (GBS, compressive plexopathy) 5. NMJ (myasthenia, tick paralysis, cholinergic crisis) 6. Muscle (rhabdomyolysis, myositis)

Describe typical pain of pericarditis, expected labwork abnormalities (Ch 82):

1. Chest pain (Sharp, Pleuritic, Varies with position. Relieved by sitting forward and worse lying down/deep breath in/swallowing. May radiate to shoulders/diaphragm) 2. Hx of fevers and myalgia 3. The friction rub - is typically only heard in sound-proofed cardiologists offices...

List six ways to determine if peritoneum has been violated in penetrating trauma: (Ch. 46)

1. Clear missile path through abdomen - (in/out wounds) 2. Bowels/organs visualized on Local Wound Exploration 3. "End" of the wound tract not visualized on LWE 4. X-ray showing retained foreign body or free air 5. Laparoscopy showing diaphragm injury 6. CT showing peritoneal violation

List 4 techniques for the diagnosis of herpes (Ch 98)

1. Clinical: History of similar lesions in same distribution supports clinical diagnosis though insensitive and nonspecific. 2. PCR diagnostic test of choice - highest Sn/Sp in presence of active lesions. 3. Direct fluorescent antibody (DFA) 4. Serology for HSV

List four major complications of CKD (Ch 97)

1. Hyperkalemia 2. Bleeding 3. Pulmonary edema 4. Infection 5. Hypocalcemia 6. Hyperphosphatemia 7. Hypermagnesemia

Name an additional 5 risk factors for adolescent suicide (Ch 115)

1. Demographic (sexual orientation (lesbian, gay, bisexual, unsure). 2 & 3. Biopsychosocial ---- weight concerns (perceive self as overweight; prior fasting, diet pills, or vomiting/laxatives for weight control) -----sexual health (prior sexual intercourse; sex before age 13; four or more partners; sexually active; no use of condoms) 4. Environmental. Exposure to violence (i.e. carried a weapon, in a. physical fight). 5. Sociocultural. Participation in goth subculture.

List 6 indications for starting seizure prophylaxis in traumatic head injury patients: (Ch. 41)

1. Depressed skull # 2. paralyzed/intubated pt. 3. Any seizure at any time (or hx of same) 4. PENETRATING brain injury 5. Any brain bleed (epidural, SDH, ICH) 6. Severe head injury: GCS <8

Describe treatment options for IBS (Ch 95)

1. Dietary changes (no one actually knows what to consistently suggest!) 2. MedS: Dicyclomine (IBS-P) Loperamide (IBS-D) Rifaximin Lactulose (IBS-C) TCA's SSRIs 3. Peppermint oil 4. Probiotics

Describe the radiologic features of toxic megacolon (Ch 95)

1. Dilatation of the right colon greater than 6 centimeters, 2. Dilatation of the transverse colon; 3. Absence of normal colonic haustral markings, 4. Air-filled crevices between large pseudopolypoid projections extending into the colonic lumen.

Name 6 organisms responsible for bloody diarrhea: (Ch. 31)

1. EHEC 2. Campylobacter 3. Shigella 4. Yersinia 5. Salmonella 6. Vibrio

List 6 LIFE THREATENING causes of acute pelvic pain in pregnant women (not to miss!): (Ch. 33)

1. Ectopic 2. PID 3. Tubo-Ovarian abscess 4. Appendicitis 5. Hemorrhagic ovarian cyst (ruptured) 6. Bowel/uterine perforation

List 3 causes of pelvic pain in women <20 wks pregnant that are of reproductive tract origin: (Ch. 33)

1. Ectopic 2. Threatened abortion 3. Ovarian hyper-stimulation syndrome (think of IVF)

List 6 causes of bleeding in early pregnancy? (Ch. 34)

1. Ectopic! 2. Miscarriage (TISCIMS) 3. Implantation bleeding 4. GI/GU bleeding 5. Trauma 6. Gestational trophoblastic disease

List 4 patients that get acalculous cholecystitis (Ch 90)

1. Elderly 2. Admitted patients recovering from non-biliary tract surgery 3. AIDS patients with secondary CMV or cryptosporidium infection 4. Men, with uncontrolled diabetes (high risk for emphysematous cholecystitis as well)

What is the unusual laboratory feature seen with APS? (Ch 118)

1. Elevated PTT in the setting of a normal PT/INR Due to interference of the coagulation study by aPL antibodies. 2. False positive VDRL "The Venereal Disease Research Laboratory (VDRL) assay to test for syphilis contains cardiolipin and thus will commonly be falsely positive in patients with anticardiolipin antibodies or APS."

Name 8 emergent diagnoses of 'sore throat': (Ch. 23)

1. Epiglottitis 2. Bacterial tracheitis 3. Ludwig's angina 4. Gonorrhea 5. Foreign body 6. Angioedema 7. Retro-pharyngeal abscess 8. Peritonsillar abscess

List 3 complications of GERD (Ch 89)

1. Esophagitis Progressing to Barrett's metaplasia, erosion, ulcerations, scarring Stricture formation due to persistent scarring and inflammation 2. Reflux induced asthma Microaspiration of 3. Adenocarcinoma of the esophagus 4. Esophageal perforation → mediastinitis

5 Questions for mom during imminent delivery: (Ch. 11)

1. Estimated gestational age? 2. Is it a multiple gestation? 3. Is meconium present? 4. Any history of vaginal bleeding? 5. Any meds taken during delivery or intrapartum?

3 HARD indicators of adequate prefusion during CPR: (Ch. 9)

1. EtCO2 >10 mmHg 2. Diastolic pressure >40 3. ScVO2 >40%

6 ways to monitor CPR: (Ch. 9)

1. EtCO2 >10 mmHg (ideal = 20 mmHg) 2. Palpble carotid or femoral pulses 3. Diastolic pressure >40 4. ScVO2 >40% (only monitored in ICU) 5. Coronary perfusion pressure >15 mmHg

What is the differential for Pneumonia visible on CXR? (Ch 76)

1. Exposure to mineral dusts (e.g., silicosis) 2. Chemical fumes (e.g., chlorine and ammonia) 3. Toxic drugs (e.g., bleomycin) 4. Radiation 5. Thermal injury 6. Oxygen toxicity 7. Immunologic diseases (e.g., sarcoidosis, 8. Goodpasture's syndrome, and collagen vascular disease) 9. Hypersensitivity to environmental agents (e.g., farmer's lung disease) 10. Tumors may appear initially as a postobstructive infection or adenopathy with peripheral infiltrates. 11. Lymphangitic spread of lung malignancy (looks like interstitial PNA)

List DSM-5 criteria for the diagnosis of Factitious Disorder Imposed on Self (FDIS) (Ch 114)

1. Falsification of psychological or physical signs or symptoms, or induction of disease or injury associated with identified deception 2. The individual presents to others as injured, ill, or impaired 3. The deceptive behaviour is apparent even in the absence of external incentives 4. The behaviour is not better explained by another mental disorder

List 5 RFs for sudden death in HCM (Ch 82)

1. Family member with a history of sudden cardiac death 2. History of syncope / SVT's 3. Massive LVH 4. Abnormal hypertensive response to exercise 5. Young age of diagnosis 6. Stimulant abuse 7. Ischemic heart disease

List 3 benefits of needle aspiration for a PTX over TT (tube thoracotomy) (Ch 77)

1. Fast 2. Less pain. 3. Less infection

List 5 causes of acute appendiceal obstruction and describe the pathophysiology of appendicitis including pain location (Ch 93)

1. Fecaliths (hard stools) (65%), 2. Appendicoliths (calcified deposits) 3. Lymphoid hyperplasia (primary or secondary to an enteric infection). 4. Foreign bodies (eg, vegetable matter, inspissated barium) 5. Tumors 6. Intestinal parasites.

List 5 agents used for procedural sedation (with doses): (Ch. 4)

1. Fentanyl ---- 1-2 mcg/kg IV 2. Ketamine--- 1-2 mg/kg IV; 4-5 mg/kg IM; 6 mg/kg IN 3. Propofol---- 0.5-2 mg/kg IV 4. Midazolam-- 0.05 mg/kg IV; 0.2 mg/kg IN 5. Etomidate--- 0.1 mg/kg IV

Describe the pharmacologic treatment options of Acute Abnormal Uterine bleeding (Ch 100)

1. Hormonal treatments (conjugated equine estrogen) 2. Combination oral contraceptive pills 3. Progestin-only contraceptive pills 4. NSAIDs 5. Ibuprofen 6. Mefenamic acid 7. Tranexemic acid 8. Naproxen

What is the typical sequence of ECG changes in pts with pericarditis? (the three stages) (Ch 82):

1. First hours to days of illness a. Diffuse ST seg. Elevation Reciprocal depression in aVR and V1 ***unlike MI, pericarditis has concave upward ST segments, and no T wave inversions, no dynamic changes, no reciprocal changes, and no evolution of Q waves*** b. PR seg depression 2. Normalization of ST segments Flattening of the T waves → T wave inversion 3. ECG normalizes (occasionally T waves can stay inverted).

List 11 indications for head CT post-seizure: (Ch. 18)

1. First seizure 2. suspected acute intracranial process 3. hx of acute head trauma 4. hx of malignancy 5. immunocrompromised 6. fever 7. persistent headache or altered LOC 8. hx of anticoagulation 9. new focal neuro signs 10. age >40 with no epilepsy hx 11. Focal before generalized

List 4 anatomic/physiologic differences between kids and adults in relation to trauma management: (Ch. 38)

1. Force more widely distributed through a child's more fragile frame = more injuries to bone, viscera, spinal cord 2. Kids have larger BSA = lose heat faster 3. Higher metabolism = greater proportional demand for fluids, lytes, O2, and glucose 4. Capacity to maintain BP despite 30-40% blood loss -SERIAL vitals and basically no role for inotropy

List four red-flag diagnosis associated with lower back pain (Ch 54):

1. Fracture (particularly with trauma or trauma hx). 2. Cauda Equina Syndrome (sudden compression of multiple lumbar or sacral nerve roots, note urinary retention 99.99% specific to rule-out) 3. Spinal infection (Spinal epidural abscess or osteomyelitis of the vertebral bodies - usually Staph. Aureus) 4. Malignancy

10 Causes of Apparent Life Threatening Event (ALTE): (Ch. 10)

1. GERD (18%) 2. Apneic NYD (9%) 3. Seizure (25%) 4. Infection (28%) 5. Congenital pulm or vasc malformation (2%) 6. Cardiac dysrrhythmias 7. Hypoglycemia 8. Breath holding spell 9. Head injury (Non-accidental trauma) 10. Reactive airways

Name 6 critical causes of vomiting: (Ch. 29)

1. GI - Boerhaave's, ischemic bowel, GI bleed 2. GU - testicular torsion 3. Neurological - meningitis, ICP incr. from tumour, ICH 4. Endocrine - DKA, pregnancy 5. Cardiac - MI 6. Toxin - sepsis, organophosphates, tylenol OD, etc

What are the 3 main areas providing afferent inputs to the vomiting center (in the medulla)? (Ch. 29)

1. GI tract --vagal & sympathetic inputs from viscera, pharynx, heart 2. Vestibular system --inner ear 3. Chemorecptor Trigger Zone (CTZ) --floor of 4th ventricle, partly outside blood brain barrier --activated by meds, toxins, hormones

What are the three main areas providing afferent inputs to the vomiting centre? (Ch. 29)

1. GI tract - vagal and sympathetic impulses 2. Vestibular system (inner ear) 3. CTZ - chemoreceptor trigger zone (this area is OUTSIDE the blood brain barrier affected by: Hormones; Peptides; Medications; Toxins)

List 5 clinical findings suggestive of tension PTX (Ch 77):

1. General signs of PTX : absent breath sounds, inflated ipsilateral chest, hyperresonance etc 2. Tachycardia (typically >120) 3. Hypoxia w/ profound dyspnea 4. Hypotension 5. JVD 6. Displacement of trachea to CONTRALATERAL side (away from PTX)

List four causes of dilated cardiomyopathy (Ch 82):

1. Genetic mutations of the cytoskeleton / sarcomere proteins 2. Environment... 3. Myocarditis 4. Peripartum DCM 5. ETOH (toxins) 6. Ischemia 7. Autoimmune - e.g. SLE

List 6 causes of each of genital ulcers (Ch 98)

1. Genital herpes 2. Primary syphilis 3. Chancroid 4. Lymphogranuloma venereum (rare) 5. Granuloma inguinale (rare) 6. Neoplasm/Trauma

What are two large vessel vasculitides? (Ch 118)

1. Giant cell arteritis (GCA) 2. Takayasu's arteritis

List 4 protozoal causes of gastroenteritis. For each, describe the clinical presentation (Ch 94)

1. Giardia lamblia Sudden onset diarrhea, malaise, weight loss, N/V, cramps, ++ flatulence and foul, floaty stool (greasy) 2. Entamoeba histolytica Aka: AMBEIASIS Most infections are asymptomatic 3. Cryptosporidium 4. Cyclspora cyetanesnsis

List 10 causes of ↓ vision post blunt eye trauma. (Ch. 71)

1. Globe rupture 2. Hyphema 3. Lens subluxation / dislocation 4. Traumatic uveitis 5. Traumatic hemorrhage 6. Retinal injury: Hemorrhage, detachment, tear, commotio retinae(Berlins edema) 7. Vitroeous hemorrhage 8. Orbital wall fracture 9. Retrobulbar hematoma 10. Optic nerve injury: Avulsion, trasection, compression, contusion of optic nerve

List 4 differential diagnoses for polyarticular, assymetrical arthritis (Ch 116)

1. Gonococcal arthritis 2. Lyme arthritis 3. ARF 4. Reactive arthritis 5. Viral arthritides

Describe 6 rectal STDs (Ch 96)

1. Gonorrhea (neisseria) 2. Chlamydia & lymphogranuloma venereum 3. Syphilis 4. Condyloma acuminatum 5. HSV 6. Chancroid (diagnosis of exclusion).

List 5 potential bacterial causes of pharyngitis (Ch 75)

1. Group A beta-hemolytic streptococcus 2. Non-group A strep. 3. Neisseria gonorrhoeae 4. Neisseria meningitidis 5. Mycoplasma pneumoniae 6. Chlamydia trachomatis 7. Corynebacterium diphtheriae 8. H. influenzae

Name at least 6 etiologies of exudative tonsillopharyngitis BESIDES Group-B strep: (Ch. 23)

1. Group-A strep 2. N. gonorrhea 3. EBV/mono 4. H. influenza 5. Corynibacterium diphtheriae 6. Candida species (?HIV?) 7. Strep pneumoniae 8. Chlamydia trachomatis

Name 6 viral arthritides (Ch 116)

1. Hepatitis B 2. Hepatitis C (mimics rheumatoid arthritis) 3. HIV (usually monoarticular) 4. Parvovirus B19 (cause of fifths disease in children) 5. Rubella virus; Rubella vaccine virus (look for maculopapular rash and lymph nodes) 6. Alphaviruses (ross river, chikungunya) - all mosquito borne

What are the main pharmacological classes of anti-emetics and their sites of action? (Ch. 29)

1. Histamine antagonists - acts on vestibular system ---dimenhydrinate and meclizine ---good for motion sickness 2. Muscarinic antagonists - anticholinergic/GI system ---scopolamine (patch) ---good for motion sickness only 3. Dopamine antagonists - acts on D2 receptors in CTZ ---prochlorperazine, haldol, promethazine ---metaclopramide (prokinetic - for gastric emptying) 4. Serotonin antagonists - acts on 5-HT receptors in CTZ ---ondansetron

What are the 3 causes of jaundice in pregnancy? (Ch. 28)

1. Hyperemesis gravidarum 2. Acute fatty liver of pregnancy 3. Intrahepatic cholecystitis

Define the three classes of hypertension relevant to emergency medicine practice (Ch 84)

1. Hypertensive emergency: "acute target-organ dysfunction, manifesting with newly developed clinical sequelae or consequential diagnostic test abnormalities" Poorly controlled chronic HTN: known HTN patient with asymptomatic elevated BP without end organ dysfunction Watch out for the following drugs as they can muddy the waters by increasing BP: NSAIDS/steroids/decongestants/appetite suppressants/over-the-counter stimulants/oral contraceptives/TCAs or rebound from short-acting antihypertensives (eg clonidine) 3. Elevated BP without prior history of HTN: can't be diagnosed with HTN in Ed (think white coat) but 70% of these patients go on the be diagnosed after appropriate w/o by GP or outpatient clinic

List 3 general ways to decrease ICP: (Ch. 41)

1. Hyperventilate 2. Diuretics or osmotic agents 3. CSF drainage

In head injuries, what are the secondary systemic insults we want to desperately avoid to prevent further neurologic damage? (Ch. 41)

1. Hypotension (SBP <90 mmHg) 2. Hypoxia (PaO2 <60 mmHg) 3. Anemia 4. Hyperthermia (>38.5 deg) 5. Hypercarbia 6. Coagulopathy 7. Seizures

5 Physiologic complications of massive transfusion: (Ch. 7)

1. Hypothermia 2. Electrolyte abnormality hypomagnesemia hypocalcemia hyper/hypo K+ 3. Acidosis (from schock or excess citrate) 4. Coagulopathy 5. Thrombocytopenia

5 "H" causes of pulseless electrically activity (PEA): (Ch. 9)

1. Hypovolemia 2. Hypoxia 3. Hydrogen ion (aciosis) 4. Hypothermia 5. Hypo/Hyperkalemia

Name 6 critical complications of vomiting: (Ch. 29)

1. Hypovolemia 2. Metabolic alkalosis 3. Hypokalemia 4. Mallory-Weiss tear 5. Boerhaave's syndrome 6. Aspiration

List 7 causes of altered LOC in the trauma patient: (Ch. 41)

1. Hypoxia 2. Hypoglycemia 3. Hypotension 4. Post-seizure 5. Post-intoxicating drugs 6. Brain/brainstem compression from mass effect 7. Bleed

Management steps for massive hemoptysis: (Ch. 24)

1. Identify massive hemoptysis and determine which lung 2. ABC-IV-O2-monitors-advanced airways-suction 3. Intubation with large bore ETT into "good" lung 4. CT scan of chest

What are some of the common causes of infective bursitis? (Ch 117)

1. Idiopathic : Most Common 2. Infection (most often due to Staphylococcus aureus) 3. Trauma (which may predispose to infection) 4. Rheumatologic disorders (eg, gout, pseudogout, ankylosing spondylitis, and rheumatoid and psoriatic arthritis) 5. Other systemic diseases (eg vasculitis)

List five causes of constrictive pericarditis (Ch 82)

1. Idiopathic or viral - 42 to 61 percent 2. Post-cardiac surgery - 11 to 37 percent 3. Post-radiation therapy - 2 to 31 percent, primarily after Hodgkin disease or breast cancer 4. Connective tissue disorder - 3 to 7 percent 5. Post-infectious (tuberculous or purulent pericarditis) - 3 to 15 percent 6. Miscellaneous causes (malignancy, trauma, drug-induced, asbestosis, sarcoidosis, uremic pericarditis) - 1 to 10 percent

Name three causes of acute mitral regurgitation (Ch 83):

1. Idiopathic rupture 2. Ischemia induced papillary muscle dysfunction 3. Post-MI papillary muscle rupture 4. Infective endocarditis

List 3 CV physiologic compensatory mechanisms in CHF (Ch 81):

1. Increased stroke volume Due to an increase in preload (Frank-Starling) This adaptation eventually plateaus, while creating massive myocardial oxygen demand. 2. Increased cardiac muscle size (hypertrophy +/- fibrosis/apoptosis) Initially helps the heart pump the same CO; but then forms more pathologic myocyte isoforms leading to more mass than capillary blood flow → fibrosis → myocyte death 3. Increase systemic vascular resistance Takes blood to the brain and heart - but away from skin, muscles, kidneys Forces the heart to do extra work

List common delayed complications of Endovascular repair (Ch 86)

1. Infection 2. Ischemic complications - Spinal cord ischemia - CVA - Extremity ischemia - Visceral ischemia (Celiac/SMA/Renal) - Post implantation syndrome (fever, leukocytosis and reactive inflammation, can get uni or bilateral reactive pleural effusions - SIRS to the max) 3. Aortoenteric fistula 4. Pseudoaneurysm (anastomotic aneurysm) 5. Chylous Ascites / Chylothorax 6. Endo Leak (blood flow outside the graft lumen - BUT contained within the aneurysm sac - and it grows) - in as many as 20% of patients who have had repair! Type I - separation of proximal / distal end of graft from aortic wall (incompetent seal) Type II - retrograde bleeding from branch vessels into aneurysm sac (eg lumbar arteries feeding) Type III - leaking between components of the graft (in a bifurcation graft) Type IV - leakage through the graft fabric itself (gore tex popped a hole!)

List the complications of a pacemaker insertion (Ch 80):

1. Infection ~%2 incidence Usually staph aureus 20-25% can be blood culture positive. So get cultures ***NEEDLE ASPIRATION UNDER FLUORO ONLY*** May cut insulation around pulse generator 2. Thrombophlebitis 30-50% incidence!!! But only 1% symptomatic secondary to good collateralization ⅓ of all patients can have complete venous occlusion Watch out for SVC syndrome Dx with Duplex U/S 3. Pacemaker syndrome Loss of AV synchrony

List 8 risk factors for a wound infection (Ch 59):

1. Injury > 8-12 hours old 2. Locations with poor blood supply ( Leg and thigh > arms > feet > chest > back > face > scalp) 3. Contaminated wound 4. Blunt mechanism 5. SubQ sutures 6. Repair material (sutures > staples > tape) 7. Anaesthesia with epi (really?) 8. High-velocity Missile injuries

Give 3 examples of IMMUNOLOGIC sequelae of IE (Ch 83):

1. Intermittent fever 2. Osler nodes / roth spots / rheumatoid factor 3. Malaise, chills, anorexia 4. non specific : weakness, myalgias, back pain, dyspnea, chest pain, cough, h/a 5. murmur usually absent in EARLY disease present in < 30%

List 4 types of non-occlusive vascular trauma? (Ch. 48)

1. Intimal flap/dissection 2. Compartment syndrome 3. AV fistula 4. Pseudoaneurysm* (tear in adventitia) *true aneurysm involves all 3 layers of vessel

Indications for repair of nerve injuries in hands: (Ch. 50)

1. Involvement of motor branch of nerve 2. Digital nerve injuries proximal to DIP 3. Clean single nerve lacerations

What are the zones of burns?(Pathophysiology)(Ch. 63)

1. Irreversible coagulation and necrosis, formed immediately 2. Ischemia with impaired microcirculation 3. Transient hyperemia

List 5 ECG findings of HF (Ch 81):

1. LVH: 2. LAE 3. RAE 4. RV Strain: ST depression and T wave inversion in the leads corresponding to the right ventricle, i.e The right precordial leads: V1-3, often extending out to V4 The inferior leads: II, III, aVF, often most pronounced in lead III as this is the most rightward-facing lead 5. Fascicular blocks and bundle branch blocks

List the components of the popliteal fossa (Ch 57):

1. Laterally = biceps femoris tendon 2. Medial = semimembranosus and semitendinosus tendons 3. Superiorly = above muscle bellies 4. Inferiorly = two heads of the gastrocnemius muscles Interiorly = popliteal artery + vein, peroneal and tibial nerves

List 5 orthopedic emergencies (ORTHO consult!): (Ch. 49)

1. Long bone fracture 2. Open-book pelvis fracture 3. Neurovascular compromise (AVN, compartment syndrome, vascular injury, etc) 4. Open fractures 5. Fractures involving joints

List 5 high risk fractures for child abuse (Ch 66)

1. Long bone fracture in a preambulatory infant 2. Metaphyseal fractures 3. Rib fractures 4. Scapula fractures 5. Spinous Process fractures(Skull, multiple, wide >3cm, growing, involving more then one cranial bone, occipital)

What leads to metabolic alkalosis in vomiting? (Ch. 29)

1. Loss of H+ ions in vomit (from gastric content) 2. Alkalosis further promoted by: --volume contractions --hypokalemia --chloride shift causing H+ to shift intracellularly --increased aldosterone

List 5 CT findings suggestive of increased ICP: (Ch. 20)

1. Loss of basilar cisterns 2. effaced sulci (gyri pushed together) 3. decreased ventricular size 4. midline shift 5. loss of grey-white differentiation

What are the 3 Anti-Phospholipid antibodies? (Ch 118)

1. Lupus anticoagulant: most thrombogenic 2. Anticardiolipin antibody 3. Anti-β2-glycoprotein I antibody.

Post cardiac-arrest care goals: (Ch. 9)

1. MAP 70-90 mmHg 2. CVP 10-15 mmHg 3. Hgb >100 4. Lactate <2 5. Temp 32-34 deg ? 6. SaO2 94-98% 7. ScVO2 >65% 8. DO2 400-500 ml/min 9. VO2 >90 ml/min

High risk populations for pulmonary contusions? (Ch. 45)

1. MCV 2. pedestrian struck by motor vehicle 3. shock-wave injury (bomb blast) 4. Ballistics or missle injury 5. CHILDREN (especially if NO rib fracture!!)

List 6 causes of Horner's Syndrome: (Ch. 43)

1. MS 2. Encephalitis 3. Tumours 4. Lateral Medullary syndrome (Wallenburg's) 5. Thyroid mass or removal (iatrogenic) 6. Trauma to base of neck 7. Pancoast tumour 8. Thoracic aneurysm 9. Sympathectomy

List 4 causes of pneumopericardium and one specific PEX finding (Ch 82)

1. Malignant: Esophageal cancer / lung cancer erosion 2. Iatrogenic: Post-EGD/thoracic surgery 3. Infectious: necrotising staph. Aureus 4. Post-traumatic: blunt chest trauma PEX finding: Hamman's crunch!

Describe treatment of monkey bites(Ch. 54)

1. Mechanical debridement and vigorous irrigation 2. Valacyclovir 1g orally qh x 14 days or acyclovir 800mg po Q 5 times daily x 14 days(Highly virulent herpes simiae) 3. Amox-clav

List the types of small bowel obstruction (Ch 92)

1. Mechanical obstruction: 2. Neurogenic / functional obstruction: 3. Pseudo-obstruction: 4. Closed loop vs. open loop

Describe 3 injuries common in little-leaguers elbow (Ch. 52)

1. Medial epicondyle avulsion # (wrist flexors) 2. Compression # of the subchondral bone of radial head 3. and/or the capitellum (lateral condyle)

What are the common causes of altered LOC in elderly? (Ch. 16)

1. Medication changes 2. Over-the-counter meds 3. Infection 4. Alteration in living environments 5. Stroke

List 4 characteristics of malingering (Ch 114)

1. Medicolegal context of the presentation (e.g., the patient was referred by his or her attorney) 2. Marked discrepancy between the person's claimed stress or disability and objective findings 3. Poor cooperation during the diagnostic evaluation or poor compliance with previously prescribed treatment regimens 4. The person exhibits or has a history of antisocial behaviour

3 main settings where pulse oximetry is NOT helpful: (Ch. 5)

1. Methemoglobinemia --cyanide poisoning, antimalarials 2. Carboxyhemoglobinemia --CO poisoning 3. Poor light penetration --nail polish, LOW perf states, dark-skin

List 4 agents used for rapid tranquilization and their usual doses (Ch 110)

1. Midazolam - 2.5 to 5mg IM (rapid onset) 2. Lorazepam - 1 to 2mg PO or IM 3. Diazepam - 5 to 10 mg PO or IM (longer acting) 4. Haloperidol - 5 to 10mg PO or IM 5. Ziprasidone - 10 to 20mg PO or IM 6. Olanzapine - 10mg PO or IM

What are the depths of procedural sedation (PSA)? (Ch. 4)

1. Minimal - anxiolysis 2. Moderate - "conscious sedation" 3. Dissociative - protective reflexes intact 4. Deep - unrousable unless w/ painful stimulus 5. General anaesthesis - unrousable, no reflexes

What are two presentations of gonococcal arthritis (Ch 116)?

1. Mono-oligoarticular arthritis 2. True disseminated gonococcal infection (sometimes termed arthritis-dermatitis syndrome: bacteremia, diffuse migratory arthralgias, characteristic skin lesions, and tenosynovitis)

List 5 Risk Factors for Pelvic Inflammatory Disease (Ch 98)

1. Multiple sexual partners 2. Age <25 3. Partner with an STI 4. History of STI 5. History of PID

What are the keystones of management for urolithiasis (Ch 99)

1. NSAIDS (decrease pain, decrease ureterospasm, and renal capsular pressure by dropping GRF to the obstructed kidney) 2. Fentanyl prn 3. Ondansetron prn 4. Fluids (although no evidence for this) < note: this is different from nephrolithiasis!

What are the 6 broad categories for causes of constipation? (Ch. 32)

1. Neurologica: Parkinson's, MS, spinal cord injury 2. Metabolic: DM, hypercalc, hypomag, hypothyroid 3. Structural: tumour, stricture, volvulus, rectocele 4. Meds: opiates, iron, antidep, diuretics, antichol, antipsych 5. Psych: abuse, eating disorders, depression 6. Other: immobility, dehydration, post-op, dietary

List 4 CXR findings of esophageal perforation (Ch 89)

1. Pneumothorax 2. Subcutaneous air / emphysema 3. Hydropneumothorax 4. Pneumomediastinum 5. Pleural effusions 6. Pulmonary infiltrates 7. Wide mediastinum

List the 2 most common causes of viral gastroenteritis. Differentiate the two based on patient population and course of illness (Ch 94)

1. Norovirus 2. Rotavirus ** (the only one with a vaccine) (Note also don't forget: Sapovirus, Adenovirus, Astrovirus) These usually follow the 60 hour rule - i.e. they take < 60 hrs to incubate and last < 60 hrs. All can have N/V/D, belly cramping, fevers, myalgias, headaches.

What conditions make the Ottawa Ankle Rule not valid? (Ch 58)

1. Not applicable in subacute and chronic injuries 2. Not validated in kids < 5-years-old. 3. Does NOT apply to hind or fore foot injuries. 4. Not reliable in patients with: Intoxication Distracting injuries Gross swelling that prevents palpation of bony tenderness Peripheral nerve damage

List at least 2-indications for performing an arthrocentesis (Ch 116)

1. Obtain joint fluid for analysis (only definitive way to diagnosis septic or crystal arthritis!) 2. To drain tense hemarthrosis (trauma or hemophilia) 3. Assess for blood - determining whether a laceration communicates with the joint space 4. Intra-articular injection of analgesics and anti-inflammatories in the setting of acute/chronic arthritis

List features that are typical for IBS (Ch 95)

1. One out of three days of: Bloating Abdominal pain (relieved with defecation) Constipation OR diarrhea OR both Upper GI symptoms (dyspepsia / nausea) 2. Mild abdominal tenderness

The basic approach to post-cardiacarrest patient: (1:2:2:3 Rule) (Ch. 9)

1. One set of full vitals 2. Two machines to call for: ECG, CXR 3. Two blood tests: Full cardiac labs and ABG 4. Take a history, do exam, and look at the old charts

List 4 categories of medical therapy for IBD and give one example for each (Ch 95)

1. Oral aminosalicylates (5-ASA, mesalamine) 2. Steroids (methylprednisolone, prednisone) 3. Immunosuppressants (cyclosporine) 4. Antibiotics (Flagyl + Cipro)

What is the classic triad of Behcet's disease? (Ch 118)

1. Oral aphthous ulcers 2. Genital ulcers 3. Uveitis

Five types of facial fractures: (Ch. 42)

1. Orbital (blowout most common, beware retrobulbar hematoma) 2. Midface fracture (Le Fort fractures, tripod fracture) 3. Mandibular fracture 4. Dental/alveolar --Ellis system for dental fractures --Alveolar ridge fracture 5. TMJ fractures

What are some major considerations when managing the airway in penetrating neck trauma? (Ch. 44)

1. Orotracheal RSI - consider an awake look first -- C/I if massive facial trauma or laryngotracheal injury 2. Nasotracheal Intubation - only in breathing pts --C/I if suspected midface/basilar skull/laryngeal #s 3. Surgical Intervential Airway --C/I if large neck hematoma or laryngeal injury or <10yo 4. Awake fibre optic

Give 3 examples of VASCULAR sequelae of IE (Ch 83):

1. Osler nodes 2. Splinter hemorrhages 3. Janeway lesions 4. Roth spots 5. Splenomegaly

What are the 3 pathological processes leading to elevated bilirubin? (Ch. 28)

1. Overproduction of heme 2. Failure to conjugate (hepatocellular dysfunction) 3. Decreased clearance (biliary excretion problem)

List 6 risk factors for ectopic pregnancy: (Ch. 33)

1. PID 2. IUD 3. smoker 4. IVF 5. previous ectopic 6. pelvic/tubal/ovarian surgery

Equipment needed for neonatal resuscitation: (Ch. 11)

1. PPE - gown, gloves, mask 2. Warm blankets to dry infant 3. Radiant warmer 4. Airway stuff (suction, BVM, O2, scope, ET tube/stylet) 5. Airway securing stuff (tape, scissors) 6. Confirmatory devices: steth, CXR machine 7. Umbilical vein cath stuff (cath #3.5 and #5, 3 way stop cock, provodine, drapes, scalpel, hemostat, suture)

List 6 overuse syndromes of the knee and lower leg (Ch 57):

1. Patello-femoral pain syndrome 2. Iliotibial band syndrome 3. Peripatellar Tendinitis 4. Plica syndrome (redundant folds of synovium) 5. Popliteus tendinitis 6. Bursitis

Describe the stepwise management of a Rectal FB (Ch 60):

1. Patient sedation / analgesia 2. Removal with single digit and abdo pressure 3. Anoscopy / Vaginal Speculum w/ Ring Forceps 4. Advancement of foley catheter past object with gas insufflation to "Break Vacuum" effect 5. Vacuum device 6. Surgery

List 3 phases of delay in the management of AMI; and describe the time-points in ED management of AMI (Ch 78):

1. Patient-bystander Factors that prevent the patient to make initial contact with medical personnel (EMS) 2. Preadmission Time from decision to seek medical care to first contact at the ED 3. Hospital Delay to PCI / lysis

Describe key management points in the treatment of appendicitis. (Ch 93)

1. Patients should remain NPO. IV fluids (normal saline or lactated Ringer's) may be administered to maintain hydration and support hypotensive patients. 2. Pain and nausea should be treated with parenteral opiate analgesia and antiemetics. There have been a number of good quality studies that support the concept that opiate analgesia does not negatively affect a patient's abdominal examination when the patient has an abdominal condition that requires surgery. 3. Antibiotics 4. Surgery!

List 3 features of sacral sparing: (Ch. 43)

1. Perianal sensation intact 2. Rectal motor function 3. Great toe flexor activity

What are the two types of cavities caused by bullet wounds? (Ch 65)

1. Permanent cavity 2. Temporary cavity

List 5 indications for Rescue PCI (Ch 78)

1. Persistent chest pain 2. Persistent ST elevation 3. Cardiogenic shock 4. Post-reperfusion ischemia 5. Ventricular dysrhythmias

What are two rare hip conditions predominantly affecting young (2-14 year-old) males (Ch 56)?

1. Perthes disease AVN of the femoral head (more common in boys) 2. Slipped capital femoral epiphysis (SCFE) Peaks at 13.5 yrs (more common in boys!)

4 main categories of indications for Trauma Team Activation: (Ch. 36)

1. Physiologic 2. Anatomic 3. Mechanistic 4. Logistical

List 4 causes of pelvic pain in women >20 wks pregnant that are of reproductive tract origin: (Ch. 33)

1. Placenta previa 2. Placenta abruption 3. Round ligament pain 4. Braxton hicks

List 3 views of the shoulder (Ch. 53)

1. True anteroposterior or '45-degree lateral' 2. Transscapular lateral or 'Y-view' 3. Axillary lateral views

List common prehepatic/posthepatic/ and hepatic causes for jaundice: (Ch. 28)

1. Prehepatic (overproduction of heme) ---hemolysis, hypoalbuminemia, acidemia, drugs 2. Hepatic (failure to conjugate) ---toxins (etoh, tylenol), budd-chiari, hepatitis (viral, autoimmune), HELLP, Fatty liver 3. Posthepatic (decreased clearance) ---Gallstone disease (CBD stone or cholangitis)

List at least 5 emergency causes of a sore throat - the must-not miss diseases! (Ch 75)

1. Primary HIV 2. Epiglottitis 3. Bacterial tracheitis 4. Corynebacterium Diphtheriae 5. Ludwig's angina 6. Epstein barr virus 7. Cancer 8. Gonorrhea / Herpes Simplex 9. Foreign body 10. Angioedema 11. Peri-tonsillar abscess 12. Retro-pharyngeal abscess

What are some of the clinical findings of a vascultiis? (Ch 118)

1. Pulmonary infiltrates and nodules 2. Alveolar hemorrhage 3. Glomerulonephritis 4. Upper airway disease 5. Purpura 6. Peripheral nervous system involvement 7. Central nervous system involvement

Describe PALS cardiac arrest algorithm: (Ch. 10)

1. Quality CPR (>100bpm, min pauses) 2. Monitor and IV set up 3. Ventilation: 10-12 breaths/min 4. Shock shockable rhythm @ 2-4 J/kg 5. Epineph 0.01 mg/kg q 3-5 min 6. Pulse check q 2 min 7. Address reversible causes (5Hs, 5Ts)

What are the components of prehospital management of AMI (Ch 78):

1. Recognition that this may be an ACS event 2. Activation of the EMS system 3. BLS care (for cardiac arrests) 4. ACLS care: - 12 lead ECG - to rule in/out STEMI - Administration of chewable ASA 180-325 mg PO - Administration of SL nitroglycerin - Possibly administration of oxygen if Sp02 <94%

Describe PALS septic shock algorithm: (Ch. 10)

1. Recognize SICK kid --> IV/IO access, high flow O2 2. Fluid resusc (20cc/kg) 3. Start antibiotics 4. Correct hypoglycemia/hypocalcemia 5. Fluid refractory shock: inotrope IV/IO (dopamine, epi) 6. Catechol-resistant shock: hydrocortisone 7. Add vasopressors vs norepi as needed for warm vs cold refractory shock 8. Still refractory: ECMO

List 6 non-pharmacologic therapies for anxiety (Ch 112)

1. Reduction of environmental stimulants (quiet, private room) 2. Dimming of lights, with music/aromatherapy 3. Collateral help from family, social worker 4. Breathing techniques 5. Avoidance of caffeine and EtOH 6. Psychotherapy 7. High intensity, supervised exercise

List 10 common causes of hip pain without an obvious fracture (Ch 56):

1. Referred pain - lumbar/spine/knee Diskitis Lumbar disk herniation Knee pathology 2. Hip bone pathology AVN of the femoral head Perthes disease / SCFE Occult fracture Bone tumour Osteoarthritis Transient synovitis / septic joint / osteomyelitis 3. Hip soft tissue pathology Bursitis Iliopsoas abscess DVT Arterial insufficiency 4. Abdominal pathology Inguinal hernia Retroperitoneal pathology PID Nephrolithiasis 5. GU pathology UTI Testicular torsion External genitalia abnormalities 6. Sports med stuff Sports hernia FAI Labral injury Osteitis pubis

What is your initial care for a snake bite out of hospital?(Ch. 55)

1. Remove victim from snake - bring snake with you if possible or take good pictures 2. Rapid transportation 3. Remove clothing or jewlery around the bite 4. Slow the spread of venom, calm the patient down, avoid movement of effected area(splint!), no alcohol! 5. Consider the monash method or immobilization and compression technique. Can use snuggly fitted band immediately after bite(<30 minutes) 6. Ice applied to the bite wound(Comfort) 7. Document the evolution of wound and limb edema Hospital ASAP!!!! Dont: Incise bitewounds, suck them, drink whiskey, clam juice, split chickens or electrocute them

Two major goals of therapeutic intervention for post-arrest brain: (Ch. 8)

1. Restore cerebral blood flow 2. Prevent secondary tissue damage

List four causes of Mitral Stenosis (Ch 83):

1. Rheum. Heart disease (#1 Cause at >85%) - Note: symptoms develop after the latency period of 10-30 yrs post Group A strep. infection 2. Congenital mitral stenosis / annular calcification 3. Aging 4. Post-infection 5. Degeneration

List 6 DDx for atraumatic nonseptic bursitis (Ch 117)

1. Rheumatoid arthritis 2. Pseudogout 3. Ankylosing spondylitis 4. Hypertrophic pulmonary osteoarthropathy 5. Oxalosis 6. Gout

What are four key mechanisms of injury in blunt abdominal trauma? (Ch. 46)

1. Rupture/burst (i.e. seatbelt injuries) 2. Crust (i.e. direct blow) 3. Acceleration/deceleration 4. Iatrogenic (i.e. PPV, CPR/Heimlich, tube thoracotomy)

List 8 life-threatening causes of ABDOMINAL pain (aka abominable pain): (Ch. 27)

1. Ruptured ectopic pregnancy 2. Ruptured/leaking abdominal aortic aneurysm 3. Ischemic bowel 4. Intestinal obstruction 5. Perforated viscus 6. Acute pancreatitis 7. Appendicitis/diverticulitis 8. Ascending cholangitis

List the 8 *life-threatening* causes of headache: (Ch. 20)

1. SAH 2. Meningitis 3. CO poisoning 4. Brain tumour 5. Temporal arteritis 6. Acute angle closure glaucoma 7. Intracerebral hemorrhage 8. Cerebral venous sinus thrombosis

What are the common causes of altered LOC in an infant? (Ch. 16)

1. Sepsis 2. Trauma (non-accidental?) 3. Metabolic

List 4 differential diagnoses for monoarticular arthritis (Ch 116)

1. Septic arthritis 2. Gout 3. Pseudogout / CPPD 4. Osteoarthritis 5. Trauma / hemarthrosis

Relative contraindications to therapeutic hypothermia: (Ch. 9)

1. Severe cardiogenic shock 2. Uncontrolled bleeding (cold worsens coagulopathy) 3. Pregnancy 4. Coagulopathy/ESRD 5. Drug O/D or status epilepticus

Indications for ICP monitoring (2 of them)? (Ch. 41)

1. Severe head injury (GCS <9) 2. Moderate head injury (GCS 9-12) who cannot be monitored with serial neurological exam

Name 3 protective factors against suicide (Ch 115)

1. Social support & family connectedness 2. Pregnancy 3. Parenthood (particularly for mothers) 4. Religiosity and participating in religious activities

List 5 somatic symptom and related disorders (Ch 113)

1. Somatic symptom disorder (SSD) 2. Illness anxiety disorder (formerly hypochondriasis) 3. Functional neurological symptom disorder (formerly conversion disorder) 4. Factitious disorder 5. Psychological factors affecting other medical conditions 6. Other specified somatic symptom and related disorders 7. Unspecified somatic symptom and related disorders

List 5 Ultrasound findings of acute cholecystitis (Ch 90)

1. Sonographic Murphy's sign 2. Gallbladder wall thickening (>5 mm)* 3. Gallbladder wall edema (double wall sign) 4. Pericholecystic free fluid* 5. Gallstones* *** = 90% PPV of acute cholecystitis

How do you manage high pressure injection injuries to hand? (Ch. 50)

1. Splint and elevate affected hand 2. Analgesia 3. Tetanus prophylaxis 4. Broad-spectrum antiobiotics 5. Avoid digital blocks (risk of iatrogenic compartment syndrome) 6. Emergent hand surgeon consult

Describe PALS tachycardia algorithm: (Ch. 10)

1. Stable or unstable?? 2. *Unstable* --> shock 3. If *stable wide complex*: adenosine or amiodarone if regular... or shock 4. If *stable narrow complex*: vagal moves -->adenosine 5. If *sinus tachy* --> treat cause

Which pneumonias can present with cavitating lesions (abscesses) on x-ray? (Ch 76)

1. Staph. Aureus (MRSA) 2. Anaerobes 3. Aerobic gram-negative bacilli, 4. Fungal disease ie PCP 5. TB 6. Noninfectious processes (e.g., malignancy and pulmonary vascular disease).

List 5 common bacteria responsible for infective endocarditis (Ch 83):

1. Staphylococcus aureus (30%) 2. Strep. Viridans - 18% 3. Enterococci - 10% 4. Coag-negative staph. - 10% 5. Strep. Bovis 6. Other strep. 7. non-HACEK gram negative bacteria 8. Fungi (candida, aspergillus) 9. HACEK (haemophilus, actinobacillus, cardiobacterium, eikenella, kingella) 10. Polymicrobial

List at least 3 of 7 techniques for reduction of an anterior shoulder dislocation (Ch. 53):

1. Stimson / hanging weight 2. Traction-contertraction 3. Snowbird 4. External rotation method of leidelmeyer 5. Milch 6. Scapular manipulation 7. (Bonus): Hippocratic method

List 5 complications of prosthetic valves (Ch 83)

1. Structural failure (rare, present in shock). 2. Thrombosis of the valve (CHF) 3. Systemic embolization (thromboembolic) (avoided by higher INR goal of 2.5-3.5 , usually CNS strokes, at high risk for hemorrhagic conversion) 4. Hemolysis (hemolytic anemia due to sheer forces fatigue, jaundice, dark urine, dyspnea) 5. Endocarditis (highest during the initial months post-op early <60 days) Note: Vary based on valve type: mechanical bioprosthetic

What are the deep spaces of the neck? List 4 deep space infections of the neck (Ch 75)

1. Submandibular space: sublingual and submaxillary spaces. Think Ludwig's Angina here 2. The parapharyngeal space contains the carotid artery, the jugular vein, the cervical sympathetic chain, and cranial nerves IX through XII 3. The retropharyngeal space lies in the midline and extends from the base of the skull to the superior mediastinum (level of T2) 4. Danger space Posterior to the retropharyngeal space lies this "danger" space, which extends from the base of the skull to the diaphragm 5. The prevertebral space extends from the base of the skull to the coccyx

Cerebral edema on CT? 3 key points in identifying cerebral edema on CT: (Ch. 41)

1. Sulcal effacement 2. Loss of gray-white differentiation 3. Ventricular compression

List 4 unique considerations in the management of pregnant trauma patient: (Ch. 37)

1. Supine hypotensive syndrome (of pregnancy) 2. Physiologic alterations: incr blood vol. may mask shock, incr cardiac output by 40% --> incr bleeding, less venous return --> incr. blooding from leg wounds) 3. Pulmonary considerations: decr. FRC, incr O2 consumption, incrased ventilation leads to hypoCO2, difficult BVM --> so be quick to tube 4. GI considerations: reduced sphincter tone leads to incr. apiration risk, incr acid production --> early decompression

Describe PALS bradycardia algorithm: (Ch. 10)

1. Suppl O2 2. Start CPR if HR <60 and signs of HYPOtension 3. Epineph 0.01 mg/kg IV/IO q 3-5 min 4. Atropine 0.02 mg/kg IV/IO (one repeat) 5. Consider trans-cutaneous pacing

List 5 types/sites of anorectal abscess (Ch 96)

1. Supralevator abscess 2. Intersphincteric abscess 3. Ischiorectal abscess 4. Postanal abscess 5. Perianal abscess

List 5 RFs for 1° spontaneous PTX (Ch 77):

1. Tall 2. Male 3. Smoker 4. Changes in Ambient atmospheric pressures 5. Hx Mitral valve prolapse (controversial) 6. Marfan Syndrome 7. Family History of primary PTx

List 6 DDx for tendinopathy (Ch 117)

1. Tendon rupture 2. Ligamentous injury 3. Inflammatory arthritis (ex. rheumatoid) 4. Fractures (ex. avulsion) 5. Tumors 6. Tenosynovitis

5 "T" causes PEA: (Ch. 9)

1. Thrombosis (cardiac - MI) 2. Thrombosis (resp - PE) 3. Toxins 4. Tamponade 5. Tension pneumothorax

2 most IMPORTANT determinants of good outcome in cardiac arrest: (Ch. 9)

1. Time to defibrillation 2. Quality CPR

What are the common causes of altered LOC in teens or young adults? (Ch. 16)

1. Toxic ingestion 2. Recreational drug use 3. Trauma

What are the common causes of altered LOC in children? (Ch. 16)

1. Toxic ingestions 2. Sepsis 3. Trauma

List 3 types of occlusive vascular trauma? (Ch. 48)

1. Transection 2. Thrombosis 3. Reversible arterial spasm (esp in kids)

Some specific disorders you need to watch for when dealing with elderly patient trauma: (Ch. 39)

1. Traumatic brain injury - contusion, bleed, skull # 2. Thoracic trauma - rib/sternal #, pneumothorax (in COPD) 3. Upper extremity - distal radius > humeral head > elbow 4. Lower extremity - akle #, hip/pelvic #, tibia plateau #

Specific management to preserve brain, post-arrest: (Ch. 8)

1. Treat hypotension, hypoperfersion, hypoxia (keep vitals and ABG in normal range) 2. Maintain body temp 3. Treat HYPERglycemia 4. Seizure management 5. Immobilization/sedation/head position

List 5 causes of restrictive cardiomyopathy (Ch 82):

1. Tropical endomyocardial fibrosis 2. Amyloidosis 3. Scarcoidosis 4. Hemochromatosis*** only treatable kind! 5. Slcerderma 6. Neoplasm 7. Glyocogen storage diseses

Describe the treatment of pericarditis associated with: Uremia, Neoplasm, and SLE (Ch 82):

1. Uremia Find any underlying cause (infection) Intensive dialysis **NSAIDS are contraindicated! And ineffective. Steroids for those who don't respond to dialysis 2. Neoplasm Think lung, breast, lymphoma, leukemia metastatic disease (primary dz very rare) Pericardiocentesis With sclerosing or chemo agents 3. SLE / RA / connective tissue diseases Corticosteroids

List 5 lab or investigative findings in bacterial endocarditis (Ch 83):

1. Usually nonspecific (leukocytosis, CRP elevated, anemia, microscopic hematuria, abnormal CXR or ECG) 2. Need three (3) blood cultures from separate sites 3. TTE non-diagnostic in 20% of people (60% sens) 4. TEE much better 5. Duke Criteria are the best: 95% sens, 99% spec.

List 6 UNCOMMON causes of dyspnea: (Ch. 25)

1. Valvular heart disease 2. Cardiomyopathy 3. Ruptured diaphragm 4. Thyrotoxicosis 5. Guillain-Barre or Myasthenia 6. Tick paralysis

List 10 differential diagnoses for acute painless visual loss(Ch. 71)

1. Vascular occlusion: CRAO, CRVO 2. Retinal detachment 3. Vitreous detachment 4. Posterior vitreous detachment 5. Hemianopsia due to stroke 6. Pituitary tumor 7. Macular degeneration 8. Optic neuritis and non-arteritic ischemic optic neuropathy 9. Toxic: Ethylene glycol, methanol, thiamine deficiency 10: Hysteria

What precautions should be taken prior to engaging in a potential sexual abuse case? (Ch 67)

1. Victim consent 2. Evidence gathering(Chain of custody, powder free gloves!)

List 5 diagnostics tests that may be utilized for dysphagia.List 5 diagnostics tests that may be utilized for dysphagia (Ch 89)

1. Video esophagography 2. Barium swallow 3. Manometry 4. Impedance monitoring 5. CT - imaging with/without contrast for vascular / malignancy / external compression

List 8 differential diagnoses for hepatitis (Ch 90)

1. Viral hepatitis 2. Alcoholic hepatitis 3. Bacterial 4. Fungal 5. Parasitic 6. Medication induced (e.g. tylenol) 7. Nutritional / herbal supplements (e.g. mushrooms) 8. Autoimmune

7 NON-emergent causes of peripheral neuropathy: (Ch. 13)

1. Vitamin B12 deficiency 2. Diabetic neuropathy 3. Toxins (alcohol) 4. External compression 5. Paraneoplastic syndromes 6. Trauma 7. Connective tissue disorders

Describe 6 management priorities in acute pancreatitis (Ch 91)

1. Volume resuscitation - ideally Ringer's Lactate 2. Pain control 3. Correct any absolute electrolyte deficits: 4. Correction of hyperglycemia (insulin deficiency) 5. Treat nausea 6. Initiate oral or enteral feeding ASAP 7. Consider early U/S post-diagnosis and then → ERCP for anyone with cholangitis or biliary obstruction (jaundice, elevated bilirubin, sepsis) 8. Consider antibiotics only if: Septic (tachycardia and tachypnea don't count) Evidence of infected necrotizing pancreatitis Usually surgical, endoscopic or radiologic intervention only considered until ~4 weeks when the collection becomes walled off.

5 steps of neonatal resuscitation: "WARM and PASS the neonate; Ventilate then Intubate; Must compress with a LOW Rate to avoid a cold fate" (Ch. 11)

1. Warm/dry, PASS (position access, stimulate, suction) 2. Ventilate (CPAP or PPV) then BMV then intubate 3. Compress at low rate: CPR if HR<60 4. Resusc meds through ETT (epi) or IV (epi, naloxone, glucose) if indicated 5. Therapeutic hypothermia (33.5-34.5 deg)

CXR findings for blunt aortic injury: (Ch. 45)

1. Wide mediastinum in 50-92% 2. obscured aortic knob 3. loss of PA window 4. Displaced NG tube 5. widened paratracheal stripe 6. depressed left main bronchus 7. left pleural effusion 8. left apical cap 9. left tracheal deviation

When do you start NAPQI in acute ingestion(Ch. 148)

1. Within 4-8 hours in acute ingestion - For tylenol regular and ES peak concentration in 1 hour, complete absorption in 4 hours 2. Anyone presenting with potential hepatocellular toxicity from massive overdose 3. Anyone with previous liver disease for whom an APA level cannot be measures within 8 hours 4. Ask your local tox for more details!

List 7 symptoms of Esophageal FB (Ch 60):

1. Witnesses or self reported history of ingestion (eg coin / food / toys / bones / batteries / wood / glass) 2. Atypical chest pain / neck pain 3. FB sensation 4. odynophagia / dysphagia 5. drooling 6. retching / vomiting 7. Anorexia, 8. wheezing

List 3 risk factors for atypical presentation of ACS (Ch 78):

1. Women 2. Elderly (Especially > 85 yrs) 3. Diabetics 4. Non-Caucasian 5. No prior hx of MI 6. Dementia

Key points for management of phalangeal and metacarpal fractures: (Ch. 50)

1. X-ray is part of the physical exam for hands. 2. Shaft fractures cannot tolerate any rotation. 3. Fractures of the head or bases are complicated! Consult.

Subungal hematoma management: (Ch. 50)

1. X-ray to ensure no fracture 2. relieve subungal pressure with trephination 3. distal phalanx # with sub. hem. should be tx like open # with prophylactic antibiotics

6 features that define an ictal event: (Ch. 18)

1. abrupt onset 2. brief duration 3. loss of consciousness 4. purposeless activity 5. unprovoked 6. post-ictal stage

List 6 causes of LGIB in pediatrics: (Ch. 30)

1. anorectal fissure 2. infectious colitis 3. IBD 4. juvenile polyps 5. intussusception 6. Meckel's diverticulum

Five ALL-important questions to ask yourself when approaching a patient with "diplopia": (Ch. 21)

1. binocular or monocular? 2. if binocular, is it from an orbitopathy (mechanical)? 3. if binocular, is it from a CN palsy (III, IV, VI?) 4. if binocular, is it involving brainstem? 5. if binocular, is it a neuromuscular d/o?

Features of *pulmonary* blood that helps us confirm hemoptysis: (Ch. 24)

1. bright red 2. alkaline 3. slightly darker red clots

Most common causes of hemoptysis: (Ch. 24)

1. bronchitis/bronchiectasis 2. cancer 3. TB 4. CF 5. AV malformations

List 5 common presentations of conversion disorders (Ch 113)

1. weakness or paralysis; 2. abnormal movements; 3. swallowing symptoms; 4. dysphonia or slurred speech; attacks or seizures; 5. anesthesia; and 6. visual, olfactory, or hearing disturbances.

Toxic dose of buprivicaine: (Ch. 4)

1.5 mg/kg 3 mg/kg WITH epi

What are the 4 elements of COPD

1. chronic inflammation from trachea down to alveoli. - Mediated by a neutrophil/CD8_/macrophage/lymphocyte respones (different from ASTHMA which is primarily eosinophils) 2. Mucous plugging - Increase in goblet cell proliferation and size, increase mucous production and plugging 3. Endothelial barrier damage caused by inhibition of the body's mucociliary response 4. Centrilobar emphysema - Loss of connective tissue and subsequent airway patency/stenting through expiration via radial support (OBLITERATION) COPD is not fully reversible.

List some of the risk factors for gout (Ch 116):

1. chronic obesity, 2. hypertension, 3. diabetes, 4. thiazide diuretics 5. cyclosporine use, 6. lead or radiocontrast exposure. 7. Purine-rich diets (meat; seafood, especially anchovies and shellfish; beer; and legumes) predispose at-risk individuals to attacks; 8. high-fructose corn syrup and soft drinks are also implicated.

List main treatment strategies for acute angle closure gloucoma: (Ch. 22)

1. decrease production of aqueous (timolol, acetazolamide drops) 2. decrease incraocular pressure (head of bed at 30 deg) 3. constrict pupils (if IOP >30) (pilocarpine drops) 4. decrease inflammation (prednisolone drops)

Seven contraindications to NIPPV: (Ch. 2)

1. decreased LOC 2. no resp drive 3. increased secretions 4. vomiting 5. hemodynamic instability 6. facial trauma/deformity 7. agitated/combative patient

List 6 causes of LGIB in adults: (Ch. 30)

1. diverticular disease 2. angiodysplasia 3. colitis (inflammatory, infectious, ischemic) 4. anorectal sources 5. neoplasm 6. UGIB

List 6 causes of UGIB in pediatrics: (Ch. 30)

1. duodenal ulcer 2. gastric ulcer 3. esophagitis 4. gastric erosion 5. esophageal varices 6. M-W tear

List 6 indications for laparotomy in peds trauma: (Ch. 38)

1. hemodynamic instability despite aggressive resuscitation free fluid on FAST and instability 2. massive bleeding (intraperitoneal) 3. pneumoperitoneum / intraperitoneal bladder rupture / grade V renovascular injury 4. gunshot wound 5. evisceration 6. peritonitis

List 11 anatomical differences that distinguish peds spines from adult ones: (Ch. 38)

1. higher fulcrum C2-3 (higher c spine injuries) 2. larger head size- greater flexion and extension injuries 3. smaller neck muscle mass 4. increase interspinous lig. flexibility 5. flatter facet joints 6. incomplete ossification at multiple bony sites 7. anterior surfaces of the vertebral bodies are more wedge shaped 8. Epiphytes of spinous processes tips mimic fractures 9. Narrow preodontoid space 10. Pseudosubluxation of C2-3 seen on 40% of kids 8-12 yrs 11. Pre-vertebral space varies with respiration *whew*

In what 4 situations should you do a retrograde urethrogram (RUG) before foley insertion? (Ch. 47)

1. hx of urethral trauma (straddle injury) 2. scrotal or penile hematoma 3. blood at urinary meatus 4. high risk of pelvic fracture

Lost 6 causes of false positive stool guaiac tests: (Ch. 30)

1. ingestion of red meat 2. turnips 3. horseradish 4. Vit C 5. methylene blue 6. bromide preparations

What is the TRIAD for acute liver failure? (Ch. 28)

1. jaundice 2. encephalopathy 3. coagulopathy (INR > 1.5)

What are some NON life-threatening causes of headaches? (Ch. 20)

1. migraine 2. cluster headache 3. tension headache 4. post-lumbar puncture headache 5. cervical muscle strain 6. TMJ disease/dental pain

Steps in afferent pain pathway: (Ch. 3)

1. nociceptive impulse enters dorsal root ganglion 2. CROSSES over at level of input 3. ascends contralateral spinothalamic tract 4. enters thalamus 5. interpreted by cortex as PAIN!

List 6 causes of UGIB in adults: (Ch. 30)

1. peptic ulcer (gastric > duodenal) 2. gastric erosion 3. esophagogastric varices 4. M-W tear 5. esophagitis 6. gastric cancer

List 8 clinical findings suggestive of increased ICP: (Ch. 20)

1. persistent vomiting 2. altered mental status 3. HTN and bradycardia (Beck's triad) 4. Bulging fontanelles 5. Diffuse, severe headache 6. Loss of venous pulsations in the eye 7. Papilledema seen at the optic disc 8. Headache worse when lying down or in the AM

List 5 CXR findings of HF (Ch 81):

1. pleural effusions 2. cardiomegaly (enlargement of the cardiac silhouette) 3. Kerley B lines (horizontal lines in the periphery of the lower posterior lung fields) 4. upper lobe pulmonary venous congestion (bat wing appearance) 5. interstitial oedema.

Complications of placental abruption: (Ch. 37)

1. premature labour 2. stillbirth (exponential rise in fetal mortality) 3. maternal coagulopathy (DIC)

Nailbed laceration management: (Ch. 50)

1. repair with 5-0 or 6-0 absorbable suture once remaining nail has been removed 2. replace nail and secure it with tape or suture into nailbed to splint open 3. Tetanus prophylaxis 4. New nail will grow back 70-160 days later

Indications for intubation in a neonate: (Ch. 11)

1. respiratory distress 2. increased work of breathing 3. desaturations 4. tachypnea 5. surfactant administration via ETT

Describe 5 techniques for removal of FB from ear (Ch 60)

1. right-angle probe 2. ballooned catheter (Swanz is my favorite) 3. suction catheter (small) 4. alligator forceps / bayonet forceps 5. Irrigation

Clinical triad for cauda equina syndrome: (Ch. 13)

1. saddle paresthesia 2. urinary retention/fecal incontinence 3. focal neurological deficit (motor or sensation)

List 5 commonly missed traumatic injuries: (Ch. 36)

1. scalp lacerations 2. extremity fractures 3. urethral injuries 4. posterior injuries 5. Injuries in axilla, groin, or buttocks

Describe how you would reduce a jaw? (Ch. 42)

1. sedation and analgesia to relax mastication muscles 2. place thumbs in buccal sulcus bilaterally, and provide downward traction on the mandible while rotating the chin upwards and backwards

Dose and indication for core peds resuscitation med: *Procainamide* (Ch. 10)

15 mg/kg ; for stable wide complex tachy

How long should you treat an acute gout attack with NSAIDS? (Ch 116)

24 hrs post symptoms relief (usually 2-6 days)

What is the prevalence of mental illness in sexual assault patients? (Ch 67)

25%

Dose and indication for core peds resuscitation med: *Magnesium sulfate* (Ch. 10)

25-50 mg/kg ; for Torsades or refractory VT/VF

Define "flail chest": (Ch. 45)

3 or more ribs, fractured at 2 points leading to PARADOXICAL chest wall motion **one of the MOST SERIOUS chest wall injuries (8-36% mortality)**

General prevalance of epilepsy: (Ch. 18)

3%

Toxic dose of lidocaine: (Ch. 4)

3-4 mg/kg 7 mg/kg WITH epi

Technique for CPR in neonates: (Ch. 11)

3:1 compressions to ventilations rate 90:30/min hands encircle thorax compress 1/3 AP depth reassess every 30-60 sec

Describe the findings in CN III, IV, and VI palsies + list 2 causes for each. (Ch. 71)

3rd nerve palsy ptosis, unable to turn eye inward/upward Mydriasis (II into brain, III out to eye) Non-ischemic optic neuropathy (due to diabetes); orbital apex mass/tumour 4th nerve palsy diplopia worse with downgaze head tilt the opposite way trauma or vascular disease (aneurysm) 6th nerve palsy MOST common esotropia worse with lateral gaze turning of head towards paretic side Wernicke's, aneurysms, neoplasm, vascular disease, trauma, MS, meningitis, thyroid disease, increased ICP

Describe the management of severe third trimester bleeding and post-partum hemorrhage? (Ch. 34)

3rd. trimester bleeds: placental abruption and placenta previa/increta = C-section ASAP Post-partum: -Early: uterine atony (IV oxytocin, massage, blood), uterine trauma -Late: retained products, infection, atony = operation, antibiotics, massage

Lactate above what makes us worried? (Ch. 6)

4

What is the management approach for the different types of mesenteric ischemia? (Ch 92)

4 cardinal goals: 1) Restore mesenteric blood flow 2) Treat underlying condition 3) Treat underlying persistent vasospasm (if present) 4) Reduce further clot propagation

What are 4 areas of narrowing in the esophagus that FBs get stuck? (Ch 89)

4 natural areas of narrowing: 1. Cricopharyngeus (upper esophageal sphincter) → #1 spot things get stuck In kids things get stuck at the cricopharyngeus; adults @ UES 2. Aortic arch 3. Left mainstem bronchus 4. Lower esophageal sphincter (LES) at the diaphragmatic hiatus

Describe the metabolism of acetopminophen and what changes in overdose(Ch. 148)

4 pathways - Glucuronidation - Sulfation - CYP2E1 - Direct renal clearence - N- acetyl -P aminophenol(Tylenol) gets oxidized via CYP2E1 to NAPQI --> Toxic! NAPQI combines with glutathione and gets excreted via the kidneys With ingestion this system gets overrun - Glucuronidation and sulfation becomes saturated, glutathione deplets and NAPQI binds directly to hepatocytes = Hepatocellular death

Approximately what percentage of patients with lupus have antiphospholipid syndrome? (Ch 118)

40%

List 6 risk factors for cholelithiasis (Ch 90)

5 Fs: fair: more prevalent in the Caucasian population 1 fat: BMI >30 female fertile: one or more children forty: age ≥40 Also - rapid weight loss - cystic fibrosis - drugs

Dose and indication for core peds resuscitation med: *Amiodarone* (Ch. 10)

5 mg/kg ; for stable wide complex tachy, persistant pulseless VT/VF

What is the 5 year survival rate for pancreatic cancer? (Ch 91)

5% at 5 years from the time of diagnosis - with high risk of metastasis to the liver and lungs

Between what BPs can the brain autoregulate to effectively preserve cerebral blood flow? (Ch. 8)

50-150 mmHg

Explain the difference between migraine without aura (common migraine) and migraine with aura (classic migraine)

80% of migraines are without an aura. WITHOUT AURA: P - pulsitile O - gradual onset U - unilateral location N - nausea/vomiting (or photo/phonophobia) D - disabling (can't do routine activities) D - duration (4-72 hours) WITH AURA: - recurrent attacks lasting minutes - unilateral fully reversible visual/sensory/ or other CNS symptoms that develop gradually - and are usually followed by headache with migraine symptoms Common auras: - scintillating scotomas (bright rim around visual loss) - sensory - speech/language - motor -brain stem - retinal Diagnostic of MIGRAINES WITH AURA: - at last one aura symptom gradually over 5 minutes or two more auras in succession - Each aura symptom lasts 5-60 minutes - At least one aura symptom is unilateral and followed/accompanied with a headache within 60 minutes - Not a better diagnosis and not a TIA

Describe bladder calculi and its management (Ch 99)

<5% of calculi Usually in older men with infected residual bladder urine with urea splitting organisms (Struvite stones!) Causes: Indwelling catheter BPH Neurogenic bladder Irradiation Schistosomiasis Urinary symptoms, with sudden obstruction of the bladder outlet Usually need surgical removal after U/S or CT dx

What are the age cutoffs for observation vs. treatment in the uncertains diagnosis of pediatric AOM? (CH. 72)

<6mo: Treat 6mo-2 year: May consider observation if non severe illness >2 year: May consider observation even if severe illness

Threshold of fetal viability? How can you guestimate? (Ch. 37)

>24 weeks or >500g estimated by uterine fundus being ABOVE umbilicus

At what bilirubin concentration is jaundice first detectable? (Ch. 28)

>25 mg/L (in tissues with high albumin concentration --> eyes, skin)

3 Categories of blood antigens (Ch. 7)

A B AB

What is your toxicologic differential for substances causing wide AGMA(Ch. 147)

A CAT PILES MUD - Alcoholic ketoacidosis - Cyanide - CO - Colchicine - Acetominophen - Toluene - Paraldehyde - Iron, isoniazid, ibuprofen - Lactic acidosis - Ethylene glycol - Salicylates - Methanol, metformin - Uremia - DKA

What is a kerion? How is it treated? (Ch 120)

A kerion is a fungal infection affecting hair follicles that is characterized by intense inflammation, and a boggy, erythematous mass, typically affecting the scalp. May contain frank pus. Treatment: Systemic antifungals, 6 to 8 weeks. If bacterial superinfection exists, an antibiotic is added. Surgical drainage of kerions is not helpful and should be avoided.

Define Cardiac Index, and provide a normal range (Ch 81):

A measurement that relates cardiac output from the LV to the body surface area. Normal 2.5-4 L/min/m2 (Determined by: CRAP - Contractility, Rate - heart-rate, Afterload, Preload).

What is the oculo-cephalic (doll's eyes) reflex? (Ch. 16)

A reflex that, when normal, makes a brainstem lesion unlikely: 1. Observe the eyes with moving head side-to-side 2. If eyes remain frozen in their orbits (gaze in direction the head is facing) this is ABNORMAL 3. If eyes maintain forward gaze despite head rotation, this is a positive doll's eyes signs and is considered a NORMAL response

Define Seasonal Affective Disorder & it's treatment (Ch 111)

A specifier of MDD: "This specifier can only be used with a recurrent major depressive disorder. The criteria for this include the following: a regular temporal relationship between onset of depressive episode and a particular time of year, full remissions at a specific time of year, two depressive episodes within 2 years that demonstrate a temporal relationship, no nonseasonal episodes within the same period, and substantially more seasonal depressive episodes than nonseasonal episodes over the person's lifetime. Treatment: phototherapy

What is Waddell's triad? (Ch. 46)

A triad of injuries seen in pediatric ped-struck traumas: -femur injury -intra-abdominal or intrathoracic injury -head injury

Describe the acute management of SBO (Ch 92)

A) Hemodynamically stable If yes, go on If no, then fluid resuscitation with balanced crystalloid solution Don't forget to replace electrolytes B) Early surgical consultation if bowel strangulation / closed loop / surgical abdomen secondary to perforation C) NG placement: only if N/V uncontrolled with modern day antiemetics or ALOC with risk of aspiration (make sure you decompress that hostile abdomen if you need to intubate!!!) Dogmalysis! D) Antibiotics in the sick, hypotensive or rigid abdomen (think gram negative and anaerobic coverage: eg. 2nd gen cephalosporin or carbapenem) E) NPO for complete bowel obstruction, fluids only for partial F) Pain control

Approach to altered LOC: (Ch. 16)

A- Alcohol, anaphylaxis E- Epilepsy, endocrine (myxedema), electrolyte, encephalitis I- Infection (neurologic, systemic) O- Oxygen low, opioids, overdose U- Uremia T- Toxins, trauma I- Insulin (neuroglycopenia) P- Psychogenic S- Structural, space-occupying lesion, SAH, stroke, shock

Features of APGAR score: (Ch. 11)

A- activity (muscle tone) P- pulse G- grimace (reflex irritability) A- appearance (colour) R- respitation 0, 1, or 2 for each.

ASA classification system: (Ch. 4)

ASA I- healthy patient ASA II- mild systemic disease- no limitation ASA III- severe syst. disease- definite limitation ASA IV- severe syst. disease- constant threat to life ASA V- moribund patient not expected to survive without surgery

What are the Grey-Turner and Cullen's signs? (Ch. 46)

A. Cullen's sign B. Grey-Turner sign (think: 'turn' pt to the side to see 'Turner' sign) Both represent retroperitoneal bleed >12hrs old

How is aspirin eliminated from the body?(Ch. 149)

ASA can reversibly bind to albumin, but free salicylate is eliminated. At therapeutic concerntrations elimination follows first order kinetics, but once it is greater then 30mg/dl elmination follows zero-order The metabolic pathways become saturated, and the pH-sensitive urinary excretion of salicylic acid determines the apparent e half-life, prolonging significantly (up to 15 to 30 hours) with large overdoses.

Describe basic burn dressing management. (Ch. 63)

ABC!!!! Analgesia Tetanus Leave blisters intact, debride ruptured blisters Dressings for partial-thickness burns Clearly infected, purulent wounds should be managed in an 'open' manner Topical antimicrobials (e.g. neomycin, mupirocin, silver sulfadiazine) Nonadherent dressing Daily dressing exchange and gentle cleansing with water and soap The second method for burn management is with occlusive dressings - Support a moist wound-healing environment - Less pain as dressings are not exchanged daily - Most appropriate for superficial partial-thickness burns with no signs of infection Dressing such as Mepilex or a nano-crystaline silver-containing occlusive dressing should be applied and left in place for approximately 1 week ***Avoid silver sulfadiazine if transferring patient to a burn center as it can alter the appearance of the burn

Management approach for acute chest pain or dyspnea: (Ch. 26)

ABCs Monitor Oxygen Vitals IV access Equipment Labs, ECG, CXR, H&P

You need to intubate a patient with severe AECOPD. What vent settings would you use?

AC Ventiliation (give their lungs a break, let the machine work. Need a volume control because pressure controls will not work in an AECOPD patient...pressures in lungs may be too high already) FiO2 100% TV 6-8cc/kg (lung protective strategy) RR 8-10 bpm I:E 1:4 or 1:5 Flow rate 80-100L/min PEEP should be low... need their lungs to be able to expire! so 2-5 Watch the peak flow pressures Permissive hypercapnia is the name of the game

Describe neurologic findings you might expect with major artery ischemic strokes(ie ACA, MCA and vertebrobasilar) (CH. 101)

ACA - Judgement and insight - Grasp and suck reflexes may be present - Bowel/bladder incontinense - Altered mentation - Paralysis and hypesthesia on the lower limb opposite the side of the lesion(Leg>arm) - Apraxia or clumsiness of gait MCA - Marked motor and sensory disturbances contralateral. arm>leg Vertebrobasilar - CN deficits - Cerebellar involvement - Neurosensory tracts - Decreased LOC - Nausea/ vomiting - Vertigo, syncope, diplopia, visual field defects, weakness, paralysis, dysarthria, dysphagia, spasticity, ataxia, or nystagmus may be associated with vertebrobasilar artery insufficiency. - May have crossed deficits

Main causes of *cardiogenic shock*: (Ch. 6)

ACS Cardiomyopathy/myocarditis Cardiac dysrhythmias Negative inotropic drug ingestion Structural heart damage (papillary muscle rupture, flail mitral valve, septal wall rupture)

List DDx for ↑ Troponin (Ch 78)

ACSss Memory Trick: 1. Acute coronary ischemia (ACS, Cocaine, Variant angina, Coronary embolism/vasculitis) 2. Comorbidities causing myocardial injury (Renal failure, sepsis, ARDS, stroke, SAH) 3. Systemic shock states Distributive shock states (sepsis, CO poisoning, burns) and Cardiogenic shock states (myocarditis / myocardial contusion / cardiomyopathy)

Define acute, persistent, and chronic diarrhea: (Ch. 31)

ACUTE: 0-14 days - usually infectious (bacterial or viral) PERSISTENT: 14+ days - usually bacterial or protozoan CHRONIC: 30+ days - usually non-infectious

When do you admit someone with an LGIB? (Ch. 30)

ADMIT if not obviously hemorrhoids, fissure, or proctitis

List four common sites for hypertensive intracranial hemorrhage and six symptoms associated with hypertensive ICH(Ch. 101)

AFFECTED AREA (FREQUENCY) Putamen (44%) Thalamus (13%) Cerebellum (9%) Pons (9%) Other cortical areas (25%) COMMON CLINICAL PRESENTATION Contralateral motor/sensory loss Limb pain, speech difficulty Uncoordinated movements of trunk and limbs Numbness, weakness, ataxia, dizziness Numbness, weakness, language disturbances

What is cancrum oris? (Ch. 70)

ANUG with erosion through teeth and extension into lips and buccol mucosa

General approach to the multi-trauma patient? (Ch. 36)

ATLS approach: Airway Breathing Circulation: control deadly bleeding Disability (GCS/Pupils/Gross Motor) Exposure ----- Secondary survey ; AMPLE hx ; Investigate ------ TIPS: Prepare your team! Control deadly bleeding - TQs, bind pelvis, scalp lacs, Decompress the chest Bind the pelvis

Describe the physiology of acute urinary retention (AUR) (Ch 99)

AUR results from a disruption of this coordinated physiology caused by an increased resistance to flow via mechanical (eg, urethral stricture, clot retention) or dynamic means (eg, increased α-adrenergic activity, prostatic inflammation) or decreased neurogenic control of the detrusor muscle (eg, drugs inhibiting bladder contractility).

What condition can complicate scaphoid fractures that go undetected? (Ch. 51)

AVN The more proximal the scaphoid fracture the greater the risk of AVN

What are signs a fetus is in distress (in traumatic injury of mom)? (Ch. 37)

Abnormal fetal HR (not 120-160) Decreased HR variability Late decelerations (fetal hypoxia) Decreased fetal movement

Name 10 characteristics of a panic attack (Ch 112)

Abrupt surge of intense fear or discomfort that reaches a peak within minutes, in which four or more of the following occur: Palpitations Sweating Trembling Shortness of breath or feeling of being smothered Feeling of choking Chest pain or discomfort Nausea or abdominal distress Feeling dizzy or light-headed Chills or heat sensations Paresthesias Derealization or depersonalization Fear of losing control or going "crazy" Fear of dying

4 contraindications to Ketamine: (Ch. 4)

Absolute -laryngospasm concerns (infants <3 mo) Relative -psychosis -globe rupture (risk of incr IOP) -significant CAD (catecholamine surge)

List 1 absolute contraindication and 4 relative contraindications to DPL: (Ch. 46)

Absolute CI: -Need for laparotomy Relative CI: -Prior abdominal sx or infections -Coagulopathy -Obesity -2nd or 3rd trimester of pregnancy

Red flags that require hospital admission in syncope: (Ch. 15)

Absolute: 1)chest pain 2)unexplained SOB 3)hx of CHF or valve heart disease 4)pt with ECG changes

Describe the approach and management of hemodynamically unstable pelvic fracture (Ch 55):

Access -> stabilize -> control hemorrhage -> resuscitate with blood -> definitive treatment Ensure excellent IV or central access above the pelvis Consider early activation of massive transfusion protocol Typical to require 10 - 20 units of prbcs in first 24 hours Bind/stabilize pelvis with sheet or pelvis binder Consider requesting orthopedic external fixation of pelvis

Define what is meant by an "acute exacerbation" of COPD (AECOPD)

According to GOLD: - change in the patient's baseline dyspnea, cough, and/or sputum that is beyond normal day-to-day variations. - acute in onset - may warrant a change in the regular medications that the patient takes

Describe the difference between Alkali and Acid injuries(Ch. 64)

Acidic - Protein denaturation and coagulative necrosis - The necrosis forms an eschar which limits the depth of acid penetration - Free hydrogen ions are easily neutralized on the skin by copius water irrigation Alkali - Saponification and liquefactive necrosis of body fat - Produces soluble protein complexes which permit the passage of hydroxyl ions deep into the tissue - No eschar --> penetrates deeper into the tissue

What should be on your differential for folliculitis? (Ch 120)

Acne, keratosis pilaris, fungal infection, hidradenitis suppurativa

Types of immune mediated complications of transfusion: (Ch. 7)

Acute -Intravascular hemolytic transfusion reaction -Transfusion related acute lung injury (TRALI) -Allergic reaction (urticaria to anaphylaxis) -Febrile transfusion reaction Delayed -EXTRAvascular hemolytic transfusion reaction -Transfusion associated graft vs. host disease

Types of non-immune mediated complications of transfusions: (Ch. 7)

Acute -Transfusion associated circulatory overload (TACO) -Bacterial contamination Delayed -Transmitted viruses (Hep B, C, HIV, CMV)

Define acute, chronic and recurrent otitis media. (CH. 72)

Acute - signs and symptoms of an infection with evidence of effusion Note: also called acute suppurative or purulent otitis media' Chronic - Also called chronic suppurative OM, refers to chronic discharge from perforated tympanic membrane Recurrent - >3 episodes of AOM over 6 months or >4 in one year

List 6 causes of gastritis (Ch 89)

Acute Helicobacter pylori infection Acute phlegmonous gastritis (rare, but lifethreatening!) Shock states Lymphoma Adenocarcinoma NSAID use ETOH abuse / use Drugs: 5FU, bisphosphonates, iron Smoking Bile / pancreatic secretions Stress Glucocorticoids

List 6 critical causes of chest pain: (Ch. 26)

Acute MI Acute cronary ischemia Aortic dissection Pulmonary embolism Tension pneumothorax Cardiac tamponade Esophageal rupture

"Critical" NON-infectious causes of fever: (Ch. 12)

Acute MI PE Intracranial hemorrhage CVA NMS Thyroid storm Addisonian crisis (acute adrenal insufficiency) Transfusion reaction Pulmonary edema

List 10 pharmacologic agents that may cause acute psychosis (Ch 110)

Acute Psychosis mnemonic: Antihistamines Cocaine, cannabis, corticosteroids Under the table hallucinogens Tranquilizers!!! (diazepam, clonazepam) ETOH Propranolol Sedatives phenYtoin Captopril Heavy metals Opioids SteroIdS

List common misdiagnoses in patients with ruptured AAA (Ch 86):

Acute abdomen: pancreatitis, diverticulitis, cholecystitis, appendicitis, bowel obstruction Renal colic, intestinal ischemia, MSK back pain perforated viscus, Acute MI

Describe the diagnosis of prostatitis (Ch 99)

Acute bacterial prostatitis: clinical diagnosis UTI symptoms, fevers, chills, dysuria, frequency, urgency, perineal or low back pain Tender and swollen prostate on rectal exam Urine culture, blood cultures if febrile, and STI testing At risk for urinary retention and prostatic abscess

Describe the pathophysiology (in one-to-two sentences) of AMI (Ch 78):

Acute coronary artery thrombosis due to a sudden intraluminal activation of platelet-thrombotic factors leading to myocardial necrosis.

How does optic neuritis present? What are some findings on physical exam? What is the treatment? (Ch. 71)

Acute monocular vision loss over several hours to days. Ocular PAIN with eye movement Signs: RAPD, swollen optic disc Treatment: IV or PO steroids to improve vision

What is the vision losing risk of not treating opthalmic herpes zoster infection?

Acute necrotizing retinitis

List 10 reversible causes of dementia(Ch. 104)

Adverse drug reactions - Psychotropic drugs - Antihypertensive - ANticonvulsants - Anticholinergics - L-dopa - Heavy metal poisining - Carbon monoxide - Trichlorethylena - ETOH Endocrinopathies - Hypothyroid - Parathyroid - Addisons - Cushings - Panhypopituitarism - Thiamine deficiency - Niacin deficiency - Vit B12 - Folate def Intracranial process - Tumor - Hydrocephalus - Chronic head trauma - HIV - Tuberculous or fungal meningitis - Toxoplasmosis, cryptococcus, CMV, herpes, VZV Depression

List 5 pathogens responsible for infection from a dog bite(Ch. 54)

Aerobic bugs: Staphylococcus aureus, alpha- hemolytic and beta-hemolytic streptococci, Klebsiella, Bacillus subtilis, Pseudomonas, Enterobacteriaceae, and Capnocytophaga canimorsus Anaerobic bugs: Bacteroides, Fusobacterium, Peptostreptococcus, Porphyromonas, and Prevotella species

List conditions associated with rectal prolapse (Ch 96)

Affects the very young and very old 1. Elderly women with uterine or urethral prolapse 2. Young children with chronic constipation or diarrhea 3. May also occur with CF, parasitic infections, malnutrition Reduction: Apply dextrose/sucrose soaked gauze x 5-10 mins, then gentle pressure to coax the rectum back into the body Prevention: Increase dietary fibre Increase fluid intake

List 6 causes of esophageal dysphagia. "Delayed / gets stuck" (Ch 89)

Again think M&M Mechanical Intrinsic issue: strictures, webs, rings, tumours, EsophagitIs, ***foreign bodies*** Extrinsic compression: osteopHytes, ***mediastinal masses***, aortic aneurysm, thyroid goitre Motility Intrinsic: Achalasia, diffuse esophageal spasm, hypertensive LES, scleroderma, CREST syndrome, nutcracker esophagus Extrinsic: Gastric volvulus Alcoholism, diabetes, GERD,

List 5 indications for an Xray in lower back pain (Ch 54):

Age < 18, > 50 yrs. Hx of malignancy (of any type!) or B-symptoms (weight loss, fevers, night sweats) Hx of fever, IVDU, immunocompromised Recent trauma Progressive neurologic deficits or cauda equina syndrome Duration > 4-6 weeks

List at least 4 risk factors for anaphylaxis (Ch 119)

Age and sex Pregnant women, infants, teenagers, elderly Route of administration Parenteral > oral Higher social economic status Time of the year Summer and fall (the outdoor seasons) History of atopy Emotional stress Acute infection Physical exertion History of mastocytosis

List 4 risk factors for overwhelming sepsis from a dog bite(Ch. 54)

Alcoholic liver disease Fungating or surgical asplenia Lung disease Corticosteroid use

List some vestibulotoxic drugs: (Ch. 19)

Aminoglycosides Anticonvulsants Alcohols Quinines Quinidine Minocyclines

What first line antibiotic is a good choice for cat and dog bites?(Ch. 54)

Amox-Clav 875mg PO BID For penicillin allergies: TMP-SMX Moxifloxacin

Describe your approach to Acute Necrotising Ulcerative Gingivitis (Ch. 70)

Analgesia In order to start brushing and eating again, need to tx pain. Systemic Tylenol / NSAIDS combined with topical viscous lidocaine Antibiotics Penicillin or tetracycline (avoid in children with primary teeth due to staining) Improved Oral hygiene & Rinses Regular brushing, warm saline rinses, and may consider dilute 3% hydrogen peroxide (avoid in patients who cannot avoid swallowing) Referral to dentist or periodontist They will start feeling better, but still need further specialist care. Make sure they follow-up with a dentist, and that you document those instructions clearly

Compare Aneurysm and Pseudoaneurysm (Ch 86)

Aneurysm: - Occur due to trans-mural weakening: loss of collagen/elastin/arterial muscle integrity - A localized dilatation of the aorta, involving all three layers of the artery (IMA). Pseudoaneurysm: - Arise from a defect in the arterial wall - Arise from a leaking anastomosis after AAA repair. - A collection of flowing blood that communicates with the true lumen, but is enclosed with (contained by) just the adventitia or surrounding soft tissue

What is the oculo-vestibular (cold caloric) reflex? (Ch. 16)

Another test to assess the viability of the brainstem in the unconscious pt: 1. Elevate pts head to 30 degrees 2. Instill 10-30mL of ice cold water into one ear 3. Observe eye movements 4. *If brainstem is intact*, cold water in the right ear will result in SLOW conjugate deviation towards the right (cold ear) for 30-120 seconds with FAST nystagmus beats to the left (normal ear) 5. *If brainstem NOT intact*, there will be NO eye movement with irrigation.

Describe your approach to tx epistaxis including both anterior and posterior bleeds(Ch. 72)

Anterior Usually Litte's area or kesselbach's plexus -SIlver nitrate (bad with active bleeders) -cautery (don't use longer than 10-15 seconds can perforate) start peripheral to central -bilateral septal cautery not advised -can use gelfoam or surgicel -rhino rockets / rapid rhino or epistats (leave in for 48-72hrs) -traditionally patients with packing are placed on antibiotics to prevent risk of toxic shock syndrome, this practice is controversial Posterior bleeds - large bleed despite adequate anterior packing -commercially available (epistat) or the good old foley -REMEBER NEED TO APPLY TRACTION -Pack both nares with foley, blow up balloon partially with sterile water (saline can crystallize and cause deflation issues later), tug until well seated, apply traction (I use the green umbilical clamps). Can add a little more fluid to balloon. -Leave packs in for 2-5days as per ENT. Antibiotics here are important (Keflex or Amox-Clav) -Refractory cases may need CT and IR

What is the usual direction of jaw dislocation? (Ch. 70)

Anterior dislocation

What is the most common site of compartment syndrome? (Ch. 49)

Anterior lower leg compartment

What does the anterior drawer (of the ankle) test for? (Ch 58)

Anterior talofibular ligament (ATFL)

What does the inversion stress-test (also known as the talar tilt) test for? (Ch 58)

Anterior talofibular ligament (ATFL) and the Calcaneofibular ligament (CFL)

Describe the "trangles" approach to neck anatomy: (Ch. 44)

Anterior triangle: -Contains neurovascular and aerodigestive tract structures Posterior triangle: -Fewer vital structures

What is the treatment of herpes zoster? (Ch 120)

Antiviral medications are indicated, especially within <72 hours of onset of rash, to decrease the duration of symptoms and associated pain. Antiviral therapy may be initiated with acyclovir, famciclovir, or valacyclovir. [Uptodate recommends continuing administration of antiviral therapy as long as new lesions are appearing]. Doses to know: Valacyclovir: 1000 mg three times daily for seven days Famciclovir: 500 mg three times daily for seven days Acyclovir: 800 mg five times daily for seven days (this is the GO-TO drug for PREGNANT women with herpes zoster)

How do you treat an opthalmic herpes zoster infection?(Ch. 71)

Antivirals: Acyclovir/valacyclovir/familovir, within 72 hours. Po just as good as IV Topical steroids Topical antibiotics Analgesia Optho follow up

What is Anton's syndrome? (Ch. 71)

Anton's syndrome is characterized by bilateral blindness, normal pupillary reflexes, and, interestingly, denial of blindness. It is this denial of blindness that may be incorrectly assumed to be evidence for a functional process. Diagnosis: Bilateral occipital lobe lesions

Nexus Rule for C-Spine: (Ch. 43)

Any stable trauma patient with at least one of the following MUST have imaging of C-spine: "PAID Now" Posterior midline tenderness Altered mental status Intoxication Distracting injury Neurological deficits **notice no consideration of mechanism in Nexus

List 8 Ddx for Thoracic Back Pain (Ch 54):

Aortic dissection Pneumonia Myocardial infarction PE Ruptured esophagus Pancreatitis Thoracic disc herniation Usually not diagnosed until 20 months after the first clinical presentation! Tumour / hematoma with nerve impingement Disk infection Pyelonephritis

What is the classic presentation of ascending cholangitis? (Think of a triad & pentad) (Ch 90)

Ascending Cholangitis usually occurs after sustained blockage of the common bile duct (gallstone, malignancy, stricture). Bacteria then propagate (from the duodenum) and invade the surrounding tissues. Usually E coli, klebsiella, enterococcus, bacteroides. Think: Charcot's Triad: RUQ pain, fever, jaundice (also applies to hepatitis and cholecystitis!) Reynold's Pentad: Charcot's Triad + hypotension and altered mental status

Arousal and cortical activation controlled by what? (Ch. 16)

Ascending reticular activating system (ARAS) - found in the dorsal brainstem

How do you access an AV fistula? (Ch 87)

Ask for help! (read online/call a friend!) If you are the most qualified person to do it: Try to choose a different peripheral venipuncture site if possible! Maintain complete sterility Clean hands Complete sterile attire Good skin prep. Sterile gauze and supplies **NO tourniquet allowed** Access the venous side of the fistula for venipuncture Apply gentle sterile pressure for 5 mins post-puncture (don't fully occlude the fistula)

Describe an approach to reading C-Spine X-rays: (Ch. 43)

Assess the A-B-C-D-Es *A*dequacy of film - all 7 cervicl vert. + top of T1 *A*lignment - ant vert, ant spinal, post spinal, spin proc line *B*ones - vert. bodies, facets, spinouc processes *C*artilage - intervertebral space narrowing/widening *D*ens - dens itself, predental space *E*xtraaxial soft tissue - 6/2 rule (6mm at C2, 22mm at C6), distance btw spin proc's

List 8 risk factors for death from asthma

Asthma hx - Previous severe exacerbation - Two or more hospitalizations for asthma in the past year - Three or more ED visits for asthma in the past year - Hospitilizations for an ED visit for asthma in the past month - Use of more then two MDI short-acting beta 2 agonist canisters per month - Current use of or recent withdrawl from systemic steroid - Difficulty perceiving asthma symptoms or severity of exacerbations Social hx - Low SES - Psychosocial problems - Illicit drug use, especiall cocaine or heroin inhaled Comorbidities - CV disease - Chronic lung disease - Chronic psychiatric disease

How are upper limb DVTs managed? (Ch 88)

At similar risk of PE, so is treated like a lower extremity DVT May or may not have to remove the catheter (depending on why it's there) Because it can cause a PE - anyone with a u.e. DVT proximal to the elbow require definitive treatment Optimal dosing and duration is debated Usually at least 3 months of anticoagulation (do your risk analysis for everyone though!) Infusion phlebitis ) isolated brachial vein thrombosis - post recent IV infusion may be treated like a superficial thrombophlebitis of the lower leg, but good evidence is lacking. Remember, that we not only treat DVT's to prevent PE's, but ALSO because DVT's damage the valves of veins - which can lead to venous insufficiency (see prev. chapter). This can become a chronic disabling disease! Pain, varicosities, ulcers, skin changes, swelling - all with high morbidity!

What increased risk of cervical injury do Down's Syndrome and Rheumatoid arthritis patients both have? (Ch. 43)

Atlantoaxial instability leading to subluxation or dislocation

List 6 causes of urinary retention in women (Ch 99)

Atonic bladder Inflammation post-partum or herpes simplex virus Bartholin's abscess Acute urethritis or Vulvovaginitis Pelvic masses Pelvic organ prolapse (bladder, rectum, anus)

List at least 5 causes of AVN (traumatic and atraumatic) (Ch. 56):

Atraumatic: Chronic corticosteroid use Chronic ETOH abuse Sickle cell anemia Dysbarism Chronic pancreatitis HIV Traumatic: Post-hip dislocation 5% if reduced in 6 hrs, 50% if reduced in 12 hrs Post-femoral neck fracture Athlete with an energy deficit or overtraining

Describe your management for post root canal pain? (Ch. 70)

Attempt nerve block, if no improvement may need to open root canal to relieve buildup of gas / pressure Adjust occlusion Refer to Dentist / Surgeon who preformed the procedure

List 4 features of physician behaviours that encourage disclosure of intimate partner violence (Ch 68)

Attentive listening Conveyance of compassion and concern Nonjudgmental Respect patient's right to autonomy in decision making BONUS: Educational material around ED Note: Gender of DR does not appear to be factor in disclosure.

What is the minimal volume of intraperitoneal blood detectable on bedside U/S? (Ch. 46)

Average of 500ml

List at least 3 of the 5 radiographic cues to posterior arch fractures (Ch 55):

Avulsion of L5 transverse process Avulsion of ischial spine Avulsion of lower lateral lip of the sacrum Displacement at the site of a pubic ramus fracture Asymmetry or lack of definition of bony cortex at the superior aspect of the sacral foramina

NON-neurologic causes of muscle weakness? (Ch. 13)

Dehydration Electrolyte imbalance Anemia Myocardial ischemia Vasodilatory shock Infection

List three diagnostic steps to be performed in the ER for stroke(Ch. 101)

Blood glucose test (+ PTT, INR, CBC, ) ECG To exclude A. Fib or acute MI Cranial imaging

Universal recipient is what blood type? (Ch. 7)

Blood type AB+

Universal donor is what blood type? (Ch. 7)

Blood type O- (no antigens on their RBCs surface)

List 6 causes of bloody diarrhea (Ch 94)

Bacterial causes: "Clotty salty excrement screws your vitals" (episode 31) Campylobacter, Salmonella, EPEC, Shigella, Yersinia, Vibrio

What is the four categories of pelvic fractures according to the Young-Burgess classification (Ch 55):

Based on mechanism of injury of pelvic fractures: Answer: APC, LC, VS, CM (Details: Anterior-Posterior Compression (APC) APC I: symphysis < 2.5cm (stable) APC II: symphysis > 2.5cm, SI disruption (partially stable) APC III: symphysis > 2.5cm, SI shearing (completely unstable) Lateral Compression (LC) LC I: ipsilateral sacral crush injury (stable) LC II: ipsilateral sacral crush injury, disruption of posterior SI ligaments, possibly iliac wing fracture (partially unstable) LC III: internal rotation of ipsilateral hemipelvis with external rotation of contraleral hemipelvis ('windswept pelvis') Vertical Shear (VS) Vertical displacement of sympysis and SI joints ALWAYS UNSTABLE Combined Mechanism (CM) ALWAYS UNSTABLE)

Clinical TRIAD for cardiac tamponade: (Ch. 45)

Beck's Triad: 1. JVD 2. Muffled heart sounds 3. hypotension

List 6 diseases that may require prophylaxis in child sexual abuse and the drugs with dosage that you would use above and below 45kg? (Ch 66)

Below 45kg 1. Gonorrhea: Ceftriaxone 125mg IM, Cefixime 8mg/kg *One dose 2. Chlymadia: Erythromycin base 50mg/kg/day QID x 14 days 3. Trichomoniasis: Metronidazole 15mg/kg/day PO divided TID x 7 days 4. Syphilis: Benzathine penicillin 50,000 units/kg IM One dose max 2.4 million units 5. HSV: Acyclovir 80mg/kg/day divided tid x 7-10 days 6. Heptatitis B HBIG 0.06mL/kg IM, vaccine 7. HIV: Contact local infectious disease Above 45kg 1. Gonorrhea: Ceftriaxone 250mg IM, or cefixime 400mg po *One dose 2. Chlamydia: If older then 8: Azithromycin 1g po once or doxycycline 100mg po bid x 7 days 3. Bacterial vaginosis: metronidazole 500mg po bid, metronidazole 0.75% 5g intravaginally daily x 5 days or clindamycin cream 2% 5g intravaginally qhs x 7 days 4. Trichomoniasis: Metronidazole 2g po * one dose 5. Syphilis: Benzathine penicillin 2.4 million units *one dose 6. HSV: Acyclovir 400mg po tid x 7-10 days, valacyclovir 1g po bid x 7-10 days 7. Hepatitis b: Same as above 8. HIV: Same as above

List 2 advantages and disadvantages each for CT and US in the diagnosis of appendicitis. (Ch 93)

Benefits: 1. Decreased cost relative to other imaging modalities 2. Lack of ionizing radiation exposure, 3. decreased time to diagnosis. 4. May help us look for other etiologies in young women (pelvic ultrasound for ovarian masses) Limitations of US: 1. decreased specificity and increased pain due to the transducer pressure needed for the graded compression 2. a number of US examinations cannot visualize the appendix (ie, nondiagnostic) for a number of reasons, including lack of operator experience, patient factors (eg, obesity), superimposed bowel gas, or atypically located appendix.

List 8 RFs for stroke in young patients(Ch. 101)

Big categories Hypercoagulable states Vasospasm Post-infectious Traumatic Connective tissue disorders Pregnancy Use of oral contraceptives Antiphospholipid antibodies Protein C and S deficiencies Sickle cell anemia Polycythemia Migraine syndromes Recreational drugs: **cocaine Amphetamines Recent infection from varicella or fungal meningitis Carotid / vertebral trauma leading to dissection Spinal manipulation / cough / yoga / vomiting

Define Bipolar I and Bipolar II (Ch 111)

Bipolar I disorder includes at least one manic episode, and patients have typically had one or more major depressive episodes, although a depressive episode is not necessary for diagnosis. Bipolar II disorder involves a hypomanic episode and at least one major depressive episode. A hypomanic episode includes the features of a manic episode without psychosis, marked impairment of function, or the need for hospitalization.

List 3 classes of venomous marine injuries, and describe key principles of management for each?(Ch. 55)

Bites Generally from cephalopods such as octopi. The octopus has a pair of modified salivary glands that secrete venom from its beak. The blue ringed octopus in particular has reported fatalities from its venomous neuromuscular inhibitor and vasodilator. Treatment is fully supportive, as no anti-venin exists Nematocysts NEmatocysts are, in essence, spring-loaded' venom glands that passively discharge on mechanical or chemical stimulation. They are found in animals known as cnidaria, which include jelly fish, man-of-war, hydraods, box jellyfish, nettles and anemones. They are unique in that they can function even when the animal is dead or limb has been severed. The Toxin is primarily antigenic, and allergic reactions are common. Severity is related to number of nematocysts, species, and the patient's auto pharmacologic response. Death is usually from cardiopulmonary collapse, although death from drowning is more common. Management includes removal from water, treatment of allergic reaction, removing nematocysts, and pouring ideally hot vinegar over affected area. Immersion in fresh water may increase nematocyst firing and is not recommended. Stings Sea animal that sting include sea urchins, cone shells, stingrays, sea snakes, stone fish and many others. Irrespective of which creature causes the sting, although some are worse than others, namely the cone shell and stingray, care is supportive, inducing removing sting apparatus, tetanus, antibiotics (cipro is a good choice) and good cardiopulmonary care.

List common odors on physical exam in overdose(Ch. 147)

Bitter almonds: Cyanide Carrots: Cicutoxin(Water hemlock) Fruity: DKA, isopropanol Garlic: Organophosphates, arsenic, dimethyl sulfoxide, selenium Gasoline: Petroleum distillates Mothballs: Naphthalene, camphor Pears: Chloral hydrate Pungent aromatic: Ethchlorvynol Oil of wintergreen: Methylsalicylate Rotten eggs; Sulfur dioxide, hydrogen sulfide Freshly mowed hay: Phosgene

Indications for giving cryoprecipitate: (Ch. 7)

Bleeding with low fibrinogen Dysfibrinogenemia Bleeding in von Willebrand's disease that is unresponsive to DDAVP and no factor VIII

Mechanism of action of local anaethetics: (Ch. 4)

Block *sensory* > motor neuron condutction (reversible Na channel block is best in unmyelinated sensory fibres)

What is blue toe syndrome and what is its significance? (Ch 87)

Blue toe ysndrome is caused by acute arterial occlusion due to microemboli (atheroemboli) made of cholesterol, calcium, platelets, and other debris that break off from proximal aneurysms/plaques and lodge distally AKA: "trash foot" In the CNS, these phenomena lead to TIA's and Strokes In the lower extremities these form painful cyanotic areas on the feet (usually toes), Ie. Blue toes The key is to try and identify the proximal source of the atheroemboli, which may include AAA, iliac/femoral/carotid/popliteal artery atheromas, etc. Most of these patients should have good peripheral pulses, asymmetric distribution of these lesions, and no evidence of systemic vasculitis. Treatment includes identification and removal of the proximal source. Usually CT-angiography is the test of choice to identify the proximal lesion. Definitive treatment includes local endarterectomy, vascular bypass, angioplasty, etc. Medical management is with antiplatelets and anticoagulant to prevent further thrombosis.

List the three typologies of intimate partner violence perpetrators (Ch 68)

Borderline or dysphoric individual Antisocial of generally violent individual Non violent outside home with no psychopathology. Often evidence of passive dependency or OCPD

Which vessel is most commonly injured in upper and lower extremety trauma? (Ch. 48)

Brachial and femoral arteries

List the 5 most common cancers metastatic to bone (Ch 56):

Breast Kidney Lung Thyroid Prostate

Hard signs of laryngeotracheal injury: (Ch. 44)

Bubbling of any neck wound Massive subcutaneous Air Bony crepitus Clothesline mechanism of injury

What is Budd-Chiari Syndrome? (Ch 90)

Budd-Chiari Syndrome refers to hepatic vein outflow obstruction (not portal vein obstruction). Usually associated with disorders that promote excessive clotting, such as Factor V Leiden, Protein C/S deficiency, thrombophilia, antithrombin III deficiency, leukemia, Bechet's disease, OCP use, nocturnal hemoglobinuria. Can present with fulminant hepatic failure or insidious jaundice/ascites.

Describe San Francisco Syncope Rule: (Ch. 15)

C- CHF H- Hematocrit <30% E- ECG abnormality (VT, ischemia, LBBB, long QT) S- Shortness of breath S- SBP <90 *Any of above puts pts at high risk for adverse outcome ---Consider admission

What screening tool is used for delirium? how do you use it? (Ch. 104)

CAM! 1. Acute onsent and fluctuating course 2. Inattention 3. Disorganized thinking 4. Altered LOC Need 1 and 2, plus either 3 OR 4

"Emergent" NON-infectious causes of fever: (Ch. 12)

CHF Dehydration Post-ictal Sickle cell disease Transplant rejection Pancreatitis DVT

List 6 toxins that are radiopaque(CH. 147)

CHIPS CHloronated hydrocarbons, calcium salts, crack vials Heavy metals Iodinated compounds Psychotropics, packets of drugs, play-doh, potassium salts Enteric coated tablets Salicylates, sodium salts, sustained release preparations

What lab test to order if suspecting myositis? (Ch. 13)

CK

What liver diseases are associated with alcohol abuse? (Ch 90)

Chronic alcohol use progresses to: steatosis (after 2 weeks), fibrosis, cirrhosis (after 5 yrs) and then can lead to hepatocellular carcinoma.

List the name and pathologic features of the cranial nerves. (Ch. 105)

CN I: Olfactory nerve. - Unilateral Anosmia. - Trauma/ tumor CN II: Optic Nerve. - Unilateral vision loss. - Trauma/ tumor/ Inflammatory(Optic neuritis)/ Ischemic CN III: Oculomotor nerve. - Ptosis/ Eve deviated laterally and down/ Diplopia/ Dilated and non reactive pupil/ Loss of accomodation. - Trauma/ Ischemic/ Vascular/ Myasthenia gravis CN IV: Trochlear nerve. - Cannot move eye downward or lateral/ diplopia/ head tilt to overcome eye restriction. - TRAUMA! CN V: Trigeminal nerve. - Partial facial anesthesia/ episodic, lancinating facial pain associated with benign triggers. - Trauma/ Tic Douloureux CN VI: Abducens nerve. - Lateral eye movement restriction/ Diploplia with lateral gaze - Tumor, Elevated ICP CN VII: Facial Nerve. - Hemifacial paresis(LMN lesion -> whole head, UMN -> Forehead spared)/ Abnormal taste/ sensory deficit around ear/ Intolerance to loud noises. - LMN: Infection(Bell's Palsy), Lyme disease, Otitis media. UMN: Stroke, tumor CN VIII: Vestibulocochlear nerve. - Unilateral hearing loss/ tinnitus/ vertigo. - Tumor(ENT's favorite acoustic neuroma) CN IX: Glossopharyngeal nerve. - Painful paraoxsyms to throat and neck. - RARE/ brainstem lesion/ glossophayrngeal neuralgia CN X: Vagus Nerve. - Unilateral loss of palatal elevation/ relfux of lqiuid through nose/ Hoarse voice. - Brainstem lesion/ injury to recurrent laryngeal nerve during surgery CN XI: Spinal Accessory nerve. - Downward and lateral rotation of scapula and shoulder drop. - Trauma CN XII: Hypoglossal nerve. - Tongue deviations(UMN: tongue deviated towards opposite side. LMN: Tongue deviates toward lesion and atrophies). - Stroke/ tumor/ ALS/ Metastatic disease

What cranial nerves are ususally involved in diabetic mononeuropathy(Ch. 105)

CN III palsy with pupillary sparing

What is your toxicologic differential for hypothermia(Ch. 147)

COOLS - Carbon monoxide - Opioids - Oral hypoglycemics, insulin - Liquor - Sedative hypnotic

What is your toxicologic differential for miosis(Ch. 147)

COPS - Cholinergics, clonidine, carbamates - Opioids, organophosphates - Phenothiazines, pilocarpine, pontine hemorrhage - Sedative-hypnotic

How is mesenteric ischemia diagnosed? List 4 lab abnormalities expected in acute mesenteric ischemia (Ch 92)

CT abdo/pelvis with IV contrast is the most utilised, but mesenteric angiography is the GOLD STANDARD! Duplex U/S is not widely used due to inability to image past proximal vessel or rule out bowel infarction. Lab: Leukocytosis, Elevated hematocrit secondary to hemoconcentration Metabolic acidosis. Biomarkers you can consider: lactate (sens 86% spec 42%), D-dimer (sens 96% spec 40%), interleukin (IL)-6,

What is the important step for all orbital cellulitis work-ups? (Ch. 22)

CT to rule out abscesses!

Which imaging tests can be used to diagnose PE? List advantages and disadvantages of each. (Ch 88)

CT-PE protocol +'ves: finds alternate diagnoses, rapid and fast, no iodine contrast used. Highly sensitive and specific. -'ves: ionizing radiation V/Q scanning +'ves: less radiation -'ves: not as sensitive, does not find as many alternative diagnoses, must be stratified based on high-intermediate-low-normal probability. For example, someone with an intermediate probability scan still requires a CT-PE scan.

Dose and indication for core peds resuscitation med: *Calcium* (Ch. 10)

CaCl 20mg/kg ; for hypocalcemia or hyperkalemia

What are the 4 most common causes of infectious diarrhea? (Ch 94)

Campylobacter (Stool PCR) Non-typhoid Salmonella (Stool PCR) Shiga toxin-producing Escherichia coli (STEC - Sorbitol MacConkey and serotyping for O157) Shigella

List 5 regular anti convulsants that can be re-initiated in the ED(Ch. 102)

Carbamazepine: 8mg/kg po ae: drowsiness, nausea, dizziness Gabapentin: 900mg/day at 300mg TID for three days ae: Somnolence, dizziness, ataxia and fatigue Lacosamide: Oral and IV preps Lamotrigine: 6.5 mg/kg po as: nausea Levetiracetam: 1500mg oral load. IV 60mg/kg ae: fatigue and dizziness Phenytoin: 20mg/kg in max doses of 400mg every 2 hours or 18mg/kg IV at <50mg/min ae: IV hypotension, bradyarrythmia, extravasation Fosphenytoin: 20 PE/kg at a max rate of 150 PE/min, can give IM Valproate: 30mg/kg, max rate of 10mg/kg/min

Types of *cellular toxins* causing shock (and their antidotes): (Ch. 6)

Carbon monoxide (oxygen) Cyanide (hydoxocobalamin) Methemoglobin (IV methylene blue) Hydrogen sulfide (O2,supportive,sodium nitrite?)

List ten different causes of wheeze

Cardiac - Valvular disease - CHF COPD exacerbation - Pulmonary infection - Pneumonia - Allergic bronchopulmonary aspergillosis - Lofflers syndrome - Chronic eosinophilic pneumonia Upper airway obstruction - Laryngeal edema - Laryngeal neoplasm - Foreign body - Vocal cord dysfunction Endobronchial disease - Neoplasm - Foreign body - Bronchial stenosis Pulmonary embolus Carcinoid tumor Allergic rxn or anaphylaxis Miscellaneous - GERD - Non cardiogenic pulm edema - Addison's disease - Invasive worm infection

System-based approach to chest pain with some important causes other than the *critical 6*: (Ch. 26)

Cardio--cocaine-induced angina/vasospasms Pulmonary--bronchitis, pneumonia, pleuritis GI--Mallory-Weiss, cholecystitis, pancreatitis MSK--rib #, costochondritis Neuro--spinal root comp., thoracic outlet syndrome Skin--herpes zoster, post-herpetic neuralgia Other--psychological, hyperventilation/anxiety

What are ten risk factors for elder abuse? (Ch 69)

Caregiver risk factors - Alcohol or drug abuse - Mental illness - Financial stress - Stress as a result of caring for the elder - Stress related to outside factors - Financial dependance on elder - Unrealistic expectations regarding caregiver responsibility - Lack of caregiving skills - Long duration as caregiver Elder Risk factors - Physical or functional impairment - Financial dependance on caregiver - Cognitive impairment or dementia - Social isolation - Low social support - History of family violence - Previous traumatic event exposure - Aggressive behaviour - Female - Advanced age - Incontinence - Frequent falls Environmental and family factors - Shared living - Overcrowded living - Lack of family and community support - Socially isolated Risk factors for institutional abuse - Poor working conditions - Inadequate training, experience Low wages Low staff to patient ratio

Describe scabies and it's treatment (Ch 120)

Caused by the penetration of the obligate human parasitic mite Sarcoptes scabieivar hominis into the epidermis. The pruritus is typically worse at night. Clinical findings include small (<5 mm) papules or pustules and small raised or flattened burrows. In the ED, treatment should be instituted based on a clinical suspicion of the diagnosis of scabies. First line treatment of scabies is topical permethrin 5% cream (whole body) and repeatFd in 1-2 weeks. Items that cannot be washed and or dry-cleaned can be decontaminated by sealing the items in an airtight container for at least 72 hours.

Describe the management chancroid (Ch 98)

Ceftriaxone 250 mg IM single dose OR Azithromycin 1000 mg PO single dose OR Ciprofloxacin 500 mg PO BID x 3 days.

Describe the pathophysiology of cyanide poisoning and the antidote(Ch. 64)

Cellular toxin - Inhibits oxidative phosphorylation - Cellular hypoxia and death - Leads to profound lactic acidosis, elevated mixed venous O2 saturation, shortened QT interval, normal pulse oximeter Antidote - Amyl nitrite: pearls are broken open and breathed for 30 seconds on 30 sec. Off - Sodium nitrite: 300 mg IV dose over 5 minutes - Sodium thiosulfate: 12.5 g IV dose Hydroxocobalamin 5g diluted in 200ml of 5% dextrose IV over 30 min

Describe the three common partial cord syndrome, and list 3 most likely causes of each. (Ch. 106)

Central cord syndrome: Bilateral motor paresis(U>L). Distal > proximal. UE burning dysthesias - Hyperextension injury - Falls - MVC in elderly with OA ad stenosis - Cervical canal narrowing(Disc protrusion/ tumor) Brown-sequard syndrome: Ipsilateral motor loss. Ipsilateral vib/ proprioception. Contraletal pain and temperature - Penetrating injury - Tumor, epidural hematoma, AVM, spondylolysis, DDD, herpes myelitis, radiation or iatrogenic spinal injury Anterior cord syndrome: Loss of motor function, pinprick, light touch below level. Preserve posterior column - Post op herniation in aortic surgery - Infection, MI, vasospasm - Herniated bone fragments

Cognition is controlled by what? (Ch. 16)

Cerebral cortex - determines content of consciousness

List three potential complications of ischemic stroke.(Ch. 101)

Cerebral edema progressing to increased ICP and deterioration (needing ICU care) Hemorrhagic transformation GI bleeding CHF Hospital related complications: DVT, PE, UTI's, pneumonia Post-stroke seizures Post-stroke delirium and depression

List four anatomic abnormalities associated with thoracic outlet syndrome. (Ch 87)

Cervical rib syndrome = an extra rib (70% bilateral) Scalenus-anticus syndrome = the neurovascular Bundle is pinched by the anterior scalene muscle. Costoclavicular syndrome = shoulders moved back/down due to muscle hypertrophy/trauma Hyperabduction syndrome = when the arms are positioned in a hyper-abducted position and the pectoralis minor muscle compresses the neurovascular Bundle.

What is the causes of chancroid? (Ch 98)

Chancroid - Ulcerating infection caused by gram-negative organism Haemophilus ducreyi. Common in developing world but extremely uncommon in Western world. Incubation of less than a week preceded by tender erythematous papule that rapidly ulcerates to form multiple, irregular, inflamed, painful, and 'dirty' ulcers. Painful inguinal lymphadenopathy is common.

List the PPE for first responders or caregivers in chemical exposure injury(Ch. 64)

Chemical-resistant clothing with a hood Boots Eyewear Two layers of gloves Respiratory mask

List the treatment options of chlamydia and gonorrhoea infections (Ch 98)

Chlamydia (urethritis, cervicitis, proctitis, pharyngitis) Azithromycin 1 g PO single dose or Doxycycline 100 mg PO BID x 7 days. Gonorrhea (urethritis, cervicitis, proctitis, pharyngitis) Ceftriaxone 250 mg IM single dose plus Azithromycin 1 g PO single dose.

Broad differential diagnosis for coma: (Ch. 16)

D- Drugs I- Infection M- Metabolic E- Environmental S- Structural

Describe the diagnosis and tx of Otitis Externa. (CH. 72)

Clinical diagnosis: itchy, painful, red, swollen auditory canal. Pain with tugging on auricle or tragus. Associated adenopathy. -Otalgia -Ear pressure / fullness -hearing loss -jaw pain TX: Cleaning Tap water, sterile saline, 2% acetic acid or Burows solution (aqueous solution of aluminium triacetate) Treat infection: Canada = ciprodex! Topical antibiotics: cure rate 80% within 10days. A combination of polymyxin B, neomycin, and hydrocortisone 3 or 4 drops to the affected ear four times a day ofloxacin (5 drops) or ciprofloxacin with hydrocortisone 3 drops given twice a day may be better, as less interval means more patient compliance. In canada we have Ciprodex (ciprofloxacin and dexamethasone)

Describe the distribution of pityriasis rosea (Ch 120)

Clinical presentation includes multiple pink or pigmented oval papules or plaques 1 to 2 cm in diameter on the trunk and proximal extremities. A history may reveal an initial larger patch ("herald patch") that precedes the widespread eruption. The lesions are parallel to the ribs, forming a Christmas tree-like distribution on the trunk and extremities.

What is spinal shock? (Ch. 43)

Clinical syndrome characterised by temporary loss of neurologic function and autonomic tone below lesion. Lasts 24 hours to 2 weeks, and is heralded by the return of the bulbocavernosus reflex.

What are the indications for steroids in a patient with pharyngitis? (Ch 75)

Cochrane review in 2012 concluded that symptoms were improved, but recurrence rates did not change. The study was also underpowered for complication, and not enough children were included for analysis. *** infectious mononucleosis, peritonsillar abscess, and post-tonsillectomy patients were excluded, as these patients usually are managed with steroids.*** The plot thickens with a recent publication in JAMA, which showed no difference in intervention versus placebo with complete resolution of symptoms at 24hrs, but did show a reduction in symptoms at 48 hours. This is only in Adults.

List the DSM V criteria for Major Depressive Episode (Ch 111)

Coles notes: - A change from baseline, with 2 weeks of daily depressed mood or loss of interest/pleasure and at least 5 of the SIGE CAPS items - Impaired social/job function - Not caused by a medical condition or drug ingestion - Not explained by another mental health disorder - No history of mania

Phenotypic characteristics of coral snakes(Ch. 55)

Color pattern: Red and yellow are adjacent. If red touches black, not venemous. Of note: In brazil this does not apply! only NA

List 5 classes of venomous snakes(Ch. 55)

Colubridae - Boomslang and bird snake Elapidae: Cobras, kraits, mambas and coral snakes Viperidae: True vipers Crotalidae: Pit vipers(Rattle snakes, copper heads, water moccasins, pygmy rattlesnakes) Atractaspididae: Mole vipers

List 3 common UTI pathogens, and list 3 additional pathogens in complicated UTIs (Ch 99)

Common causes Coli - 75% or more Saprophyticus Klebsiella pneumonia Proteus mirabilis Complicated, unusual causes: ESBLs (extended-spectrum β-lactamases) coli Klebsiella Pseudomonas Serratia Citrobacter Salmonella / Shigella Mycobacterium tuberculosis

List 3 common viruses that can trigger an AECOPD episode.

Common viruses include: - rhinovirus - respiratory syncytial virus - coronavirus - influenza virus

What is the difference between a simple and a complex seizure? (Ch. 102)

Complex seizure includes a loss of conscioussness where as a simple seizure the patient may be fully aware

What is the CURB65 score? (Ch 76)

Confusion Uremia (blood urea nitrogen >20 mg/dL) Respiratory rate greater than 30 Blood pressure less than 90 systolic or less than 60 diastolic, 65 years (age) or greater. How to use it: The risk of 30-day mortality increases with a greater number of these factors present: 0.7% with zero factors 9.2% with two factors 57% with five factors. Patients with zero or one feature can receive o/p management 2 - Admit 3 - consider ICU

What is ophthalmia neonatorum? In which time-frame are each bacteria expected to be causative? What is the treatment? (Ch. 71)

Conjunctivitis in the first month of life Need to rule our Gonorrhea and chlymadia!(Gonn: 2-4 days after birth, chlymadia 2-15 after birth) Tx: Ceftriaxone IM 50mg/kg(If systemic), Topical polymyxin B, saline washes, topical erythromycin ointment

Differentiate between conus medullaris and Cauda equina syndrome. (Ch. 106)

Conus Medullaris(L1) - Weakness may be of an upper motor neuron type. Usually bilateral Causes: Central disc herniation, neoplasm, trauma, vascular insufficiency - Fecal/ overflow urinary incontinence, impotence, distal motor weakness, sensory in saddle distribution Cauda equina - Weakness may be of lower motor neuron type(Unilateral) Causes: Midline rupture of an intervertebral disc, tumor - Other findings same as above, with urinary retention is most consistent finding

How do you judge the severity of a alkali/acid burn to the eye? (Ch. 71)

Corneal cloudiness

Describe the Nexus CT chest rule" (Ch. 45)

Courtesy of: http://www.mdcalc.com/nexus-chest-decision-instrument-blunt-chest-trauma/

Name 6 critical causes and 6 complications of vomiting. (Ch. 29)

Critical diagnoses: Boerhaave's syndrome; Ischemic bowel; Testicular torsion ICH / Ischemic stroke; DKA; Pregnancy; MI Organophosphate overdose; Tylenol / Digoxin / ASA OD Complications: see shownotes

What does *cyanosis* mean? (Ch. 14)

Cyanosis = an imbalance between oxygenated and deoxygenated hemoglobin... By definition: *deoxy*hemoglobin is >5g/dL (absolute amount) *That is why anemic individuals with low O2 sats (but less overall hemoglobin) may not be cyanotic. They only turn blue when SaO2 is SEVERELY low.

What is the treatment of UV keratitis? (Ch. 71)

Cycloplegic, topical borad spectrum abx, po analgesics ** should resolve in 24 hours

Define Cyclothymic Disorder (Ch 111)

Cyclothymic disorder is characterized by chronic mood swings that do not meet criteria for a hypomanic or depressive episode. The mood episodes must occur over at least 2 years, present for at least half the time, and the individual cannot be symptom free for more than 2 months at a time.

Describe treatment of acute angle closure glaucoma. If IOP is greater then 30, how does this change treatment? (Ch. 71)

Decrease production of aqueous humor! - Timolol 0.5% 1 drop, then repeat in 30 minutes - Apraclonidine 1% 1 drop once - Acetazolamide 500 mg PO - to reduce aqueous humour production - Methazolamide 50mg PO instead of acetazolamide of the patient has sickle cell disease Decrease IOP - Head of bed at 30 - Anti emetics - Analgesice Decrease inflammation - Prednisolone 1% 1 drop q15 minutes IOP >30 --> Emergency! - Contrict pupil: Pilocarpine 4% 1 drop then repeat in 15 minutes - Osmotic gradient: mannitol 2g/kg IV

List 8 general medical conditions that cause depression/mood disorders (Ch 111)

Degenerative: Parkinson's disease CAD MI Stroke End stage renal disease AIDS Connective tissue diseases Hyperthyroidism Neoplastic Pancreatic CA Brain neoplasm Lymphoma

Broad differential diagnosis of confusion: (Ch. 17)

DIMES (Drugs, infection, metabolic, environmental, structural) or -Primary intracranial disease -Systemic disease secondarily affecting CNS -Exogenous toxins -Drug withdrawal

List 8 risk factors for candida infections (Ch 120)

DM, HIV infection, pregnancy, obesity, smoking, malnutrition, malignancy, or treatment with corticosteroids, antibiotics, or immunosuppressive agents.

What colour is methemoglobinemic blood when exposed to air? (Ch. 14)

Dark brown-purple, like chocolate

List clinical findings that support a diagnosis of PID as described by the CDC (Ch 98)

Diagnosis should be considered and presumptive treatment initiated in any sexually active woman at risk for STD's who presents with lower abdominal or pelvic pain and one or more of the following findings on pelvic examination: 1. Cervical motion tenderness or 2. Uterine tenderness or 3. Adnexal tenderness

Describe diagnosis and management of Tic Douloureux(Trigeminal neuralgia) Ch. 105

Diagnosis: Normal head and neck exam. No neurologic deficits. - Episodic, unilateral facial pain associated with non painful triggers Management - Carbamazapine 100mg po BID Other options include: Phenytoin, baclofen, valproate, lamotrigine, gabapentin, levetiracetam Dispo: Neurology +/- Neurosurgery or ENT is decompression required

Describe Raynaud's Disease and how it is treated. (Ch 87)

Diagnostic criteria: Precipitated episodes after cold or emotional upset Bilateral symptoms No or minimal gangrene (tissue loss) No disease condition that could cause secondary Raynaud's phenomenon is present Symptoms have occurred for at least 2 years The classic attack is tri-phasic: Pallor (chalk white). → cyanosis → rubor/red Complete closure of the palmar/digital arteries Slight flow of blood Arterial flow returns to baseline - with hyperemia Usually has a benign course. True Raynaud's disease can be managed supportively. (occasionally CCB's are used for symptomatic relief).

Some important neonatal-specific disorders to remember: (Ch. 11)

Diaphragmatic hernia Meningomyelocele Omphalocele Choanal atresia Pierre-Robin sequence

What is the hyperoxia test? (Ch. 14)

Differentiates causes of cyanosis between R to L shunt and V/Q mismatch in lungs: Give 10 min high flow O2 and if sats improve, likely V/Q (lung) issue *Beware: imprecise tool

When do we give empiric antibiotics for diarrhea? (Ch. 31)

Direct from ROSEN'S: "Antibiotic treatment is initiated in patients with a suspected invasive process and severe diarrhea, systemic symptoms, fever, or abdominal pain and in patients who appear toxic."

Describe the appearance of a brown recluse spider and where you might find them(Ch. 55)

Distinguishing features include a violin shaped darker area on the cephalothorax and three pairs of eyes instead of four Found in protected areas such as woodpiles and rocks. More common is south- central US, but report as far as the canadian border.

How is diverticular disease managed in the ED? Which patients should be admitted to hospital? List 3 complications (Ch 95)

Diverticulosis High fiber diet, period. Uncomplicated diverticulitis Antibiotics (?some controversy about this!!) See Box 85.3 in Rosen's Soft / liquid diet (for comfort - not mandatory) NSAIDS / narcotics for pain relief (better Rx a stool softener or the constipation will worsen the disease!!) Most young, immunocompetent patients can be managed with outpatient treatment and follow-up in 2-3 days. Consider admission for IV abx if: Inadequate po intake, high pain No social supports Elderly Poor follow up

Describe the post-exposure prophylaxis for exposure to Hepatitis C (Ch 90)

Doesn't exist - Unknown benefit from pre/post exposure prophylaxis. Can consider ISG 0.06 mL/Kg.

What species are known to cause infection with pasteurella multocida?(Ch. 54)

Dogs & Cats possum, rat, Lion, rabbit, pig, wolf, Monkey, Cougar

What are door-to-needle and door-to-balloon timelines by AHA recommendations? (Ch 78)

Door to needle/drug (fibrinolytic): 30 mins Door to balloon: 90 mins

List commonly used anti-emetics including their dose and their receptor site of action. (Ch. 29)

Dopamine D2 antagonists (metoclopramide) Serotonin receptor antagonists (ondansetron) Cholinergic & histamine receptors antagonists (diphenhydramine, scopolamine, dimenhydrinate) Cannabinoids

What are the anatomical borders of the "snuff box"? (Ch. 50)

Dorsally - Extensor pollicis longus Volarly - Extensor pollicis brevis and abductor pollicis longus Proximally - radial styloid Distally - approximate apex of the triangle (where Ext. Pol. Longus and Ext. Pol. Brevis/Abd. Pol. Longus meet) Floor - radial artery, scaphoid, trapezium

What are the symptoms of a black widow spider bite? What might an ECG show?(Ch. 55)

Dull crampy pain in the bite, with spread to the rest of the body over the next hour. The abdomen may become board like with minimal tenderness. Pregnant women may go into premature labor. Other symptoms include dizziness, restlessness, ptosis, N/V, pruritus, dyspnea, and well... just about anything! ECG may indicate dig toxicity like changes

List the DSM V Criteria for a Manic Episode (Ch 111)

Duration > 1 week (or less if hospitalization needed) 3 or more of GST PAID symptoms Impairs social/job function NOT caused by a medial or drug ingestion

Soft signs of laryngeotracheal injury: (Ch. 44)

Dysphonia / aphonia / dyspnea / stridor / hemoptysis / subcutaneous emphysema / laryngeal crepitus / loss of anatomic landmarks / pain with tongue movement hoarseness / tenderness over larynx

What should always be your first-line agent for anaphylaxis? (Bonus: what is the adult dosing?) (Ch 119)

Epinephrine. All other drugs can wait! Adult: 0.3 to 0.5 mg IM* (1 :1000 concentration) in anterolateral thigh every 5 to 10 minutes as necessary

What is the etiology for a vitreous hemorrhage? What is the tx?(Ch. 71)

Etiology: Diabetic retinopathy, retinal tears, sickle cell disease, age related macualr degeneration Tx - Bed rest, elevated head of bed - NO anticoagulation - May need laser photocoagulation or retinal tear repair

List 6 signs of elevated ICP in infants (0-1 yrs): (Ch. 38)

In infants: 1) full fontanel 2) split sutures 3) altered state of consciousness 4) paradoxical irritability 5) persistent emesis 6) setting sun sign

List classic risk factors for PE x 10 (Ch 88):

Every PE has its birth as a DVT somewhere in the body, which migrates to the lungs. See Table 88-2 for a detailed list. These are the ones that are highly associated with PE in the ED population: Hypercoagulable states: Inherited thrombophilia Active cancer Estrogen Prior PE / DVT Endothelial damage Surgery or Trauma within the last 4 weeks requiring hospitalization/GA Venous stasis: Surgery or Trauma within the last 4 weeks requiring hospitalization/GA Clinical signs/symptoms: Dyspnea Hemoptysis Pulse > 100 O2 Sat < 95% Unilateral leg / arm swelling Notice that smoking is NOT on that list! Smoking does not seem to increase the risk for PE compared with another patient who is a non-smoker with the same clinical presentation.

Describe the ER diagnosis and management of Spontaneous Bacterial Peritonitis (SBP) (Ch 90)

ED treatment should begin after paracentesis (don't forget to check the coags/plts first!): If neutrophils (PMN's) > 250 cells/mm = ceftriaxone 2 g IV x 5 days. If PMN's < 250 cells/mm and infectious signs/symptoms = cefotaxime 2 g IV q8hrs x 5 days

List two potential side effects of haloperidol(Ch. 104)

EPS Prolonged QT

Fracture blisters. Common sites of eruption? (Ch. 49)

Elbow, ankle, foot, knee

Describe the Ellis classification for dental fractures: (Ch. 42)

Ellis I -enamel only, can have outpatient follow-up Ellis II -enamel and dentin visible (yellow substance of tooth), can have outpatient follow-up but can benefit from covering/protection of dentin Ellis III -enamel, dentin, and pulp visible (small red line or dot), need early referral to dentist or endodontist

Describe the classification and management of tooth fractures(Ch. 70)

Ellis classification: Ellis I - enamel only, can have outpatient followup Ellis II - enamel and dentin visible (yellow substance of tooth), can have outpatient followup but can benefit from covering/protection of dentin Ellis III - enamel, dentin, and pulp visible (small red line or dot), need early referral to dentist or endodontist

List 8 causes of an elevated D-dimer (Ch 88)

Endothelial damage Aging Recent surgery Infection New indwelling catheters Inflammation StrOke MI Venous stasis Prolonged bed rest / limb casting Hypercoagulable state: Active malignancy Pregnancy

List the imaging findings of adnexal torsion on ultrasound (Ch 100)

Enlargement of ovary Associated ovarian mass Loss of enhancement Edema Free pelvic fluid Loss of venous waveforms Loss of arterial waveforms

Post-mortem C-section procedure: (Ch. 37)

Ensure high quality CPR is underway! chlorhex splash midline vertical incision from epigastrium to symphysis pubis vertical incision of the uterus deliver fetus clamp and cut cord Should be less than 3 minutes

What are the steps to a lateral canthotomy(Ch. 71)

Ensure the patient has one of the absolute / relative indications for this procedure: DIP A CONE Informed consent Don PPE Wash the area with saline 1-3 ml 1% lidocaine with epi. Into the lateral canthus (consider light procedural sedation) Devascularize with hemostat Incise the lateral canthus Pull lower lid down and localize the inferior canthal tendon - then cut it with iris scissors Reassess, and repeat for the superior canthal tendon if needed

List 3 causes of acute scrotal swelling in adults (Ch 99)

Epididymitis Hernia Trauma Tumor Torsion Fournier's gangrene

List the ddx of loss of conscioussness post seizure(Ch. 102)

Hypoglycemia CNS infection CNS vascular event Drug tox Psychiatric disorder Met. encephalopathy Migraine Transient global amnesia

List the mechanisms of drug toxicity in CKD (Ch 97)

Excessive drug level Impaired renal excretion of drug Impaired renal excretion of metabolite Impaired hepatic metabolism Increased sensitivity to drug Changes in protein binding Changes in volume of distribution Changes in target organ sensitivity Metabolic loads administered with drug Misinterpretation of measured serum drug level

Management of placental abruption: (Ch. 37)

Expectantly until 32 weeks after which OB may elect to C/S

Fast Facts for Hands: What extends and hyper-extends the thumb? (Ch. 50)

Extensor pollicis longus

What are some potential external etiologies of mechanical small bowel obstruction? (Ch 92)

External = compress the gut from the outside, preventing peristalsis 1. Adhesions (scarring post-op) > 60% 2. Hernias (internal or external) 3. Volvulus - Usually occurs in the normal abdominal cavity; may have increased incidence during Ramadan; small bowel volvulus can commonly occur in children < 1 month of age 4. Compressing masses (tumours, abscess, hematomas)

Where can subcutaneous air be coming from? (Ch. 45)

Extra-pleural injury (from tracheobronchial tree) -air in anterior neck/supraclavicular Intra-pleural injury (break in the visceral/parietal pleura) -air in chest wall (over site of injury) Boerhaave's Syndrome -air in anterior neck/supraclavicular

List indirect signs of a foreign body on Xray: (Ch 60)

Eye: free air Ears: erosion / infection of the mastoids Nose: chronic erosive sinositis Neck: prevertebral swelling or soft tissue emphysema (cervical spondydisktis) Lungs: - Pulmonary Gas trapping (flat & fixed diaphragm on insp vs exp, shift of mediastinum away from affected side during exp) Narrowing of subglottic space Atelectasis Bronchiectasis and bronchial stenosis (late)

Fast Facts for Hands: What are the 3 wrist flexors? (Ch. 50)

FCR, FCU, PL

List DSM-5 criteria for the diagnosis of Factitious Disorder Imposed on Another (Ch 114)

FDIA is an especially pernicious variant that involves the simulation or production of factitious disease in children by a parent or caregiver. This is different from child abuse... the key discriminator is motive: e.g., the mother is making the child ill so that she can vicariously assume the sick role with all its benefits. The condition excludes straightforward physical abuse or neglect and simple failure to thrive; mere lying to cover up physical abuse is not FDIA.

How do you test for function of flexor digitorum superficialis and profundus? (Ch. 50)

FDS - hold finger in extension at the MCP, patient should be able to flex PIP FDP - hold finger in extension at the PIP, patient should be able to flex DIP

3 Types of plasma: (Ch. 7)

FFP Cryo Cryo-poor plasma

What is a factitious disorder? (Ch 114)

Factitious disorders are characterized by symptoms or signs that are intentionally produced or feigned by the patient in the absence of apparent external incentives. Common examples of simulated symptoms include a fever of unknown origin or seizure disorders. The Diagnostic and Statistical Manual of Mental Disorders, Fifth Edition (DSM-5) classifies factitious disorders into two types: 1. Factitious disorder imposed on self (FDIS) 2. Factitious disorder imposed on another (FDIA)

Components of octaplex: (Ch. 7)

Factors II, VII, IX, X + protein C & S

What are 7 risk factors for intimate partner violence? (Ch 69)

Female Younger age Exposure to childhood familial violence Physical or mental disability Use of alcohol by either party Lower SES Immigrants

What sites are at highest risk of developing AVN post-fracture? (Ch. 49)

Femoral head Talus Scaphoid Lunate Capitate

What fractures can relate to significant blood loss if not recognized early? (Ch. 49)

Femur, pelvis, tib-fib

What is a safe radiation dose in pregnancy? (Ch. 37)

Fetal demise RARE with <10 rads (equivalent to 2 CT abdos) chest Xray = 5 milli-rads pelvis Xray = 200-2000 milli-rads (0.2-2 rads) CT head = 50 milli-rads CT abdomend = 3 rads CT pelvis = 3-9 rads**

What are the grades of pit viper envenomation(Ch. 55)

Grade 0: No evidence of envenomation, but suspected bite. No pain, less then one inch of surrounding edema. No systemic or lab changes Grade I (minimal). There is minimal envenomation, and snakebite is suspected. A fang wound is usually present. Pain is moderate or throbbing and localized to the fang wound,surrounded by 1 to 5 inches of edema and erythema. No evidence of systemic involvement. No laboratory changes occur. Grade II (moderate). There is moderate envenomation, more severe and widely distributed pain, edema spreading toward the trunk, and petechiae and ecchymoses limited to the area of edema. Nausea, vomiting, and a mild elevation in temperature are usually present. Grade III (severe). The envenomation is severe. The case may initially resemble a grade I or II envenomation, but the course is rapidly progressive. Within 12 hours, edema spreads up the extremity and may involve part of the trunk.Petechiae and ecchymoses may be generalized. Systemic manifestations may include tachycardia and hypotension.Laboratory abnormalities may include an elevated white blood cell count, creatinine, phosphokinase, prothrombin time,and partial thromboplastin time, as well as elevated fibrin degradation products and D-dimer. Decreased platelets and fibrinogen are common. Hematuria, myoglobinuria,increased bleeding time, and renal or hepatic abnormalities may also occur. Grade IV (very severe). The envenomation is very severe and is seen most frequently after the bite of a large rattlesnake. It is characterized by sudden pain, rapidly progressive swelling that may reach and involve the trunk within a few hours, ecchymoses, bleb formation, and necrosis. Systemic manifestations, often commencing within 15 minutes of the bite, usually include weakness, nausea, vomiting, vertigo, metallic taste in mouth, and numbness or tingling of the lips or face. Muscle fasciculations, painful muscular cramping, pallor, sweating, cold and clammy skin, rapid and weak pulse, incontinence, convulsions, and coma may also be observed. An intravenous bite may result in cardiopulmonary arrest soon after the bite.

What antibiotics are used to prophylactically treat open fracture wounds? (Ch. 49)

Grade I: Ancef Grade II-III: add gentamicin or broad spectrum coverage (pip-tazo)

Describe the antiobiotic treatment of septic arthritis (Ch 116)

Gram +: the initial drug of choice is vancomycin 30 mg/kg daily in two divided doses, as MRSA is frequently causative. Gram -: 3rd-generation cephalosporin, such as ceftriaxone 2 g IV once daily, cefotaxime 2 g IV three times a day, or ceftazidime with gentamicin (especially if Pseudomonas infection is suspected).

List the most common bacterial causes of sore throat: (Ch. 23)

Group A beta-hemolytic strep Non-group A strep Neisseria gonorrhoeae Neisseria meningitidis

What are the hard signs of peripheral vascular injury? (Ch. 48)

H - hematoma (expanding) A - arterial bleeding (pulsatile) R - bRuit or thRill D - Don't feel Distal pulse

What is the HINTS exam for vertigo? (Ch. 19)

HINTS (Head impulse, nystagmus, test of skew)... a three part exam to investigate central causes of vertigo. Remember *INFARCT* as the findings in central vertigo: I.N. = *I*mpulse test *N*ormal F.A. = *F*ast *A*lternating (bidirectional) nystagmus R.C.T = *R*efixation with *C*over *T*est

When would you give someone HIV PEP in sexual assaults (Ch 67)

HIV PEP in high risk encounters only - Unknown assailant - Drug paraphenalia at scene

With every 1 degree increase in core body temp, how do HR and RR change? (Ch. 12)

HR increases by 10-20 beats/min RR increases by 2-4 resps/min

Describe herpetic whitlow and Tx: (Ch. 50)

HSV infection of the distal finger (Pain, pruritis, swelling of finger followed by clear vesicles) *Avoid* I+D - can result in viral dissemination and it can be hard to tell this apart from a felon or paronychia: careful history of risk factors is needed Can consider oral acyclovir, especially in recurrent infections or patients who are immunocompromised

List 3 potential medications used as chemical restraints(Ch. 104)

Haloperidol 0.5-10mg IM -- Tried and true 2nd gen antipsychotics(Olanzapine, risperidone) Benzos --> Lorazepam

What historical features are concerning for intra-ocular foreign body?(Ch. 71)

Hammering, grinding, metalworking, machine operating Explosions, firearm use

When would you give black widow spider anti venom to a patient who has been bitten? What is the dose?(Ch. 55)

Hard criteria for antivenin include: seizures, resp failure, uncontrolled HTN, pregnancy, and lack of response to prior therapies. Also consider if young, elderly or pregnant Dose is one vial in 50ml NS over 15 minutes. Allergic reactions are common.

What effect does mental illness have on sexual assault (Ch 67)

Has been shown to increase severity

What cluster of injuries is seen in shaken baby syndrome?(Ch 66)

Head Trauma: Traumatic axonal injury, Retinal hemorrhages(Dot, blot, shear and flame hemorrhages Skeletal injuries(Can have no scalp hematome or skull fractures!!) Abdominal injuries: Pancreatitis, liver and spleen lacerations, duodenal hematomas, viscus perforations

What is methemoglobin? (Ch. 14)

Hemoglobin with iron that has been oxidized from Fe2+ to Fe3+ (ferric state) and thus is unable to bind O2 or transport it to tissues or remove CO2, leading to hypoxia and acidosis

Causes of *hypovolemic shock*: (Ch. 6)

Hemorrhage Third spacing (burns, albumin deficiency) GI losses

List and describe 6 critical causes of jaundice: (Ch. 28)

Hepatic ----fulminant failure, toxins, hepatitis, ischemia, Reye's Biliary ----cholangitis (ascending) Systemic ----sepsis, heastroke CVS ----obstructing AAA----Budd-Chiari syndome Heme-Onc ----trandusion reactions (hemolysis - 1/30,000 chance) Reproductive ----pre-eclampsia, HELLP syndrome

Describe your approach to sudden hearing loss How is it treated?(CH. 72)

Here we are worried about sudden sensorineural hearing loss (SSNHL). Usually idiopathic hearing loss of >30db. This is an ENT emergency! Usually seen is 40s-50s, male:female. Can be simple as difficulty with conversation to complete hearing loss. Outcome is based upon severity of hearing loss. Look for accompanying tinnitus for a clue. Make sure you don't miss a central (stroke) cause. Examine tympanic membrane and auditory canal (cerumen impaction or FB). Solid neuro exam including weber and rinne's test Tx: ENT consult! Some literature about antivirals and steroids

What is Hutchinson's sign in the context of opthalmic infections?(Ch. 71)

Herpes Zoster infection. Involvement of the nasociliary branch of the trigeminal nerve, which manifests with zoster lesions on the tip of the nose (Hutchinson's sign), is associated with a 76% risk of ocular involvement versus a 34% risk if the nerve is not involved.

Threshold for pRBC transfusion in critical care: (Ch. 7)

Hgb <70

Threshold for pRBC transfusion for hip fractures: (Ch. 7)

Hgb <80

What are the Denver Criteria? (Ch. 43)

High risk vertebral signs and symptoms 1) Any cervical spine fracture 2) Unexplained neurological deficit incongruous with imaging 3) Basilar cranial fracture into carotid canal 4) Le Fort II or III fracture 5) Cervical haematoma 6) Horner syndrome 7) Cervical bruit 8) Ischaemic stroke 9) Head injury with Glasgow Coma Scale score<6 10) Hanging with anoxic injury

What are the high risk features of a corneal foreign body? What is the treatment? When do these need to be referred to optho?(Ch. 71)

High risk: Grinding, drilling, saws, hammering --> consider CT Tx: Full eye exam, topical anesthetic, attempt to remove(Irrigation, cotton tip applicator, 18g blunt needle) Referral: Deeply embedded, in the visual axis

Are steroid indicated for C-spinal injuries? (Ch. 43)

Highly controversial - talk to your surgeon

What are the indications for emergent hemodialysis following acute acetominophen ingestion(Ch. 148)

Highly elevated acetominophen concentration(>1000mg/L) at 4 hour mark Hepatorenal syndrome(Cr >350) Metabolic acidosis with pH <7.3 Encephalopathy Elevated lactate(>3.5)

List 6 mediators of anaphylaxis (Ch 119)

Histamine Leukotrienes Prostaglandins Tryptase Cytokines Chemokines

What are 5 clues on history, and 5 on physical exam of intimate partner violence? (Ch 68)

History Admission of IPV Vague or changing history Injuries inconsistent with history Statement that patient is "accident prone" Past history of injuries Physical exam Centrally located injury (ie. trunk, breasts) Bilateral injuries Defensive injuries Patterned injuries Head,face,neck injuries

List 6 physical exam findings of strangulation (Ch 68)

Hoarse voice Dysphagia or odynophagia Difficulty breathing LOC Incontinence Confusion Chronic concussive symptoms (from "shaken adult syndrome")

Compare a hordeolum and a chalazion based on location. What is the treatment for both? (Ch. 71)

Hordeolum: Diffuse swelling and erythema on eyelid margin Chalazion: Obstructed meibomian gland(swelling in the lid surface) --> normal lid margin Tx: Warm compresses x 15 minutes 6 times daily. Rarely need I+D or abx, if procedure needed consults our opthamologist friends

List 6 historical predictors of acute HF and 6 clinical features of acute HF (Ch 81):

Hx 1. Past history of HF 2. PND 3. SOBOE 4. Orthopnea 5. Nocturia 6. Hx of any type of heart disease Px 1. Hypertension 2. Diaphoretic 3. Pulmonary crackles 4. Pulmonary wheezes (cardiac asthma - due to peribronchial edema) - which may respond to bronchodilator therapy!! 5. JVD 6. Edema 7. S3

"Critical" causes of confusion you don't want to miss: (Ch. 17)

Hypertensive encephalopathy Hypotension (shock) Hypoxia Hypoglycemia Head infections (the "hyper hippos head")

List 4 antidotes for specific causes of refractory seizures(Ch. 102)

Hyponatremia --> Hypertonic saline --> 2-3 ml/kg in rapid sequence boluses until seizing stops Hypocalcemia --> Calcium chloride/ gluconate --> Amps until seizures stops TCA OD --> Alkalization --> 0.5-1.0 meq/kg IV bolus, maintain pH of 7.4-7.5 Salicylate OD --> Alkalization, HD in severe cases --> Same as TCA Isoniazid OD --> Pyridoxine --> 5g iv adult, 70mg/kg peds Cocaine --> Benzos --> same as normal seizures Lithium tox --> hemodialysis Alcohol --> Lorazepam --> 0.05-0.1mg/kg MDMA --> Benzo --> Be aware of hyperthermia or hyponatremia Eclampis --> MAg!!!! --> 4-6 G IV loading dose over 15-20 minutes then 1-2g/hour infusion. Can use benzos alternatively, and then give mag once seizure aborted

Classic triad of central neurogenic shock: (Ch. 6)

Hypotension Bradycardia Warm extremities

What are the potential complications of a small bowel obstruction? (Ch 92)

Hypovolemia / Shock Persistent N/V with inability to tolerate PO intake Metabolic alkalosis (contraction and loss) Perforation leading to peritonitis Intrabdominal abscess Sepsis Death

5 causes of shock: (Ch. 6)

Hypovolemic Obstructive Cardiogenic Distributive Cellular toxins

How does one differentiate between visual loss and converion/ malingering? What are two tests to discern between these causes? (Ch. 71)

Hysterical conversion: Flat affect, no emotional disturbance despite blindness Malingering: Usually overy emotional to vision loss Normal pupillary reflex, no RAPD, normal fundoscopy Test: - Hesitant to oppose index fingers - Write their names in a obvious disorderly fashion (A blind person can write there name neatly) - Mirror test: Having the person follow their eyes as the mirror is tilted - Optokinetic nystagmus

What are the 4 categories of ocular chemical injuries(Ch. 64)

I-II: Hyperemia and conjunctival eccymosis; some conjunctival haziness III-IV: Deeper penetration causing mydriasis, iris discoloration and cataract formation

Which conditions are associated with the development of abscesses and fistulas? (Ch 96)

IBD Trauma Cancer Radiation injury Infection TB Lymphogranuloma venereum Usually staph. A.; E coli; streptococcus; proteus; bacteroides

Describe the management of refractory hypotension in anaphylaxis (Ch 119)

IV Epinephrine Drip (Adults: 1 to 10 μg/minute IV (titrated to desired effect). Can also consider other vasopressors (dopamine, norepinephrine, vasopressin, phenylephrine).

Describe the pre-hospital management for ischemic stroke (Ch. 101)

Identify Early hospital notification Rapid transport Ensure CNS oxygenation and perfusion (sp02>95%)

What is the preferred term for lower back pain with no red flags on Hx or physical exam? (Ch 54)

Idiopathic Lower Back Pain (note: the cause of back pain remains unknown in 85% of patients after initial investigation).

How is a diaphragmatic injury managed? (Ch. 45)

If abdominal contents herniating through, consider intubation for PPV to counteract lung parenchyma compression, and contact thoracics.

What are some cases where the API will be limited or not helpful? (Ch. 48)

If both limbs are injured If there is an intimal flap allowing partial flow If there is good collateral flow (Proximal subclavian and iliac) If arteries have NO palpable pulses: If the artery in questions is profunda femoris, profunda brachii, peroneal arteries If the injury is a shotgun wounds. If there is venous injuries! (API will miss them)

Define status epilepticus: (Ch. 18)

New definition: 5 or more minutes of persistent seizures OR a series of recurrent seizures without returning to full consciousness in between

When suspecting viral conjunctivitis, when should you consider abx?(Ch. 71)

If no improvement after 72 hours

What the pathophysiology of lupus? (Ch 118)

If you don't know, it's because no one truly knows! This is a multisystem autoimmune disease with every clinical manifestation known to man! Key points: 1. 90% cases are women = strong role of estrogen? 2. HLA alleles, genetics a strong risk factor. 3. SLE is the "prototype of all systemic autoimmune disease." Long story short, the body starts producing antibodies via B-Cells directed towards host antigens, IE starts attacking itself.

Describe your management for Temporomandibular Myofascial Pain Dysfunction Syndrome (Ch. 70)

Imaging w/ xrays not helpful Heat pack 4-6 times / day NSAIDS Soft diet Benzodiazepines for muscle relaxation For refractory cases refer to Oral maxillofacial surgeon

Two ways of considering complications of transfusion: (Ch. 7)

Immune vs non-immune mediated adverse effects or Acute vs delayed adverse effects

Which toxins typically present late?(Ch. 147)

Important ones: - ASA - Acetominophen - Toxic alcohols - Sustained release cardiac duges - Drug packets Others include: - Mushrooms - Other sustained release preparations: CCB, BBlockers, theophylline - MAO inhibitors - Oral hypoglycemic agents - Fat soluble organophosphate insecticides - Heavy metals - Ergotamines - Warfarin - Colchicine - Mathotrexate - Alkylating agents - Neurotoxic snake venom

List 9 signs of elevated ICP in children: (Ch. 38)

In children: 1) headache 2) stiff neck 3) photophobia 4) altered mental status 5) persistent emesis 6) cranial nerve involvement 7) papilledema 8) hypertension, bradycardia, hypoventilation 9) decorticate and decerebrate posturing

List 6 presentations of an abdominal aortic aneurysm (Ch 86)

Incidental: ***most AAA are discovered incidentally!!*** Pain - in the abdomen / back / flank / chest / thigh / groin / scrotum / Often vague, dull This can be in the contexts of weeks/days of pain due to a contained, rupture aneurysm retroperitoneal Throbbing, colicky Acute / severe pain suggests impending rupture An awareness of an abdominal mass / fullness Back pain - due to vertebral body erosion Nausea, Vomiting, weight loss, bowel obstruction symptoms: Due to compression of the duodenum Ureteral colic Ureters being obstructed by the mass Syncope Upper or lower GI bleeding Aorto-enteric fistula: when an unrepaired AAA erodes through - usually the duodenum. It may initially cause intra-abdoMinal infection/abscess and then progress to unexplained GI bleeding. High-output congestive heart failure:

What are the inclusion and relative exclusion criteria for acute ischemic stroke in the 3-4.5 hr time window?

Inclusion Diagnosis of ischemic stroke causing measurable neurological deficit Onset of symptoms within 3 to 4.5 hours before beginning treatment Relative exclusion Older than 80 years old Severe stroke (NIHSS > 25) Taking an oral anticoagulant regardless of INR History of both diabetes and prior ischemic stroke

List indications and contraindications to NIPPV in COPD (Rosen's box!)

Inclusion criteria (one or more) - moderate to severe dyspnea with accessory muscle use or paradoxical abdominal motion - RR >25/min - Moderate-severe acidosis (pH<7.35) or hypercapnia (PaCO2 >45) Exclusion criteria: - cardiovascular instability - respiratory arrest - uncooperative patient - high aspiration risk - recent facial or gastroesophageal surgery - craniofacial trauma - non-fitting mask GOLD 2017 suggests NIPPV for anyone in acute respiratory failure (defined as RR >30, accessory muscle use, hypoxemia requiring >25% FiO2, and increased PaCO2 from baseline or >50 for people with normal baselines)

List 5 causes of shock delivery in patient with ICD (Ch 80):

Increase VF / VT (ischemia / electrolyte disturbance / drug effect) Displaced or break in ventricular lead Recurrent non-sustained VT Sensing and Shock of SVT Oversensing T-waves Sensing non-cardiac signals

List the indications and describe the method for whole bowel irrigation(Ch. 147).

Indications - Extended release preparations - Illicit drug packets - Metals(Iron, lead) Method - Start at 500ml/hr, increase as tolerated - 2L/ hr of PEG Not in critically ill patients, gut hypoperfusion

PENETRATING chest trauma... Crack the chest or not? That is the question... how do you decide? (Ch. 36)

Indications for thoracotomy in penetrating trauma:

List the indications and method for gastric lavage(Ch. 147)

Indicatios - Litta evidence or data showing increased benefit vs. risk. Not indicated to be routinely performed Normal drugs: Lethal ingestion, early ingestion, no known antidote, not amenable to other decontamination strategies, TCA is most common Method - Intubation - Heavy sedation +/- paralysis - L lateral decubitus - 30 Fr or larger orogastric tube - Apply suction

How do you assess pain at each age? (Ch. 3)

Infants: FLACC scale Pre-schoolers: FACES scale Verbal children: visual analog scale 12 to teens: numerical rating scale (NRS) Adults: NRS Elderly: NRS or behaviour observation scale

List the typical etiologies of viral pneumonia (Ch 76):

Infants: Respiratory syncytial virus and parainfluenza virus Children and Adults: Add Influenza (mostly A) and Metapneumovirus Don't forget to order your Nasopharyngeal aspirate for your viral panel to determine what pathogen you're dealing with.

List three common complications of traditional AAA repair (Ch 86):

Infection - difficult to diagnose! Graft contamination Adjacent infectious spread Hematogenous seeding Aortoenteric fistula (AEF) Think about this in anyone with GI bleeding and a history of AAA surgery Usually another GI source is found, but if not, think AEF! Pseudoaneurysm (anastomotic aneurysm) Arise from the leaking anastomosis site

What is dacrocystitis? What is the Tx? (Ch. 71)

Infection of the lacrimal sac, due to nasolacrimal duct obstruction --> Staph aureus Tx: topical ocular and oral antibiotics (amox-clavulin) warm compresses gentle massage to express pppus

List five broad categories of rashes (Ch 120)

Infectious Allergic Autoimmune Vascular Malignancy-related

List 4 DDx for colitis (Ch 95)

Infectious colitis STEC (E coli o157h7) Diff Amoeba Ischemic colitis Radiation colitis Diverticular disease Appendicitis Inflammatory cancer In Kids: Henoch Schonlein Purpura Celiac disease Toxic megacolon

List 15 factors that trigger an AECOPD episode (Rosen's box!)

Infectious: - Viral (rhino, RSV, corona, influenza) - Bacterial (H. influenza, Strep pneumonia, Moraxella, Pseudomonas) - Atypicals (Chlamidia pneumonia and Legionella) Air pollution: - Nitrogen dioxide, ozone, particulates/dust Other: - Pneumothorax - Pulmonary embolism - Lobar atelectasis - CHF - Pneumonia - Pulmonary compression (obesity, ascites, gastric distension, pleural effusion) - Trauma (rib fractures or pulmonary contusions) - Neuromuscular or metabolic disorders - Untreated chronic disease (TB, sarcoidosis) - Medication non-compliance - Inadequate therapy - Inappropriate drugs

The synovial fluid WBCs in this arthritis typically range from 2000 to 50,000/mm3 (Ch 116)

Inflammatory

What are 10 differential diagnoses of a neck mass? (CH. 72)

Inflammatory - Adenitis(Bacterial, viral, fungal, parasitic) - Cat scratch disease - Tularemia - Local cutaneous infection - Sialoadenitis - Thyroiditis - TB, mycoacterium avium Congenital or developmental - Brachial cleft cyst - Thyroglossal duct cyst - Dermoid cyst - Cystic hydroma - Torticolis - Thymic masses - Ranula - Lymphangioma - Laryngocele Neoplastic - benign - Mesenchymal tumor - Salivay gland mass - Vascular abnormality Malignant - Primary tumor - Sarcoma - Salivary gland tumors - Thyroid or parathyroid tumors - Lymphoma Mets

List at least 5 of the extra-intestinal manifestations of IBD (Ch 95)

Inflammatory arthropathies/arthritis Iritis / uveitis / Episcleritis / Scleritis Erythema nodosum Pyoderma gangrenosum Aphthous stomatitis Ankylosing spondylitis Sacroiliitis Osteoporosis **Thromboembolic disease risk - 60% increased DVT PE Cerebral sinus thrombosis Ischemic heart disease Mesenteric ischemia Peripheral neuropathy Primary sclerosing cholangitis (UC)

What is a bursitis? (Ch. 49)

Inflammed bursitis tissue - usually arising from overuse or trauma to joint. Supportive care.

Describe the 1st, 2nd and third line management options for seizure(Ch. 102) - May include adult/ peds doses

Initial Diazepam: 5mg IV up to 20, 10-20mg PR (Peds 0.2-0.5mg/kg IV, 0.5-1mg/kg PR) Lorazepam: 2mg IV up to ma of 10mg(Peds 0.05-0.1mg/kg IV max 2) - Preferred benzo! Midazolam: 5mg, max 10mg IV/IM/IN(Peds: 0.2mg/kg IV/IM/IN max 5) - Best IM benzo Second line Phenytoin: 10 mg/kg IV infusion at 50mg/min(Peds 20mg/kg at 1mg/kg/min) --> may cause hypotension Forphenytoin: 20 PE/ KG IV infusion at 150mg/min or or 20PE/kg IM(Peds: 20PE/kg IF at a rate of 3mg PE/kg/min) Valproic acid: 20-40mg/kg IV at 3-6 mg/kg/min infusion(Peds 20-40 mg/kg IV at 1.5-3mg/kg/min Levtiracetam Third line Phenobarbital: 20mg/kg IV at 50-100 mg/min(Peds same) --> Needs intubation Pentobarbital: 5-15 mg/kg IV loading at 50mg/min, then 0.5-5mg/kg/hr infusion as needed(Peds same) --> Intubation and hemodynamic support Midazolam: 0.2mg/kg IV loading then 0.05-2mg/kg/hr(Peds same) --> titrate to EEG Propafol infusion: 1-2mg/kg IV loading at 1-2mg/kg/hour and increased by 0.3-0.6 mg/kg/hour every 5 min. (Peds same)--> Needs intubation!

Describe inpatient and outpatient treatment regimens for PID (Ch 98)

Inpatient: CefoXAtan 2 g IV q6-12h plus Doxycycline 100 mg PO/IV q12h x 10 days. Clindamycin 900 mg IV q8h plus Gentamicin 2mg/kg IV load then 1.5mg/kg q8h Outpatient: Ceftriaxone 250 mg IM single dose plus Doxycycline 100 mg PO BID x 14 days. +/- Metronidazole 500 mg PO BID x 14 days.

In basic terms, describe respiratory physiology (how do we take a breath, what happens to intrathoracic pressure, how does that affect preload?)

Inspiration = active process Contraction of costal muscles and diaphragm --> Increase intrathoracic volume --->Decreases IT pressure (increase negative pressure) --> Air passively moves in due to pressure gradient Expiration = passive process due to elasticity of lung Blood Flow - deep insp will INCREASE preload!

List one topical anaesthetic for each: intact skin, mucous membranes, and open skin... (Ch. 4)

Intact skin: EMLA patch MM: 2-4% lidocaine Open skin: LAT or LET

What is Stener's lesion? (Ch. 50)

Interposition of soft tissues between bone and adductor aponeurosis preventing it from ever reattaching and healing Diagnosed with MRI or U/S

What are the complications of OM, name 5 intratemporal and 5 intracranial? (Ch. 72)

Intratemporal complications - TM retraction with hearing loss, -ossicular erosions -retraction pocket formation - Balance and motor problems - TM perforation Chronic suppurative otitis media - Cholesteatoma (NOTE: A cholesteatoma:abnormal growth of squamous epithelium IN middle ear +/- mastoid CAN destroy the ossicles. Recurrent AOM, - retraction pockets in the ™ -cholesteatoma) - Mastoiditis Intracranial complications -Meningitis -Epidural abscess (see "Intracranial epidural abscess", section on 'Clinical manifestations') -Brain abscess -Lateral sinus thrombosis -Cavernous sinus thrombosis -Subdural empyema -Carotid artery thrombosis

What are the primary considerations in mechanical ventilation of burn patients?(Ch 63)

Intubation - RSI unless suspicion of airway obstruction - If airway obstruction, awake intubation Mechanical ventilation - Avoid barotrauma - limit plateau pressure to 35 mmH2O - PEEP can be helpful Minimize Fi02, maintain Sp02 of >92% - Low tidal volume ventilation Adjuncts - Bronchodilators can be helpful in wheezing patients - Chest physiotherapy and airway suctioning for scretion management - Dont forget about concomitant CO and cyanide intoxication

What is Horners syndrome? List 8 DDx (Ch. 71)

Ipsilateral PTOSIS, MIOSIS, ANHIDROSIS DDx: strokes tumors lung CA thyroid adenomas pancoast's tumours headache syndrome carotid dissection herpes zoster otitis media trauma to brachial plexus during delivery Inadvertent anesthetic administration into carotid sheath during dental block!

List 5 etiologies of ischemic stroke and 3 specific causes of hemorrhagic stroke. (Ch. 101)

Ischemic In situ thombosis - Atherosclerotic plaque - Small vessel: HTN, diabetes Embolic obstruction - A fib - Infective endocarditis - Extracranial proximal carotic plaque - AGE! Hemorrhagic Hypertensive vasculopathy Cerebral amyloid angiopathy (age related amyloid deposition in the cerebral vessel walls) AVM's Aneurysms Drug related - cocaine Malignant hypertension Blood dyscrasias Hemorrhagic transformation of ischemic stroke/tumour

Why is colchicine used in the treatment of gout, and what is the dosing? (Ch 116)

It impedes the inflammatory response to crystals in the fluid. 1.2 mg PO, then 0.6 mg PO one hour later Then 0.6 mg po TID

Define "judicial hanging" and expected injury pattern: (Ch. 44)

Judicial hanging: -usually distracts the head from the rest of the body due to the high fall, causing a fracture through the pedicles of C2 (hangman's), complete cord transection, and vascular disruption

How is ASA toxicity managed?(CH. 149)

Key: Urinary alkalinization(pH 7.5-8) 150 mEq of bicarb in D5W + 40 mEq of KCl @ 2-3 ml/kg/hr and correct hypovolemia(UO at 2-3 ml/kg/hour. Glucose for ANY CNS changes. Keep K > 4.5, correct hypomagnesemia Consider active charcoal Serial salicylate levels and VBG q2H Initiate hemodialysis if: - Altered mental status, coma, seizure - Renal failure - Hepatic failure - Pulm edema or resp failure - Severe acid-base imabalance(<7.1 to 7.2) - Detioration in condition - Intubation - Failure of urine alkalinization - Rapidly rising saliclate level - Levels > 100mg/dL after acute, >40mg/dL after chronic

How does primary and secondary survey change in prenant trauma patients? (Ch. 37)

Key: mother is main priority from the start Primary Survey: ABCT+UFO -Airway: secure and airway early, RSI -Breathing: Give O2! Goal PaCO2 30mmHg -Circulation: HR and BP not good predictors, uterine bleed? Avoid vasopressors! -Tilt the mom! -Uterus above umbilicus - fetus likely viable (>24wks) -FO-fetal tones Secondary Survery: -Detailed maternal /OB hx - weeks pregnant, GTPAL, vaginal bleeding? discharge? contractions? fetal movement? -Pelvic exam - signs of ferning? cervical dilation? GBS status? RH status? Bimanual exam for pelvic bone protrusion. -Fetal evaluation: FHR, movement

Describe the criteria for transfer to a transplant center(Ch. 148)

Kings criteria - Arterial pH <7.3 INR > 6.5 Creatitine 300 Grade III or IV hepatic encephalopathy

95% of lumbar disk herniations occur at: (Ch 54)

L4-S1 (Know where these dermatomes run!)

List criteria for transfer to burn unit.(Ch. 63)

Large burns (> 20% TBSA burned in adults, > 10% TBSA burned in children or elderly) Deep burns (> 5% full-thickness) High-voltage burns Burns over sensitive areas (e.g. face, ears, joints, perineum, hands, feet) Escharotomy required

In what time frame would one expect peripartum DCM? (Ch 82)

Last 3 months of pregnancy or first 5 months postpartum

What are the maximum doses of two of the most commonly used local anaesthetics (lidocaine and Bupivicaine) (Ch 59):

Lidocaine without epi: 3-5mg/kg (max 35 ml 1% in 70kg), with epi: 5-7 (max 49 ml 1% in 70 kg). Bupivicaine 2.5mg/kg without (25ml of 0.5% in 70kg) 3mg/kg with (max 42ml 0.5% in 70kg).

What is light's criteria? (Ch 76)

Lights criteria (High protein and LDH = exudate), determines presence of exudate with protein and LDH levels Pleural fluid protein to serum protein ratio >0.5 Pleural fluid LDH to serum LDH ratio >0.6 Pleural fluid level >2/3 of upper value for serum LDH Additional criteria - Confirm exudate if results equivocal Serum albumin - pleural fluid albumin <1.2g/dL

Describe the management of peritonitis in the PD patient (Ch 97)

Likely SBP Send dialysate of catheter (sterile technique) if : >100 WBCs/mm3 peritoneal fluid >50% neutrophils Positive Gram staining If fluid is normal then consider ruling out surgical cause IP antibiotics: Vanco and ceftazidime or cefepime Consider adding Heparin 500 to 1000 units to dialysate bag to decrease fibrin formation and subsequent obstruction of catheter

How long does a cooled limb with occlusive vascular injury have until irreversible ischemia sets in? (Ch. 48)

Limbs kept just above freezing to minimize metabolic demands can tolerate up to 24 hrs of ischemia!

List 5 RFs for C. diff (Ch 94)

List 5 RFs for C. diff. What are 2 therapy options? ***First off: REMEMBER TO WASH YOUR HANDS!!! Spores are not killed by hand sanitizers.*** Risk factors: 1. Recent (within 2-4 weeks) / concurrent antibiotic use 2. recent hospitalization 3. living in a long-term care facility 4. use of antacids 5. Recent person to person contact with known infected person

What are the most common intra-abdominal injuries in children? (Ch. 46)

Liver >> Spleen >> Kidney *Also remember hollow viscus perforation

Name 10 causes of epistaxis (CH. 72)

Local causes - Trauma - Uper resp infections - Nose picking - Allergies - low humidity - Polyps - FB in nose - Enviromental irritants - Nasopharyngeal mucomycosis - Traumatic internal carotid artery aneurysm - Chlymadial rhitis neonatorum - Surgery Idiopathic: Familial or habitual Systemic causes - Athersclerosis - HTN(controversial) - Anticoagulent therapy - Pregnancy - Abrupt barometric pressure changes - Hereditary hemorrhagic telangiectasia - Blood dyscrasias - Hepatic disease - DM

What is molluscum contagiosum and how is it managed? (Ch 98)

Localized skin infection presenting with one or more 2-5 mm papules with waxy appearance, and central umbilication. Spontaneous resolution typical in 6-12 months. Patients may seek primary care or dermatologic follow up for curettage, cryotherapy, or treatment with topical agents for persistent lesions.

What blood product is most likely to lead to sepsis? (Ch. 7)

Plasma (stored at room temperature)

What should be achieved during the first 6 hours of resuscitation in sepsis? (Ch. 6)

MAP >65mmHg U/O >0.5ml/kg/hr CVP 8-12 Central venous O2 >70% or mixed venous >65%

Critical causes of *syncope*: (Ch. 15)

MI Arrythmia Leaky AAA Thoracic aortic aneurysm Critical AS HOCM Pericardial tamponade PE SAH Toxin-mediated Hypovolemic shock Ectopic pregnancy (ruptured) GI bleed

What are the characteristics of viral conjunctivitis? (Ch. 71)

MORE redness and itching and eye irritation watery --> purulent d/c pre-auricular lymphadenopathy concurrent viral illness

What is the ED management of a patient with esophageal perforation? (Ch 89)

Mackler's triad: subcutaneous emphysema, chest pain, vomiting = esophageal perforation Misdiagnosis of people with esophageal perforation as ACS/PE/Aortic dissection/perforated ulcer/pancreatitis/pneumothorax/lung abscess - due to chest and abdominal pain! Management: Broad spectrum abx Admission NPO

Describe your management for atypical odontalgia (Ch. 70)

Make sure you're not missing an MI, temporal arteritis or other major referred pain cause Refer to Dentist

Differentiate otitis extrena from malignant otitis externa (CH. 72)

Malignant otitis externa - High associated with immunosuppresion, HIV - Pseudomonas****/ S. aureus/ S. epidermidis/ proteus mirabilis/ klebsiella/ aspergillus/ salmonella - Can cause osteomyelitis of the skull --> travels through the fissures of santorini **think OE with cranial nerve intracranial involvement or an ill appearing patient

What is malingering? (Ch 114)

Malingering: the simulation of disease by the intentional production of false or grossly exaggerated physical or psychological symptoms, motivated by external incentives, such as avoidance of military conscription or duty, avoidance of work, obtainment of financial compensation, evasion of criminal prosecution, obtainment of drugs, gaining of hospital admission (for the purpose of obtaining free room and board), or securing of better living conditions.

What is a mallet finger? Tx? (Ch. 50)

Mallet finger: rupture of DIP extensor tendon (avulsion or direct tendon injury) Tx: maintain DIP in complete, uninterrupted extension for 6-8 weeks

Describe the management of non-thrombosed external haemorrhoids (Ch 96)

Managed Medically using the WASH approach. ● Warm water sitz ● Analgesia ● Stool softeners ● High fiber diet

What are the expected management goals for a black widow spider bite?(Ch. 55)

Management initially is cleaning with soap, water, and tetanus. Patients with symptoms should have lab work including: CBC, lytes, BUN, Creatinine, coags, UA, and ECG. Treatment include Diazepam for muscle spasms, parenteral analgesics for pain and admission for monitoring. Antivenin Lyovac - a horse serum derivative - In high risk scenarios only

Describe your management for maxillary sinusitis (Ch. 70)

Masquerades as tooth pain - NSAIDS and Tylenol CT +/- Antibiotics f/u with ENT More on this in future chapters

Indications for giving FFP: (Ch. 7)

Massive transfusion in trauma resuscitation Coagulopathy of trauma Hemorrhage in DIC or cirrhotics Plasma exchange in TTP Emergency reversal of warfarin (10-30ml/kg)

What is a collar-button abscess? (Ch. 50)

Matching swelling and erythema on both volar and dorsal aspects of the hand suggesting *deep space hand infection*

Describe your workup and treatment of orbital cellulitis. What are two complications?(Ch. 71)

Measure IOP(>20 needs surgery) Blood cultures CT to RO: FB, emphysema, hematoma, abscess, osteomyelitis, cavernous sinus thrombosis Consider LP Admit to hospital Tx: IV antibiotics(Pip-tazo IV plus vanco OR Cef 2g IV plus metronidazole) Complications: Meningitis, cavernous sinus thrombosis

List 3 complications of herpes infection (Ch 98)

Meningoencephalitis Hepatitis or Pneumonitis Disseminated infection

List 10 causes of status epilepticus(Ch. 102)

Metabolic - Hepatic encephalopathy - Hypocalcemia - Hypoglycemia/ hyperglycemia - Hyponatremia - Uremia Infectious process - CNA abscess - Encephalitis - Meningitis Withdrawl symptoms - Alcohol - Antiepileptic drugs - Baclofen - Barbituates - Benzo CNS lesions - Acute hydrocephalus - Anoxic or hypoxic insult - AVM malformations - Brain mets - CV accident - Chronic epilepsy - Eclampsia - Trauma - ICH - Neoplasm - Neurosurgery - Posterior reversible leukoencephalopathy - Remote structural injury Intoxication - Bupropion - Camphor - Clozapine - Cyclosporine - Flumazenil - FLoroquinolones - Imipenem - Isoniazid - Lead - Lidocaine - Lithium - MDMA - Metronidazole - Synthetic cannabinoids - Theophyline - TCA

Describe the mechanism of development of Hypochloremic Metabolic Alkalosis in vomiting? (Ch. 29)

Metabolic alkalosis: Due to loss of H and Cl ions in the vomit; Alkalosis further promoted by: Volume contractions, Increased aldosterone (loss of K and HCO3- in urine)

List causes of functional small bowel obstruction (Ch 92)

Metabolic disease (especially hypokalemia) Medications (eg. narcotics) Infection (retroperitoneal, pelvic, intrathoracic) Abdominal trauma Laparotomy

What is the treatment for methemoglobinemia? (Ch. 14)

Methylene blue (reduces iron ions back to Fe2+) Oxygen (100%)

What is the pathophysiology of target-organ damage, specifically microcirculation? (Ch 84)

Microcirculation Critical luminal narrowing - occlusion and ischemia "Silent" ischemic episodes lead to primary cause of chronic end organ damage Lumen thinning - aneurysm and rupture Results in tissue edema, fibrinoid necrosis, and microangiopathic hemolytic anemia

What is the toxic dose of ASA?(Ch. 149)

Mild-moderate = 150-300mg/kg Severe = >300mg/kg >500mg/kg associated with death

Outline the GOLD classification of COPD (mild, moderate, severe, very severe) (Rosen's box!)

Mild: - FEV1/FVC <70% - FEV1 >80% predicted - May be asymptomatic Moderate: - FEV1/FVC <70% - FEV1 between 50-80% predicted - Usually symptomatic with SOB on exertion Severe - FEV1/FVC <70% - FEV1 between 30%-50% - Increasingly symptomatic - Frequent exacerbations - Deleterious effects on quality of life Very Severe: - FEV1/FVC <70% - FEV1 less than 30% or - FEV1 less than 50% with chronic respirator failure (PaO2 less than 60mmHg or PaCO2 more than 50mmHg) - May or may not have signs of right heart failure

What is the treatment for periorbital cellultitis? (Ch. 71)

Mild: Clindamycin 300mg po q8h x 10 days OR Clavulin 875 po BID x 14 days + Septra DS TID x 10 days Mod-Severe OR <1 year old: Ceftriaxone + vancomycin or clindamycin

Tx approach to cardiogenic shock? (Ch. 6)

Minimize myocardial oxygen demand and Maximize cardiac output... 1. Optimize preload with fluid (MAP >65) 2. Gentle inotropy (dobutamine 2.5mcg/kg/min) 3. Vasoconstrict (norepi) 4. Intubate 5. Determine cause 6. Send pt off to get defect fixed

List the 4 types of immune reactions and give an example of each (Ch 119)

Mnemonic "ACID" Acute IgE (i.e. Anaphylaxis) Cell-cytotoxic (Blood transfusion reaction) Immune complex (SLE) Delayed (No antibodies involved - EM, SJS, TENS)

List 10 medical disorders that may cause psychosis (Ch 110)

Mnemonic: (Refer to Box 100.1 in Rosen's for 30+) Postpartum Sarcoid thYroid Calcium and Carbon (high) 02/hypOna/glycemia SLE Itis - encephalItis Substrate deficiency states (i.e. niacin, thiamine, Vit b12.)

Describe BP targets in hemorrhagic stroke and indications for surgical intervention

Neurosurgery consultation for patients with clinical or radiographic evidence of elevated ICP Treat blood pressure that is higher then 160-180 systolic or MAP higher then 130(Recommended agents include labetalol, esmolol, nicardipine, clevidipine, hydralazine

Define Dysthymic Disorder & it's treatment (Ch 111)

Mnemonic: HE'S 2 SAD Hopelessness Energy loss or fatigue Self-esteem is low 2 (see below) Sleep is increased or decreased Appetite is increased or decreased Decision-making or concentration is impaired To meet these criteria, adults need only 2 of the symptoms in addition to depressed mood during the initial 2 years and cannot be without symptoms >2 months at a time.

Describe the Jones Criteria for Acute Rheumatic Fever (Ch 83):

Mnemonic: JONES (Major) + CAFE PAL (Minor) J - Joint involvement which is usually migratory and inflammatory joint involvement that starts in the lower joints and ascends to upper joints O - ("O" Looks like heart shape) - indicating that patients can develop myocarditis or inflammation of the heart N - Nodules that are subcutaneous E - Erythema marginatum which is a rash of ring-like lesions that can start in the trunk or arms. When joined with other rings, it can create a snake-like appearance S - Sydenham chorea is a late feature which is characterized by jerky, uncontrollable, and purposeless movements resembling twitches Minor criteria include C - CRP Increased A - Arthralgia F - Fever E - Elevated ESR P - Prolonged PR Interval A - Anamesis L - Leukocytosis

Good intubation ventilation settings... (Ch. 2)

Mode: A/C Tidal Vol: 6-8 ml/kg Rate: 12-14 bpm Pressure targets: <30 cm H2O FiO2: Start @ 1 then DOWN to maintain SpO2>90% PEEP: 5 cm H2O

Key consideration for RH negative moms following any abdominal trauma: (Ch. 37)

Mom should receive Rhogam (RhIG) within 72 hrs of incident at a 300mcg dose.

What factors make genital injury in sexual assault more likely to be observed? Less likely? (Ch 67)

More - Concomitant non genital trauma - Penile penetration - Sexual inexperience of victim - Post menopausal victim - Anal contact - Stranger assault - Sexual dysfunction in the perpetrator - Foreign objects - SAV use of alcohol - SAV is a college graduate Less - Increased time from assault to hospital presentation - menstrual bleeding at time of assault

How is feto-maternal hemorrhage diagnosed and managed? (Ch. 37)

Most concern when > 12 weeks into pregnancy. ● Do Kleihauer-betke test for transplacental bleeding -Quantifies the amount of fetal-maternal hemorrhage (>5 mL) -8-30% incidence after trauma ●all Rh neg. mothers with any abd. trauma should receive RhIG within 72 hrs of the incident at a 300 mcg dose (protects against 30 ml of blood)** --Regardless of a negative KHB test!

What are the typical bacterial causes of deep space infections? What are the different syndromes called? (Ch 75)

Most frequently isolated organisms are streptococci, staphylococci, and Bacteroides species.

How do you treat NSAID overdose?(Ch. 149)

Mostly supportive Some evidence for hemodialysis in MASSIVE overdose

List the types and potential causes of large bowel obstruction (Ch 95)

Much less common > SBO; usually is a symptom of some underlying malignancy (>50% are due to colo-rectal CA) Types: Mechanical vs. pseudo-obstruction (Ogilvie's syndrome) Causes: Malignancy (53%) Adhesions Volvulus (17%) Diverticular disease (12%) Fecal impaction Strictures (due to IBD) Hernias Pseudoobstruction (**the only type managed non-surgically)

6 empirical criteria for diagnosis of circulatory shock: (Ch. 6)

Must have >4/6: -ill appearance or AMS -HR >100bpm -RR >20 or CO2 <32 -Base def of <-4 of lactate >4 -Urine output <0.5ml/kg/hr -arterial hypotention >30min continuous duration

What is your toxicologic differential for Hyperthermia/ diaphoresis(Ch. 147)

NASA - NMA, nicotine - Antihistamines, alcohol withdrawl - Salicylates, sympathomimetics, serotonin syndrome - Anticholinergics, antidepressants, antipsychotics

Provide a differential diagnosis for chronic arterial insufficiency. (Ch 87)

NB: Less than 5% of lower extremity ulcers are caused by arterial insufficiency Chronic venous insufficiency (much more common than arterial insufficiency) Neuropathic ulceration (from uncontrolled diabetes mellitus) Nocturnal muscle cramps Aortoiliac occlusive disease leading to atheromatous emboli (causing multiple ulcerations) Vasculitis Chronic subcutaneous intravenous drug use Malignant ulcers (look for thickened, rolled, elevated edges with a central depression containing granulation tissue) Osteoarthritis from associated joints Spinal stenosis Cauda equina syndrome (disc, cancer, infection) Hypertensive ulcers (lateral ankle - blue-red areas of infarcted skin)

Describe your management for Post extraction pain (Ch. 70)

NSAIDS Look for dry socket aka acute alveolar osteitis If present, nerve block, irrigation, daily packing changes Antibiotic use is controversial (Most dentists cover with pen or clinda or doxy) Refer to Dentist / Surgeon who preformed the procedure

In a patient with conjunctivitis, what causative organism should you never miss? What are some characteristics of this infection? What is the Tx? (Ch. 71)

Neisseria Gonorrhoeae - Rapidly progressing pain, copious discharge Needs IV abs, irrigation, topical abx and tx for chlamydia

What are the two most common bacterial agent implicated in septic arthritis in adolescents and young-adults? (Ch 116)

Neisseria gonorrhoeae, Chlamydia trachomatis

List 4 causes of oropharyngeal dysphagia: "immediate or transfer dysphagia" (Ch 89)

Neuromuscular - swallowing muscles don't work Due to a cerebrovascular accident - #1 cause! Degenerative aging: Alzheimer's, ALS, Diabetic neuropathy, muscular dystrophy, Parkinson's, Brain tumour Immunologic - poly/dermatomyositis / MS / Myasthenia Gravis / scleroderma Infectious - botulism, diphtheria, polioMyelitis, rabies, Sydenham's chorea, tetanus Metabolic - thyrotoxicosis, lead poisoning, Mg deficiency Obstructive Dysphagia lusoria

Indications for giving platelets: (Ch. 7)

Platelet count <10? - no hard 'n fast rule

List 6 factors that should alert to a medical cause of psychosis (Ch 110)

New onset of symptoms Acute change in mental status Recent fluctuation in behavioural symptoms Onset > 50 yrs old Onset AFTER admission to medical care setting Non-auditory hallucinations (visual) Lethargy Abnormal vital signs Poor performance on cognitive function testing - not oriented to person, place, time.

Is CTA presidentially useful in the ED diagnosis of MI (Ch 78)?

No (at least not at this time) - They (as well as the triple-rule out multi-detector CTA) are quite sensitive, but also can be non-specifically positive (as in the case of multiple calcifications / perfusion defects in someone known to have symptomatic CAD).

Contraindications to procedural sedation? (Ch. 4)

No absolute contraindications. *Bottom line*: it's a risk, use clinical judgement

List 4 options for treatment of corneal abrasions (CH. 71)

No contact lens use until healed! No eye patches Cycloplegic prn topical antibiotics --> probably only if contact lens wearer. Tobramycin 0.5% 1-2 drops q4h Topical analgesics: Ketorolac 0.5% QID Tetanus only if dirt and organic matter in eye

List 5 patient types of intimate partner violence exposure and corresponding appropriate interventions (Ch 68)

No history of suspicion of IPV Prior history but no current exposure(Add history of PIv to medical record) Recent or current abuse but no injuries and no elements of danger on assessment (Add IPV to problem list and give educational materials) Current abuse with injuries on findings on danger assessment(Complete history, involve third parties as required (police, social services, IPV advocate)) Suspicion but denies IPV (Consider involving SS or IPV advocate and give educational materials)

What are some clinical signs of pharyngoesophageal injury? (Ch. 44)

No pathognomonic signs, but these give a hint: -Hematemesis / blood in saliva/NGT -Odynophagia / dysphagia -Subcutaneous Emphysema -Dyspnea / hoarseness / stridor / cough -Pain / neck tenderness / resistance to neck movement

The synovial fluid is clear/yellow, with a thick viscosity in this type of arthritis (Ch 116)

Non-inflammatory

Define "nonjudicial hanging" and expected injury pattern: (Ch. 44)

Non-judicial hangings: -usually lead to venous stasis and congested blood flow → loss of consciousness and body relaxation When the person is limp the ligature / force can tighten further leading to complete arterial occlusion and brain death Vagal pressures on the carotid body may also produce fatal dysrhythmias ***Vascular occlusion leads to death, rather than airway occlusion in most cases!***

Define uncomplicated UTI and antibiotic options (Ch 99)

Nonpregnant patient, with a structurally and functionally normal urinary tract Antibiotics: Septra 160/800 mg BID for three days Nitrofurantoin 100 mg BID for 5 days Fosfomycin 3 g po x 1 dose Avoiding agents with >20% local resistance to antibiotics If you're unsure if a patient has a UTI or an STI you can treat both with: Levofloxacin 500 mg po daily for 7 days Ceftriaxone 250 mg IM x 1

List 6 predisposing conditions for stress fractures (Ch 58)

Note: Most due to running: Risk factors: new footwear, change in running surface, overuse/overtraining, female athlete triad/relative energy deficit syndrome, steroid use, vitamin / mineral deficiency, cavus feet

How are teeth traditionally numbered?(Ch. 70)

Numbered 1-32, with 1 starting upper right third molar, left upper 3rd molar is 16, eventually ending at lower right third molar number 32

Define OCD (Ch 112)

OCD is characterized by recurrent, obtrusive, unwanted thoughts (obsessions), such as fears of contamination, or compulsive behaviors or mental acts (compulsions) that a person feels compelled to perform, such as hand-washing or counting

What are some complications of immobilization post-fracture especially in the elderly? (Ch. 49)

Pneumonia DVT / thrombophlebitis PE UTI Atrophy Stress ulcers GI bleed

Describe the SAD PERSONS Scale (Ch 115)

One point for each "yes" answer: S: Male sex A: Age (<19 or >45 years) D: Depression P: Previous attempt E: Excess alcohol or substance use R: Rational thinking loss S: Social supports lacking O: Organized plan N: No spouse S: Sickness This score is then mapped onto a risk assessment scale as follows: 0-4: Low, 5-6: Medium, 7-10: High *** Although the original study in 1996 found this to be a SPECIFIC tool, it is not SENSITIVE enough to be clinically useful. In short, it's useless***

What's another name for tinea unguium? What is it's treatment? (Ch 120)

Onychomycosis. Involvement of one or two nails may be treated with topical antifungal agents. More extensive infection requires systemic therapy with an antifungal agent, such as terbinafine or itraconazole.

Differentiate between herpes zoster opthalmicus and herpes zoster oticus(Ch. 105)

Opthalmicus = trigeminal nerve - Hutchinson's sign: Cutaneous involvement of nasociliary nerve - Look for ocular involvement = punctate epithelial erosions and pseudodendrites Oticus = ramsay hunt syndrome - unilateral facial paralysis(precedes rash) - Herpetiform vesicular eruption: Pinna, external auditory canal, tympanic membrane, soft palate, oral cavity, face and neck - Vesticulocochlear dysfunction

Describe the management of traumatic hyphema. What are the risks associated with a hyphema?(Ch. 71)

Optho consult! Topical beta blocker, topical alpha agonist / carbonic anhydrase inhibitor, acetozolamide or IV mannitol. +/- cycloplegics and steroids Needs admission if: >50%, decreased VA, increased IoP, sickle cell disease(high risk for increased IOP) Risks: Rebleeding in 2-5 days, corneal blood staining, glaucoma, synechia formation

How can you differentiate orbital and periorbital cellulitis? (Ch 71)

Orbital: Fever, toxic appearence, blurred vision, proptosis, painful or limited EOM, binocular diplopia, edema of optic disk, venous engorgement of the retina. Periorbital will NOT have the above

List some of the extraarticular manifestations of RA (Ch 116)

Osteoporosis, atlantoaxial subluxation Muscle weakness, synovitis, myositis, myopathies Sarcopenia, obesity Rheumatoid nodules** ocular/oral dryness (sjogren's syndrome)* Episcleritis, scleritis, uveitis Interstitial fibrosis*, pneumonias, pulmonary hemorrhage Pericarditis, myocarditis, CAD Mesenteric vasculitis Glomerulonephritis Carpal tunnel syndrome; mononeuritis multiplex Anemia*

List 4 universal antitdotes (Ch. 147)

Oxygen- Hypoxia or carbon monoxide Dextrose - insulin, oral hypoglycemic agents Narcan - narcotics Thiamine - wernicke's / chronic ETOH abuse Others N- acetylcysteine - Acetominophen Fomepizole- Methanol/ ethylene glycol Physostigmine - Anticholinergics Atropine/ pralidoxime - Organophosphates Methylene blue - Mathemoglobinemia Nitrites/ hydroxycobalamin - cyanide Deferoxamine - Iron Dimercaprol - Arsenic, lead Succimer - lead, mercury CaEDTA - Lead Glucagon - B-blockers Sodium bicarb - Salicylates, tca Calcium, insulin/ glucose - Calcium channel antagonists Dextrose, glucogon, ocreotide, pyridoxine - Oral hypoglycemic agents, isoniazid IV fat emulsion - Lidocaine toxicity

What is your toxicologic differential for bradycardia and hypotension(Ch. 147)

PACED - Propranolol, poppies, physostigmine - Anticholinesterase drugs, antiarrythmics - Clinidine, CCB - ETOH and other alcohols - Digoxin, digitalis

Describe the classification of Abnormal Uterine Bleeding (Ch 100)

PALM - Structural Causes Polyp Adenomyosis Leiomyoma (submucosal or other) Malignany and hyperplasia COEIN - Non-structural Coagulopathy Ovulatory Endometrial Iatrogenic Not yet classified

Causes of *obstructive shock*: (Ch. 6)

PE Cardiac tamponade Tension pneumo Valvular dysfunction (AS, prosth valv thrombosis) Congenital heart defects (PDA closure) HOCM

What do common antipyretics act on? (Ch. 12)

PGE2

Difference between PaO2 and PAO2: (Ch. 14)

PaO2: partial pressure of oxygen in arterial blood measured by ABG PAO2: partial pressure of oxygen in the alveoli

What does magnet application do to a pacemaker? to an ICD? (Ch 80)

Pacemaker: Converts pacemaker into asynchronous pacing mode. No longer inhibited by intrinsic electrical activity ICD: Deactivates the defibrillator (note should only be done during resuscitation efforts, comfort care or inappropriate delivery of shocks eg in SVTs)

How is aortic dissection diagnosed? (Ch 85)

Pain Pain Pearl #1: Ask the following 3 questions for all patients with torso pain Usually sudden in onset Quality of pain (most commonly "sharp" but highest LR for "tearing") Pain intensity at onset Radiation of pain (back and/or belly) Pain Pearl #2: Think of aortic dissection as the subarachnoid hemorrhage of the torso -> Think thunderclap pain for chest / abdo Pain Pearl #3: Severe colicky chest pain + opioids = heightened suspicion (tearing / ripping) Pain Pearl #4: Migrating pain has a +LR = 7.6 Pain Pearl #5: The pain can be intermittent as dissection of the aortic intima stops and starts Painless aortic dissection is a thing (about 5%) and common to have tamponade in this group

What is complex regional pain syndrome (CRPS)? (Ch. 49)

Pain syndrome that develops after a noxious event and extends beyond a single peripheral nerve and is disproportionate to the inciting event

List the differential diagnosis of painful and painless genital lesions (Ch 98)

Painful: Genital herpes Chancroid Behcet's syndrome Painless Syphilis Lymphogranuloma venereum Granuloma inguinale

List intra-abdominal sites of injury that may be missed on CT: (Ch. 46)

Pancreas** Diaphragm Small bowel Mesentery

List and describe 2 formulas for fluid rescucitation(Ch. 63)

Parkland formula: RL 4cc/kg/%TBSA burned - Half of the fluids in first 8 hours, remainder in next 16. Over next 24 hours 20-60% of patient plasma volume, titrating to maintain urine outpt at 0.5-1cc/kg/hr(1 in kids) Galveston formula - Preferred for peds - RL at 5000 cc/m2 TBSA burned + 2000 cc/M2 BSA - Half fluid in first 8, remainder in next 16

What is a partial seizure? What are the subtypes. What is a generalized seizure? What are the subtypes. (Ch. 102)

Partial seizure: involve abnormal neuronal firing within a confined population of neurons in one brain hemisphere Types: motor, autonomic, somatosensory, psychic Generalized: Abnormal neuronal firing throughout both brain hemispheres. Always involves alterations in conscioussness Types: Absence, tonic, clonic, tonic-clonic, myoclonic, atonic

Describe on tool for partner violence screening (Ch 68)

Partner violence screen (Box 68-2) Have you been hit, kicked, punched or otherwise hurt by someone wthinin the past year? If yes, by whome? Do you feel safe in your current relationship? Is there a partner from a previous relationship who is making you feel unsafe now?

What are some treatment options for condylomata acuminata (genital warts)? (Ch 98)

Patient applied: Imiquimod cream Podofilox solution or gel Sinecatechins ointment Provider administered: Surgical excision Cryotherapy Topical therapy with trichloroacetic acid (TCA) or bichloroacetic acid (BCA)

Which patient groups should receive pneumovax? (Ch 76)

Patients with: Diabetes Alcoholism Cardiovascular disease Sickle cell disease Splenectomy Malignancy Immunosuppression Older than 65 yrs

Mechanisms of posterior urethral injuries: (Ch. 47)

Pelvic fracture involving ischiopubic rami

Minimum personnel, supplies, and monitoring for procedural sedation: (Ch. 4)

Personnel: ERP & Nurse/RT Supplies for: allergic reaction, oversedation, resp depression, arrest, airway emergency Monitoring: direct visualization, vitals, EtCO2,

List causes of red-coloured urine without hematuria (Ch 99)

Phenazopyridine Nitrofurantoin Rifampin Chloroquine Hydroxychloroquine Iodine Bromide Food coloring Beets Berries Rhubarb

What are the six types of elder abuse? What is the most common? (Ch 69)

Physical abuse Sexual abuse Emotional/ psychological abuse Neglect - Most common! Abandonment Financial or material exploitation

What are the risk factors for aortic dissection? (Ch 85)

Physiologic HTN Anatomic Hx Cardiac Surgery Bicuspid aortic valve Vasculitis / Connective Tissue disorder Medium to large vessel vasculitis (Takayasu's arteritis, giant cell arteritis, and Behçet's disease) Marfan Syndrome Loeys-Dietz (connective tissue disorder phenotypically similar to Marfan's) Other Stimulant use Cardiac cath / Intra-aortic balloon insertion Blunt Trauma

Describe phenotypic characteristics of Pit-Vipers(aka rattle snakes)(Ch. 55)

Pit midway between eyes and nostrils Elliptical pupil Tail structure of single rows of subcaudal plates, triangle head

What is the classic visual defect associated with pre-chiasmal, chiasmal and post chiasmal lesions? What is your differential dignosis for each lesion? (Ch. 71)

Prechiasmal: Decreased visual field loss on affected side. Usually due to optic neuritis, ischemic optic neuritis, compressive optic neuritis, toxic/ metabolic Chiasmal: Bitemporal hemianopsia. Pituitary tumor or meningioma Postchiasmal: Left lesion = right bilateral visual field blindness. Infarction, tumor, avm or migraine disorder

List six strong predisposing or precipitating factors for delirium(Ch. 104)

Predisposing: Age, dementia, hypetension Precipitating: Coma, previous delirium, emergency surgery, mechanical vent, trauma

General approach to multi-trauma patients: (Ch. 36)

Prepare your team, assign roles Listen to story, vitals en route, vitals now *Primary Survey* (w/ appropriate interventions) -Airway - patent? FBs? ventilating? (intubate?) -Breathing - WOB? trachea midline? chest sounds? subcut air? (chest tube?) -Circulation - deadly bleeding? monitor? IV? abdo pain? (fluids, blood, TXA, thoracotomy?) -Disability - GCS, pupils, power movement x4 (collar?) -Expose patient/Environmental (logroll, tx hypothermia?) *Secondary Survey* -head to toe assessment, log roll if not done yet, FAST -AMPLE hx, tetanus? *Investigations* -CT panscan vs CT focused, X-ray of focal injuries/pain (chest, limbs, pelvis). -trauma labs (include lactate, INR, fibrinogen, G&S, BHCG) *Drugs* -pain, nausea, pressor support, abx, fluids, steroids? *Disposition* -admit, transfer, discharge?

What are the clinical presentations and radiographic findings of an ORBITAL BLOWOUT fracture? (Ch. 42)

Presentation: --diplopia with up-wards gaze (from inf rectus entrapment) --exophthalmos (from retrobulbar hematoma) --parasthesias/anaesthesia of medial cheek (stretched V2) On CT: --visible orbital floor fracture --orbital contents spilling into maxillary sinus

Clinical presentation and management of a sternal #: (Ch. 45)

Presentation: Anterior chest pain - point tenderness (impact mechanism), swelling, bruising, deformity/step visible Management: analgesia, CT (to r/o mediastinal injury), d/c if otherwise well, surgery if severely displaced.

Outline the management for a pt with ischemic stroke who is not eligible to receive thrombolysis(CH. 101)

Prevent secondary neurologic injury: Optimal fluid and electrolyte balance Avoid hypo or hyperglycemia Prevent fever Start an antiplatelet (or add clopidogrel on to ASA) within 48 hrs Consider starting LMWH in hospital after consultation with neurology Start aggressive statin therapy treating high blood pressure only if the hypertension is extreme (systolic blood pressure >220 mmHg or diastolic blood pressure >120 mmHg) Prevent aspiration, early mobilization and physiotherapy and multidisciplinary stroke unit care

What kind of cervical spinal injuries can you get from SHEAR (A/P) forces? And are they stable/unstable? (Ch. 43)

Primarily odontoid fractures (Type I - III)

List the causes of dementia(Ch. 104)

Primary - Alzheimers - Lewy body - Picks disease(Frontal lobe) Subcortical - Parkinsons - Huntingtons Vascular Intracranial prcoess - Space occupying lesion - Hydrocephalus - CNS infections - repetitive head trauma Endocrinopathies - Addisons and cushings disease - Thyroid and parathyroid disease Nutritional deficiencies - Thiamine - Niacin - Folate - Vit B12 Toxic exposure - Heavy metals - CO - Carbon disulfide Drugs - Psychtropics - ANtihypertensives - Anticonvulsanta - ANticholinergics Depression: Pseudodementia

List the indications for a lateral canthotomy. What is a contraindication (Ch. 71)

Primary Decreased visual acuity Intraocular pressure >40 mmHg Proptosis Secondary Afferent pupillary defect Cherry red macula Opthalmoplegia Nerve head pallor Eye pain Contraindication: GLOBE RUPTURE

What is the difference between a primary and secondary skin lesion? (Ch 120)

Primary results directly from the disease process. Secondary lesions result from secondary factors: scratching, healing, infections.

Classify headache based on the IHSC (international headache society classification)

Primary: - Migraine - Cluster - Tension Secondary (secondary to identifiable, distinct organic pathologic process) - H/A attributed to trauma or injury to the head or neck - Cranial or cervical vascular disorder - Nonvascular intracranial disorder - A substance or its withdrawal - Infection - Disorder of homeostasis - Headache or facial pain attributed to disorder of cranium, neck, - Eyes, ears, nose, sinuses, teeth, mouth, or other facial or cranial structures Others: - cranial neuropathies - other facial pains - other headaches

What is the most common pathophysiologic cause of PTX? (Ch 77):

Primary: Even though this is technically in a person without lung disease, the most common cause is unrecognised pleural disease and rupture of a bleb Secondary: Easy, COPD - accounts for 70% of cases Severe COPD is at highest risk (FEV1 <1L). Rupture of atypical blebs is typical cause

How long is driving licence suspended after a seizure? (Ch. 18)

Private drivers: 6 mo seizure free Professional drivers: 12 mo seizure free

When is loperamide indicated? (Ch. 31)

Probably OK if non-bloody and afebrile, but be wary ** NOT indicated in pediatrics due to risk of toxic megacolon

What is "electric alternans"? (Ch. 45)

Produced as heart swings in *massive pericardial effusion* - QRS axis changes from swing to swing

Describe management of suspected DVT (Ch 88)

Radiographic eval: U/S = 95% sens & spec. For proximal DVT Using the standard 3 point compression tests The standard scanning protocol misses the calf veins and superficial veins (which can progress to a DVT) For low risk groups with a negative 3-point U/S after a +ve d-dimer you can stop testing For moderate-high risk groups, a single negative 3 point venous U/S is NOT sufficient to exclude a DVT, they should have a follow-up U/S in 2-7 days to exclude a DVT If you perform a single FULL leg doppler U/S (whole leg including superficial veins) - you can rule out a DVT in any of the risk groups (high/med/low) If you're trying to diagnose an iliac or pelvic vein DVT - you need to use CT-venography

What is traumatic asphyxia? (Ch. 45)

Rare problem due to marked massive influx of blood to the head and neck due to sudden thoracic compression, causing retrograde blood flow from the right heart backwards into the SVC

What conditions may mimic child abuse(Rash, trauma, Infections, deficiencies) (Ch 66)

Rashes: Phytophotodermatitis, dermal melanocytosis, bullous impetigo(Looks like burn) Trauma: Unintentional burns, accidental fractures, Osteogenesis imperfecta Deficiencies: Rickets of prematurity, scurvy Infections: Congenitial syphilis/ rubella, encephalitis/meningitis/post hypoxis edema

List three disorders characterized by abnormal vasomotor response. (Ch 87)

Raynaud's disease See next question! Raynaud's phenomenon Raynaud's disease, when there is a secondary cause: Connective tissue disease, scleroderma, RA, SLE Treatment = the underlying disease. Benign Livedo reticularis Similar to Raynaud's disease, but affecting the dermal arterioles of the extremities and trunk. Acrocyanosis Persistent, painless, symmetrical cyanosis of the fingers, hands, feet. Benign disease, unlike central cyanosis. Supportive care: keep the body warm! Primary erythromelalgia Can occur in a primary or secondary form (SLE, HTN, diabetes, PVD) Paroxysmal vasodilation with burning pain, increased skin temp, and redness to feet and hands. Attacks occur in modest ambient temperatures. No tissue loss occurs, but attacks can be disabling . No definitive treatment - supportive ice/cold baths, elevation, rest.

3 major systemic effects of septic shock: (Ch. 6)

Relative hypovolemia Cardiovalscular depression Induction of systemic inflammation (-> ARDS, AKI, MODS)

Describe the emergent/urgent ED management of AECOPD.

Remember: Beta agonists, anticholinergics and corticosteroids are our workhorses for AECOPD. A - if impending airway secondary to ALOC from pCO2 rise, clinical course from work of breathing stand point, etc. Consider securing. - better evidence for using bipap in AECOPD patients (if no contraindications) B - titrating to sats of 90%. - bronchiodilators and anticholinergics. Better evidence and delivery with MDIs if patient will tolerate and follow instructions. - corticosterioids - if can manage PO, then can trial PO sterioids. C - may need volume if they've been working hard Things with little or no evidence: neb saline. Chest physio. Heliox. Oral expectorants. 2004 Cochrane Review on NIPPV https://www.ncbi.nlm.nih.gov/pubmed/14974057 Mortality NNT 10 Decreased Intubations NNT 5 Decreased Treatment Failure NNT 4

Canadian CT Head Rule: (Ch. 41)

Remember: this is for whether or not to image someone with a MINOR head injury (GCS 13-15) after witnessed LOC, amnesia, or confusion.

List 4 lesions with a positive Nikolsky sign (Ch 120)

Reminder: this sign is positive if with gentle rubbing the skin sloughs off (the top layer of the epidermis). SJS TENS Pemphigus Vulgaris staphylococcal scalded skin syndrome (+ small set of patients with bullous pemphigoid)

Describe the risk factors for Adnexal torsion (Ch 100)

Reproductive age 2o to regular development of corpus luteal cysts during menstrual cycle Relative adnexal mobility: seen in premenarchal patients Enlarged ovary (>5.0 cm) Benign neoplasm or cysts, Recent Infertility Treatment (ovulation induction & hyperstimulation syndrome) polycystic ovarian syndrome Pregnancy: Torsion rare but possible in 1st and early 2nd trimesters Hx/ tubal ligation

"Critical" (MUST NOT MISS) *infectious* diagnoses for respiratory, gastrointestinal, neurologic, and systemic systems: (Ch. 12)

Resp: bacterial pneumonia (with resp failure) GI: peritonitis Neurologic: meningitis, cavernous sinus, thrombosis Skin: necrotizing fasciitis Systemic: sepsis or septic shock, meningococcemia

"Emergent" *infectious* diagnoses per system: (Ch. 12)

Resp: peritonsillar/retropharyngeal abscess, epiglottitis CVS: endocarditis, pericarditis GI: appendicitis, cholecystits, diverticulitis, abscess GU: pyelo, PID, tubo-ovarian abscess Neuro: encephalitis, brain abscess, epidural abscess Skin: cellulitis, infected decubitus ulcer, soft-tissue abscess

Describe 4 mechanisms of diarrhea: (Ch. 31)

Rule of "Number 2": 2 fast - Hypermotility (IBS, hyperthyroid, heroin w/d) 2 strong - Osmotic (laxatives, steatorrhea) 2 broken - Inflammatory (chemo, radiation, infection) 2 confused - Secretory (cholera)

Define complicated UTI and the populations this affects (Ch 99)

S, P, E, C, I , A , L Systemic disease Pregnancy Elderly Compromised renal function or immune function Instrumentation Anatomy Long urethra (males)

What is the blood supply of the SA and AV nodes? (Ch 79)

SA node: 55% of people: blood flow from RCA 45% ppl = Left. circumflex AV node: 90% of ppl = blood flow from RCA (known as RIGHT DOMINANT)

What is your toxicologic differential for mydriasis(Ch. 147)

SAW - Sympathomimetics - Anticholinergics - Withdrawl syndromes

How does treatment of diskitis differ from epidural abscess? (Ch. 106)

SEA: Surgical consultation, antibiotics Diskitis: Broad spectrum abx, NO SURGERY

Most common causes of pediatric cardiac arrest: (Ch. 10)

SIDS (25%) Traumatic (20%) Drowning (15%)

Define sepsis: (Ch. 6)

SIRS + documented or suspected infection

Describe management of superficial thrombophlebitis + isolated calf thrombosis (Ch 88)

SLT: These patients have a clot in the greater saphenous vein that may extend above the knee. This can propagate proximally Treatment: NSAIDs for symptoms Heat Graded compression stockings **mandatory repeat ultrasound in 2-5 days** If it has migrated proximally most people at least treat these patients with LMWH or fondaparinux for 10 days, followed by repeat ultrasound ICT: Controversial! 25% risk of proximal propagation proximally High risk symptomatic patients - can consider repeat duplex U/S in 2-5 days OR anticoagulation based on bleeding / clotting risk For healthy ambulatory patients - Rosen's recommends: 325 mg ASA daily and repeat U/S at 2-5 days to look for clot propagation

What are some of the manifestations of SLE? (Ch 118). Hint: There's a popular medical school mnemonic on this one:

SOAP BRAIN MD Serositis Oral Ulcers Arthritis Photosensitivity & Pulmonary Fibrosis Blood cells (pancytopenia) Renal, Raynauds ANA Immunologic (anti-Sm, anti-dsDNA) Neuropsych Malar Rash Discoid Rash

12 causes of hemoptysis: (Ch. 24)

SPITS *Structural* --neoplasm, trauma, foreign body *Pulmonary* --bronchitis, bronchiectasis, TB, pneumonia, abscess *Iatrogenic* --post-lung core biopsy *Thrombosis* --PE, coagulopathy from cirrhosis/warfarin, DIC *Systemic* --congenital heart disease (kids) --valvular heart disease --Goodpasture's syndrome, vsculitis, SLE

List 8 dialyzable drugs(Ch. 147)

STUMBLED - Salicylates - Theophylline - Uremia - Metformin/ methanol - Barbituates - Lithium - Ethylene glycol - Depakote - valproic acid

Difference between SaO2 and SpO2: (Ch. 14)

SaO2: saturation of oxygen in arterial blood measured/calculated by ABG SpO2: saturation of oxygen in peripheral blood measured by pulse oximetry

Features of Cauda Equina? (Ch. 43)

Saddle anaesthesia Lower extremity weakness (bilateral) loss of anal sphincter tone incontinence of stool/retention of urine

Describe the diagnosis and management of scabies (Ch 98)

Scabies: Sarcoptes scabiei is the mite responsible for scabies. Diagnosis - visual inspection often reveals characteristic burrows in the skin. Excoriations, papules, and nodules are frequently seen in the groin, genitalia, axilla, and interdigital web spaces. Diagnosis may be confirmed by microscopic examination of scrapings from characteristic skin lesions, which reveals the mites. Management - topical permethrin 5% cream applied topically and washed off after 8-14 hours. Alternative agents include topical benzyl benzoate, topical lindane, or oral ivermectin.

Describe the DSM V diagnostic criteria for schizophrenia (Ch 110)

Schizophrenia has some core psychopathological features: Positive symptoms: Hallucinations Delusions Fixed, false beliefs that persist in the face of overwhelming contradictory evidence. can be bizarre and clearly implausible, or they can be reasonable and understandable yet untrue. Disorganization disorganization of behavior and thinking tangentiality and circumstantiality -ve symptoms Cognitive impairment Difficulties with attention, memory, reasoning, verbal comprehension, and decision-making Negative symptoms Blunted affect, emotional withdrawal, social withdrawal, poor rapport with other people, difficulty with abstract thinking, loss of spontaneous conversation, and stereotyped thinking.

List 6 stigmata of chronic liver disease (Ch 90)

Scleral / cutaneous icterus +/- pruritus Hepatomegaly Spider angiomata Caput medusa Patchy ecchymosis with thin skin Splenomegaly Gray / acholic stools Gynecomastia Muscle wasting Palmar erythema Dupuytren's contractures Testicular atrophy

What is Elson's test? (Ch. 50)

Screening test for central slip injury (injury to the central tendon over the PIP - causing bouttoniere deformity) https://www.youtube.com/watch?v=G9HY0qXWUvE

Differentiate between the two types of 2° heart block with respect to etiology, ECG appearance and management (Ch 79):

Second degree = one or more impulses don't reach the ventricle. Type I wenckebach or mobitz 1 progressive lengthening of PR until a beat is dropped "grouped beats" increased vagal tone conditions Type II block BELOW the AV node atropine WORSENS*** this block

In addition to hand foot mouth disease (HFMD), what other infectious rashes can manifest on the palms and soles? (Ch 120)

Secondary syphilis RMSF Kawasaki syndrome Aureus endocarditis Measles Dengue Acute meningococcemia

What is "sedation"? (Ch. 4)

Sedation= Controlled reduction of environmental awareness

Define severe sepsis: (Ch. 6)

Sepsis + 1or more signs of organ dysfunction (high lactate, new Cr elevation, low urine output)

The synovial fluid gram-stain is positive in this type of arthritis (Ch 116)

Septic

5 main subtypes of *distributive shock*

Septic Anaphylactic Central neurogenic shock Drug OD Adrenal crisis

When should stool cultures be sent for a patient? (Ch 94)

Severe illness, Fever of 38.5° C (101°F) or higher, Dysentery (mucous, bloody, tenesmotic), Persistent diarrhea for 14 days or longer (send for O+P) Patients who are immunocompromised Recently hospitalized / recent antibiotics.

Define septic shock: (Ch. 6)

Severe sepsis + tissue hypoperfusion DESPITE adequate fluid challenge (~2L)

When should O2 be used in neonatal resuscitation? (Ch. 11)

Should be avoided - damaging effects Resuscitate with ROOM AIR SpO2 goal after 10 min is >80%

List 10 clinical symptoms of MS and describe its diagnosis. Describe the typical CSF findings in a patient with MS. (Ch. 105)

Should be suspected in patients who present with episodic neurologic dysfunction that evolves over days and resolves over weeks - Optic neuritis(Painless vision loss) - Long tract symptoms(Spasticity, hyperreflexia and abormal reflexes such as babinski or hoffmans) - Brainstem syndrome - SPinal cord syndrome(ie transverse myelitis) - Uhthoff phenomenom Most common in sensory in limbs, then visual loss, then subacute motor CSF - Pleocytosis - 5 or more lymphocytes - Increased IgG - Increased oligocloal bands of IgG to 85-95%

What are some of the common sites for tendonitis? (Ch 117)

Shoulder (Supraspinatus) Biceps (Long head of biceps) Elbow (Extensor tendons i.e. tennis elbow) Ankle (Achilles) Wrist (De quervian's, or flexor carpi ulnaris / other flexors).

List underlying etiologies of sick sinus syndrome (Ch 79)

Sick sinus syndrome: Umbrella term for a group of dysrhythmias from disease of the sinus node and its surrounding tissues: Sinus bradycardia, sinus arrest, and SA exit block. A variant of SSS: bradycardia-tachycardia syndrome: these bradydysrhythmias alternating with a tachydysrhythmia.

Define the sedative/ hypnotic toxidrome. what physical exam findings would you see. What substances give you this presentation? What is the treatment? (Ch. 147)

Signs - Depressed heart rate, BP, RR - Cold - No pupil change - No bowel sounds - Dry Substances - Benzo - Barbituate - Carisoprodol - Meprobamate - Gluthemide - Alcohol - Zolpidem Tx - Supportive, RO trauma

Define the Opiate toxidrome. what physical exam findings would you see. What substances give you this presentation? What is the treatment? (Ch. 147)

Signs - Depressed heart rate, BP, RR - Cold - Pinpoint - No bowel sounds - Dry Substances - Opioids --> heroin, morphine, methadone, oxycodone, hydromorphone - Diphenoxylate Tx - Narcan!!!! - Supportive

Define the Hallucinegenic toxidrome. what physical exam findings would you see. What substances give you this presentation? What is the treatment? (Ch. 147)

Signs - Hallucinations, perceptual distortions, depersonalization, synesthesia, agitation - Mydriasis - Hyperthermia - Tachycardia - Hypertension - Tachypnea - Nystagmus Substances - PCP - LSD - Mescaline - Psilocybin - Designer amphetamines(MDMA, MDEA) Tx- Supportive

Define the sympathomimetic toxidrome. what physical exam findings would you see. What substances give you this presentation? What is the treatment? (Ch. 147)

Signs - Increased HR, RR Temp - Dilated pupils - Diaphoretic Substances - MDMA - Cocaine - Amphetamine - Cathiones - Ephedrine - Pseudoephedrine - Phenylpropranolamine - Theophylline - caffeine Tx - Benzo - Fluids - Supportive

Describe the two characteristics that are needed to diagnose otitis media. (Ch. 72)

Signs and symptoms of middle ear inflammation (think: bulging of tympanic membrane, erythema or otalgia, fever) Middle ear effusion (TM opacity, lack of tm mobility, an air-fluid level, or otorrhea)

Describe diagnosis and treatment for orbital floor fractures(Ch. 71)

Signs: Enopthalmus, ptosis, diplopia, anesthesia of cheek and upper lip, limitation of upward gaze Dx: Ct orbits Tx: Dont sneeze or blow nose, watch for signs of infection and double vision. DC home IF no globe rupture or visua impairment

What are the signs and symptoms of nonarteritis ischemic optic neuropathy? How is it different from arteritic? What is the treatment? (Ch. 71)

Signs: Painless visual loss Symptoms: RAPD, disc swelling Difference: No painful symptoms of TA, normal ESR Treatment: Steroids(Mixed results), improvement in 1/3 of patients

Who gets which antibiotics for COPD? (i.e. 2008 CTS Guidelines)

Simple exacerbation - Increased sputum and dyspnea - which are usually 2ndary to H.influenza, Moraxella, or Strep pneumo - Get amox or 2nd/3rd gen cephalosporin, or extended spectrum macrolide or doxycycline Complicated exacerbation - defined as COPD with risk factors that include --> FEV1 <50% predicted --> equal to or more than 4 exacerbations per year --> ischemic heart disease --> home O2 --> chronic oral steroids - these patients may have Pseudomonas or beta lactam resistance. They should get a respiratory fluroquinolone as first line (like levofloxacin)

What are the pathologic and clinical differences between UC and Crohn's? (Ch 95)

Simplified Summary Chart:

What is it important to know if the platysma muscle is damaged in neck trauma? (Ch. 44)

Since it is sandwiched between superficial and deep fascial layers, it's violation suggests deeper structures could also be damaged.

Describe the management of 1° and 2° spontaneous PTX (Ch 77):

Small Primary (<20%): Can elect for observation only. No need for admission if normal oxygenation and hemodynamics. Normal reabsorption rate is 1-2% of PTX air per day. Can increase this by factor of 4 with the administration of 100% oxygen and subsequent nitrogen washout. In a nutshell, this is because: lower alveolar partial pressure of nitrogen with oxygen administration increases the rate at which air diffuses across the pleural-alveolar barrier F/U ED or GP in 24-48hrs. Avoid air travel and underwater diving / scuba diving until PTX has fully resolved. LARGE primary or any secondary (>20%) Catheter insertion and aspiration of PTX. If no re-accumulation on CXR after 6 hrs, d/c home and same caveats as above. (not likely to work if age >50 or more than 2.5 L aspirated)

What size of vessel vasculitis do both Goodpasture's and Behcet's disease fall-under? (Ch 118)

Small-vessel

Tell-tale physcial signs of scaphoid fractures: (Ch. 51)

Snuffbox tenderness and swelling Limited ROM of wrist and thumb Dorsal wrist pain, distal to radius

What radiologic finding is common to find on x-ray in an acute arthritis (gout, pseudogout, septic arthritis)? (Ch 116)

Soft-tissue swelling

What are some expected seatbelt-related injuries? (Ch. 46)

Solid organ hematoma or rupture Bowel perforation Mesenteric shearing / avulsion Diaphragmatic herniation

What are the bones of the wrist? (Ch. 51)

Some Lovers Try Positions That They Can't Handle. (Scaphoid, lunate, triquetrum, pisiform, trapezium, trapezoid, capitate, hamate)

What is a non-occlusive vascular trauma? (Ch. 48)

Some residual flow through the vascular injury is preserved

Causes of positive relative afferent pupillary defect (RAPD)? (Ch. 21)

Somehow light isn't getting *to or through* the optic nerve on the effected side... --vitreous hemorrhage --loss of retinal surface --ischemia or retinal detatchment --optic neuritis (in MS)

Describe potential sources of Anhydrous ammonia and how to manage an exposure(Ch. 64)

Source: Fertilizer, some methamphetamine production Tx: irrigation of eyes, skin with water Secure airway if necessary with large bore tube

Describe potential sources of Hydroflouric acid and how to manage exposure(Ch. 64)

Sources: Glass etching, microelectronic production, rust remover, aluminum cleaner, cement and brick cleaner Management - Irrigation for 15-30 minutes - Blister removal - Local calcium gluconate gel - Calcium gluconate IV/Intraarterial or subq - Calcium gluconate infusion in hospital

What are risk factors for infection with animal bites?(Ch. 54)

Species - Cat - Human - Primate - Pig - Camel Location of wound - Hand - Joint or superficial tendon - Through and through oral - Below the knee Wound type - Puncture - Extensive tissue damage - Contaminated or devitalized tissue - Old - Sutures High risk patients - Immunosuppressed, HIV positive - Transplant patient, steroid dependant - Diabetes, chemo - Prosthetic valve - Peripheral vascular disease - Elderly - Alcoholic - Cirrhosis - Social and compliance problems

What are the three main tracts of the spinal cord and where do they dessicate? (Ch. 106)

Spinothalamic: Dessication near the entry of the spinal nerve Posterior column: Medulla Corticospinal: Medulla

Describe the 4 stages of acetaminophen toxicity(Ch. 147)

Stage 1 = pre injury(<24 hours-36 hours) - Asymptomatic - Possible N/V/abdo pain Stage 2 = Liver injury(8-36 hours) - N/V/Abdo pain - Elevated AST/ALT Stage 3= maximal liver injury(2-4 days) - Fulminant hepatic failure Stage 4 = Recover(>4 days) - Death or recovery

What characteristics of intimate partner violence increase the risk of death(5) (Ch 68)

Stalking and harassment Estrangement Access to firearms by perpetrator History of forced sex Physical abuse during pregnancy

How is aortic dissection classified? (Ch 85)

Stanford (most commonly used) Type A: 62% according to IRAD. Involves ascending aorta. Can extend distally ad infinitum. Surgery usually indicated Type B: 38% Involves aorta beyond left subclavian artery only. Often managed medically with BP control. Tend to be older, heavy smokers w/ chronic lung disease w/ atherosclerosis and HTN.

What are the two most common organisms causing folliculitis and impetigo? (Ch 120)

Staph aureus vs. beta-hemolytic strep (more erythema). Note: be aware of "hot tub folliculitis post-hot tub/swimming pool" due to pseudomonas.

How is a proximal lower limb DVT managed? (Ch 88)

Start an appropriate LMWH (Enoxaparin 1 mg/kg SC q12hrs) OR Alternatives: (Check out Uptodate for a great discussion) Fondaparinux Apixaban Rivaroxaban OR heparin infusion after a loading bolus Ideally transition to oral anticoagulation at least 3 months (some up to 12 months) Encourage ambulation as much as possible Bedrest promotes DVT extension, risk of embolization, and post-DVT syndrome

Describe the initial stepwise management of altered LOC? (Ch. 16)

Step 1: MOVIE (monitors, O2, vitals, IV, environment) Step 2: ABCs Step 3: Blood sugar, temp, ECG Step 4: Coma cocktail: Dextrose, O2, Narcan, Thiamine Step 5: CXR, CT head, ABG?, consider antibx, metabolic antidotes, trauma management, secondary survey

Describe the management of simple dental abscesses (Ch. 70)

Step 1: anesthetize area - via apical nerve block or superior / inferior alveolar blocks. Rosens recommends lido w/epi, but consider longer acting bupivicane for patient comfort. Step 2: Stab incision through gingiva, extend down through periosteum of alveolar bone. Step 3: Blunt dissection with mosquito hemostat Step 4: Consider placement of penrose or iodoform if space permits. Secure with 4-0 suture. Drains to be removed in 24-48hrs (by Dentist preferably!) Step 5: Old recommendations to over with Penicillin V 250mg QID or if pen allergy Doxycycline 100mg BID for 10 days. However, antibiotic use may not be needed if adequate drainage is achieved. Counsel for importance of warm saline rinses regularly.

How does one differentiate between viral and bacterial causes of conjunctivitis? (Ch. 22)

Still no good test... LR of 3.1 for bacteria if sticky eyelids in the AM + purulent discharge

What is Sialolithiasis? How is it tx?(Ch. 72)

Stone in the salivary gland: 85-90% of cases involve submandibular glands Ddx: Infections, inflammation, granulomatous and neoplstic process Most common viral pathogen is mumps Most common bacterial is strep viridans, S. pneumoniae and H. Flu Tx: Massage heat, sialagogues(ex lemon candies to promote salivation), antibiotics if infection, lithotripsy as per ENT. ENT FU immediately if stone cannot be removed in ED, in 4-5 days if it can be

What bugs cause bacterial conjunctivits?(Ch. 71)

Strep. pneumoniae, H. influenzae, Staph, Moraxella, Neisseria gonorrhoeae, Klebsiella, pseudomonas

Which upper extremety artery injury is potentially most lethal? (Ch. 48)

Subclavian artery due to *air embolisms* going to brain

What are the 4 major types of a luxed tooth?(Ch. 70)

Subluxation: Loosened tooth Extrusive: Partial displacement of the tooth out of the socket, along the axis of the tooth Lateral: Displacement of the tooth eccentrically Intrusive: Displacement of the tooth deeper into the socket, along the axis of the tooth

Describe the management of chronic pancreatitis (6 priorities)? (Ch 91)

Supportive Pain relief Start with NSAIDS / acetaminophen Octreotide Celiac plexus block Correction of fluid imbalance Correction of electrolyte deficits Glucose control Correction of malnutrition Oral enzyme replacement Antioxidants Healthy diet Smoking / ETOH cessation Endoscopy: Drainage of symptomatic pseudocysts / ductal leaks (refractory ascites) Stenting obstructed bile ducts

Fat embolism syndrome after long bone fracture. What can be done? (Ch. 49)

Supportive care 20% mortality rate No therapy shown to benefit

List factors which favour admission in PID (Ch 98)

Surgical emergencies cannot be excluded Pregnancy Tubo-ovarian abscess Severe illness, nausea, vomiting, or high fever Inability to follow or tolerate outpatient regimens. Failure to respond to oral therapy

List the the types of dysphagia. What is an ED approach to this condition? (Ch 89)

Swallowing is divided into oral, pharyngeal, and esophageal phases. Failure at any one of these levels results in dysphagia, which literally means "difficulty Swallowing." When it comes to causes, just think M&M. The problem is either a mechanical block or a motility problem. The upper swallowing action is very complex neuro-muscular action that involves the lips, tongue, pharynx,etc the most common causes of OPD are motility-related.

What are the fundoscopic findings of papilledema? List 8 ddx for papilledema. (Ch. 71)_

Swelling of the optic nerve head Blurring of the disc margins Hyperemia Loss of physiologic cupping Ddx: SAH tumor Idiopathic intracranial hypertension brain abscess meningitis CRVO Hypertensive retinopathy Optic disc vasculitis

Describe the clinical exam and ECG findings associated with HCM (Ch 82):

Sx: Dyspnea, chest pain, syncope, near-syncope, palpitations Px: Low S4 gallop Harsh midsystolic murmur Worse with valsalva or changing from standing to squatting position (change in preload and afterload - its a dynamic murmur) Bifid arterial pulse ECG: abnormal in 90% PACs and PVCs Multifocal ventricular ectopy Ventricular and supraventr. Dysrhythmias LVH ST segment changes T wave inversions LAE Abnormal / dagger Q waves Diminished or absent R waves in the lateral leads

List causes of esophagitis (Ch 89)

Symptoms: dysphagia, odynophagia, chest pain, bleeding → hemorrhage → perforation! 1. GERD (#1 cause) 2. Eosinophilic esophagitis Unknown etiology; typically in non-responders to PPI's and kids with atopy. Don't respond to high dose PPIs, may need steroids inhalers. 3. Infection Usually an immunocompromised (steroid use, ETOH, immunosuppression, or HIV seroconversion syndrome) person developing Candida (but can also be CMV/HSV). Usually have odynophagia not improved with antacids. 4. Foreign body / toxic ingestion Pill esophagitis - from large pills sticking to the esophageal wall. Can lead to erosions/ulcer/strictures. 5. Post-radiation

What is the treatment for tinea capitis? (Ch 120)

Systemic antifungal agent, such as terbinafine or griseofulvin. Therapy should be given for 4 to 6 weeks. The patient should be referred for outpatient follow-up with primary care within 4 weeks. Alternative therapy includes fluconazole or itraconazole for 4 to 6 weeks.

How does hypertensive encephalopathy occur? (Ch 84)

THIS IS A TRUE EMERGENCY!!!! People presenting with severe headaches, vomiting, altered mental status, seizures, coma, blurred vision, neurologic deficits in multiple nerve distributions (due to diffuse cerebral dysfunction. These people have normal CT heads (or no new interval changes) + markedly elevated BPs + papilledema. BUT A REAL THING CALLED PRES. Failure of autoregulation Leads to vasospasm -> ischemia -> increased vascular permeability -> punctate hemorrhages -> interstitial edema NET RESULTS: Diffuse vasogenic cerebral edema. You worry about exploding heads with shifting Na too fast, this is the same thing in the end. Don't let your patient's heads blow up in front of you...(to exaggerate!)

What is the risk of stroke after TIA? How do you risk stratify a pt presenting with TIA? Describe your management of TIA in the ED (CH. 101)

TIAs constitute an important warning sign for the future development of cerebral infarction. Approximately 10% of the patients who experience a TIA will experience a stroke within 3 months of the sentinel event, and one-half of these occur within the first 2 days. A simple but suboptimal assessment called the ABCD2 score (ie, ABCD squared, for Age, Blood pressure, Clinical features, Duration of symptoms, and Diabetes) was designed to identify patients at high risk of ischemic stroke in the first seven days after TIA. Tx As per the Canadian Best practise guidelines we should be starting patients with TIAs in the ED on antiplatelet therapy (as long as they have no contraindications). The NNT for this is 77. ASA alone if not currently on antiplatelet Add second antiplatelet agent (eg plavix plus ASA) if already on single agent and had TIA "The more recent CHANCE trial found that dual antiplatelet therapy with aspirin and clopidogrel for 90 days significantly reduced recurrent stroke (from 11.7% to 8.2%) in patients initially presenting with high-risk TIA or mild stroke (NIHSS below 4). Aspirin should not be given for the first 24 hours to patients who have received a fibrinolytic agent and not until a swallowing study has been performed." - Rosen's 9th Ed.

What are three ECG presentations of sick sinus syndrome? (Ch 79)

Tachy-brady syndrome A. fib with episodes of incomplete sinus block Complete sinus block

What causes the finding of a "second pupil" post trauma? (Ch. 71)

Tearing of the iris from the anterior ciliary body, usually after blunt trauma

This emergent rheumatologic disease presents with vision loss and jaw claudication (Ch 118)

Temporal arteritis.

Describe the difference between tendonitis and tendonosis: (Ch. 49)

Tendonitis: inflammatory condition characterized by pain at tendinous insertions into bone, occurring in the setting of overuse Tendonosis: contentious name that describes more chronic conditions: eg. degenerative changes, chronic tendinopathy, partial rupture

Describe what a normal capnography tracing looks like? How does that compare to someone with COPD? Why?

That slow sharp peaked rise in CO2 has to do with the obstruction. Ventilation (exhalation of CO2) is obstructed due to airflow limitations

Your patient comes in with AECOPD. He's placed on 100% FiO2 while you are seeing another patient and his sats are 100%. Several hours later, he goes into hypercapneic respiratory failure. Why?

The 100% O2 likely suppressed his hypoxic respiratory drive Make sure to target O2 sats 88-92% in these patients

What 2 lab tests can be used to help the diagnosis of SBO (Ch 92).

The 9th Ed. of Rosen's only lists two lab tests that can be used. An elevated serum lactate and CPK - can suggest a bowel obstruction with strangulation if its late in the disease process. So, add these tests on if you're at all suspicious - but know that a normal level won't rule the disease out!

Compare the mechanisms of dialysis in HD and PD (Ch 97)

The basis of all dialysis is having the patient's blood come in contact with a semi-permeable membrane. In IHD, that membrane is external to the patient and thus via venous access, we remove their blood and return it post filtration. In peritoneal dialysis the patient's peritoneum acts as the membrane. The dialysate is pumped into the patient's abdomen via a Silastic catheter.

How do you make the diagnosis of acute pancreatitis? (Ch 91)

The clinical picture of pancreatitis is variable! And the level of pain doesn't correlate with disease severity. To make the diagnosis of pancreatitis you need ⅔ of these: 1. Clinical features (see textbook for a description) 2. Laboratory results (lipase or amylase > 3x ULN) 3. Imaging tests (not a requirement!)

What is an atheroma and how is it formed? (Ch 87)

The basic lesion, the atheroma, or fibrofatty plaque, is a raised focal plaque within the intima; it has a lipid core (mainly cholesterol, usually complexed to proteins and cholesterol esters) covered by a fibrous cap. As the plaques increase in size and number, they progressively encroach on the lumen of the artery and the adjacent media. Atheromas compromise arterial blood flow and weaken the walls of the affected arteries.

Describe the Straight-leg raise (SLR) and what it tests for (Ch 54):

The straight leg raise: Sensitive for sciatica (91%) but poor specificity (26%) Method: Supine patient with legs extended. Symptomatic leg is passively raised (knee straight) Presence of radiating back pain past the knee in between 30-70 degrees suggests a L5-S1 radiculopathy If the leg elevation produces isolated low back pain without radiation it is negative

How is the d-dimer test used in the diagnosis of DVT? (Ch 88)

The symptoms of DVT can be very subtle and nonspecific (cramping, sensation of fullness in the calf) Unilateral swelling, edema, erythema, warmth, tenderness to palpation of the venous system, dilation of the superficial collateral veins, palpable venous cord Homan's sign is insensitive and nonspecific = useless! Do not rely on it! ****the lack of objective swelling is an unreliable gestalt tool to exclude the diagnosis of DVT!**** When the DVT is at the "charley horse" stage - non specific mild symptoms - it is best for us to catch it because we can reduce the morbidity and mortality of VTE Use of D-dimer: (See question 5!) People with a LOW pre-test probability can be risk stratified as "unlikely DVT" with a NORMAL quantitative D-Dimer.

List 2 findings on XR of orbital floor fracture (Ch. 71)

The teardrop sign, a bulge extending from the orbit into the maxillary sinus, An air-fluid level in the maxillary sinus are indirect signs of orbital floor injury.

What are the symptoms of a brown recluse spider bite?(Ch. 55)

The toxin causes local tissue damage and hemolysis, and system symptoms are primarily allergic. Pain usually develops over 3-4 hours with a white area of casocronstiction suorrounding a blebb. This bleb eventually undergoes necrosis, and may resemble a MRSA infection. System symptoms include hemolytic complications (thrombocytopenia, shock, jaundice, renal failure, hemorrhage) and pulmonary even, eventually culminating in DIC.

What is a pilonidal cyst? (Ch 96)

These are abscesses containing hair in the midline of the sacrococcygeal area. Bacteria enter the hair follicle and inflame the area which form a microabscess. This then perforates in the subcutaneous tissue and forms a bigger abscess in the presacral area.

What is blepharitis? What is the treatment? (Ch. 71)

Thickened, mattered red eyelid margins with pronounced blood vessels --> Not an infection! Tx: Rubbing eyelid margins with mild shampoo + cloth Warm compress QID Severe: Topical abx

Outline a stepwise approach to treating acute migraine attacks? List 8 therapy options for Migraine.

Things to consider - what's worked for the patient in the past - what are the patient's comorbidities - presence of nausea/vomiting - PO meds don't often help 1. Acetaminophen or NSAIDs 2. First line agents for moderate-severe tasks - DHE (dihydroergotamine) - Triptans - Prochlorperazine - Metocloperamide - Droperidol - Ketoralc 3. Second line agents - Morphine (hydromorphone less effective) - Magnesium - Valproic Acid

Name 6 risk factors for urolithiasis (Ch 99)

Think DRY ROCKS Dehydration / hot climates / acidosis Recurrent UTI Y chromosome Relatives with stones Odd habits (milk-alkali, laxative abuse, calcium ingestion) Calcium diseases (primary hyperparathyroidism, malignancy, sarcoid) Kidney stones Small bowel diseases (Crohns)

Describe a general approach to the evaluation of acute kidney injury (AKI) (Ch 97)

Think Pre-renal, renal, post-renal! Hx: Look for pre-renal causes: Decreased cardiac output signs & symptoms (lightheadedness,SOB, PND, orthopnea etc) Bleeding GI fluid loss Abnormal polyuria Hx: Obstructive / Post Renal: Nocturia / frequency / hesitancy or decreased urinary stream Dysuria / hematuria (infection / inflammation or neoplasm) Hx kidney stones or chronic urinary tract infection (UTI). Pearl: Hx of acute anuria (<100 mL of urine/day) = high-grade urinary tract obstruction but can also include : Severe volume depletion Severe acute glomerulonephritis Cortical necrosis Bilateral renal vascular occlusion.

List the causes of prerenal azotemia (Ch 97)

Think Volume, Cardiac, Neurogenic: Volume loss: GI losses Renal (diuresis) Blood loss Insensible losses Third-spacing sequestration Pancreatitis Peritonitis Trauma Burns Cardiac causes: Myocardial infarction Valvular disease Cardiomyopathy Decreased effective arterial volume Antihypertensive medication Nitrates Neurogenic causes: Sepsis Anaphylaxis Hypoalbuminemia Nephrotic syndrome Liver disease

List 8 causes of pruritus ani (Ch 96)

Think of mnemonic ITCH: Infection - STDs, HSV, HPV, Syphilis Topical irritants - focal dermatitis, poor hygiene, tight clothing, hemorrhoids, etc. Cutaneous conditions - Psoriasis, Lichen Sclerosis Hypersensitivity to foods or drugs!

Describe the difference between AV dissociation & 3° AV block (Ch 79):

Third degree AV block: absent conduction of ALL atrial impulses slow escape rhythm AV dissociation: Hallmark of complete heart block No electrocardiographic relationship between A/V activity R-R interval is longer than the P-P interval "Conversely, the presence of AV dissociation with an R-R interval that is shorter than the P-P interval (e.g., as occurs with accelerated junctional rhythms and VTs) does not imply third-degree heart block." **watch for Afib is accompanied by complete heart block. "regularized atrial fibrillation" is associated with digitalis toxicity

What is Vincent's Angina? (Ch. 70)

This is ANUG that extends into the fauces and tonsils

What are the classic symptoms of arterial insufficiency? (Ch 87)

This is a chronic disease which produces two different pain syndromes: Intermittent claudication Ischemic pain at rest Intermittent claudication can occur anywhere, but usually is in the calf (femoral or popliteal disease). Described as cramping pain reproduced by the same degree of exercise and relieved by 1-5 minutes of rest. Can also occur in the aortoilliac vessels causing buttocks/hip pain, aching, discomfort, weakness, or "giving out" with exercise See the discussion below on Leriche's syndrome When the disease progresses, ischemic symptoms can occur at rest. It usually starts at the foot. The classic scenario would be a person being awoken from sleep complaining of severe un-relenting pain distal to the midfoot. This pain is NOT relieved by analgesics and WORSENED with elevation. Ask about the "dangling over the bed" or sleeping in a recliner chair. Their pain should improve with standing up.

Describe clinical presentation of spinal neoplasm, investigations and management. (Ch. 106)

Thoracic is most common Back pain - Worse with recumbancy - Worse at night - Worse with valsalva, sneezing Investigations - Normal XR and ESR <20 is very unlikely cancer - MRI with IV contrast - CT myelography Tx: Radiation if cord compression, surgery

BLUNT chest trauma... Crack the chest or not? That is the question. How do you decide? (Ch. 36)

Thoracotomy in... Blunt trauma with: 1. Signs of life on arrival to the ED (any 1 of): ● blood pressure ● pulse ● cardiac rhythm ● respiratory effort ● U/S ECHO showing cardiac activity or tamponade 2. Less than 10 mins of paramedic-based CPR ● can consider doing thoracotomy **Consider intubating, giving IV fluids, and needling both chests or bilateral finger thoracostomy

Describe the pathophysiology of migraine

Thought now to be abnormal trigeminal nerve activation

What are the potential complications of AV fistula creation? (Ch 87)

Thrombosis Infection Steal phenomenon Atherosclerotic disease distal to the shunt (e.g. the vein) leading to stealing of blood through the ulnar artery via the palmar arch - can lead to finger tip ischemia Venous hypertension** Acute venous HTN is a surgical emergency that occurs in the first few weeks post-op AV fistula creation. Leads to acute, severe venous stasis in the extremity Can lead to increased distal venous dilation and life threatening rupture if not recognized and treated surgically ASAP (vein ligation) Bleeding

Describe 4 potential targeted investigations for patients presenting to the ED with suicide (Ch 115)

Through a FOCUSED, MEDICAL assessment. - Drug ingestion - Trauma or associated medical illness - Evidence of self-harm behavior such as wrist-cutting - Cognitive status - Vital signs, pupils, skin, and nervous system examinations are helpful in detecting organic conditions, particularly toxidromes associated with common ingestions (see Chapter 139). - In the case of altered mental status, the provider should determine whether the condition is caused by an organic (medical) or functional (psychiatric) cause.

What are the most common sites of compartment syndrome? (Ch. 49)

Tibia, forearm, thigh, hands/feet

Indications for giving pRBCs: (Ch. 7)

To improve O2 delivery and improve intracellular O2 consumption

List the features of transverse cord syndrome(Ch. 106)

Total loss of motors, sensory and autonomic innervation distal to the injury

What is an occlusive vascular trauma? (Ch. 48)

Total loss of perfusion distal to injury site

List 5 causes of chronic pancreatitis (Ch 91)

Toxic-metabolic ETOH abuse Smoking Obstructive Genetic Autoimmune Post-necrotic acute pancreatitis Idiopathic

Describe the classic presentation of acute mesenteric ischemia (Ch 92)

Traditional triad is the cardiac patient that presents with sudden onset non-localised abdominal pain and gastric emptying (vomiting or diarrhea) *** 50% of patients with mesenteric ischemia have a prior history of embolic events !!! *** (eg. DVT, PE, Stroke)

List 6 population risk factors for CAD (Ch 78):

Traditional: Male gender, age, tobacco smoking, hypertension, diabetes mellitus, hyperlipidemia, family history, artificial or early menopause, and chronic cocaine abuse. Non-traditional: Rheumatoid arthritis, Lupus, HIV, IBD.

Leading cause of transfusion related mortality: (Ch. 7)

Transfusion-related acute lung injury (TRALI)

List 8 causes of fecal incontinence (Ch 96)

Traumatic Spinal cord injury Post surgical or obstetric injury Neurologic Diabetes Hirschsprung's disease Spinal cord lesions (cancer, ischemia, infection) Mass effects Anorectal cancer Foreign body / fecal impaction / hemorrhoids Medical Diarrhea / laxative abuse Dementia Pediatric patients Congenital, meningocele, spina bifida Other causes Post-corrective surgery for an imperforate anus Sexual abuse Encopresis (young children experiencing emotional stress).

What is the management of adnexal torsion? (Ch 100)

Treat pain & N/V Volume resuscitate if ++ vomiting Labs & R/O pregnancy Obtain imaging STAT OB/GYN Consult for OR

What is your management for a brown recluse spider bite?(Ch. 55)

Treatment is primarily supportive, but dapsone (an antibiotic known better for leprosy and PJP Pneumonia) 50-200mg/d can be consider for prevention of local effects if used within 48 hours. Analgesic are universally recommended. Signs of systemic envenomation require admission for observation. Antivenom is available in South America, but not commonly in North America.

What are appropriate treatments for STI prophylaxis in cases of suspected sexual abuse/assault (Ch 67)

Treatment without testing - Ceftriaxone 250mg IM x 1 or cefixime 400mg po - Metronidazole 2g po x 1 - Azithromycin 1g po x1 or doxycyline 100mg po BID x 7 days

What is Ramsey Hunt Syndrome? How is it tx?

Triad: ipsilateral facial paralysis, ear pain, and vesicles in the auditory canal and auricle - Its HSV! through the 8th cranial nerve Tx - Analgesia and antiviral Acyclovir 800mg five times daily Valacyclovir 1000mg three times daily

Describe the kinetics of cardiac biomarkers (Troponins and CK) (Ch 78):

Troponins TnT, and TnI are the most sensitive and specific serum proteins for myocardial injury They start rising within 2 hrs of symptom onset and stay elevated longer than CK-MB They are nearly 100% sensitive for AMI at 12 hrs from symptom onset High-sensitivity troponins are increasingly becoming used - and become elevated within a few hours from symptom onset CK-MB Is a myocardial CK protein previously used to diagnose AMI "In the setting of AMI, CK-MB is released and is detectable in the serum as early as 3 hours after onset of the necrosis. CK-MB characteristically peaks at 20 to 24 hours and becomes normal within 2 to 3 days after injury."

True or False: Resuscitation with fluids can lead to organ injury... (Ch. 6)

True Resuscitation can lead to liver injury, ARDS, & ATN

True or false: HINTS is better at ruling out central causes of vertigo than MRI with DWI. (Ch. 19)

True-ish (If done correctly)

True or False - For a patient with a history of radio-contrast induced anaphylaxis, you can give prednisone 50 mg by mouth given 13 hours, 7 hours, and 1 hour before the procedure. (Ch 119)

True.

True or False - Septic and obstructive shock can mimic anaphylaxis, and should always be on your differential. (Ch 119)

True.

True or False - methyldopa, procainamide, and minocycline can cause lupus (Ch 118)

True. Other drugs include hydralazine, chlorpromazine, isoniazid, and quinidine.

What is a jersey finger? Tx? (Ch. 50)

Tx: urgent ref. to hand surgeon

What are the five types of infarction (Ch 78)?

Type 1: a true ACS event leading to coronary thrombosis and vasospasm Type 2: demand ischemia - "shock" of the coronary arteries due to supply/demand imbalance Type 3: sudden cardiac death (arrest) before blood could be sampled. Type 4: Iatrogenic related to PCI Type 5: Related to CABG

What is the definitive management for each type of aortic dissection? (Ch 85)

Type A : Surgery. Stat. They need a graft replacement of the dissected section of the aorta. If they are hypotensive - get that four limb BP (to rule out the possibility that you are getting a falsely low BP in one limb that has an intimal flap leading to a low BP) If it's true hypotension - do that ECHO and consider draining a big pericardial effusion while waiting for the OR Type B: depends: in general medical management. Mortality in hospital approx 10% If "complicated" (see below) consider surgery vs. interventional stent-graft placement for: Persistent pain Uncontrolled hypertension Occlusion of a major arterial trunk (arm, head, kidneys, mesentary) Localised aneurysm

What is the difference between type I and type II CRPS? (Ch. 49)

Type I - not confined to one nerve distribution Type II - demonstrable peripheral nerve injury exists

What is the Allman Classification for clavicle fractures? (Ch. 53)

Type I - stable with coracoclavicular ligament intact Type II torn coracoclavicular ligament - often displaced Type III - involve articular surface

Describe the NEER classification for rotator cuff injuries (Ch. 53).

Type I - traumatic tear (5%) Usually young patient with microtrauma in overhead sports Type II - Tears with dislocation Typicall anterior, assoiciated with future instability Type III - impingement tears (90%)

What type of immune reaction is SJS and TENS? (Ch 119)

Type IV: Cell-mediated delayed-hypersensitivity

What are the typical associative pathogens of pneumonia (Ch 76):

Typical ones: Streptococcus pneumoniae (very virulent) Haemophilus influenzae Staphylococcus aureus Atypical: Legionella Mycoplasma Chlamydophila Virals: Influenzae Parainfluenza

What is the primary screening investigation modality for major abdominal injury in pregnant patients? (Ch. 37)

ULTRASOUND (~97% acurate for detecting intrabdominal injuries in blunt trauma)

Best places for sensory testing of hand nerves: (Ch. 50)

Ulnar nerve.: volar tip of little finger Median nerve: volar tip of index finger Radial nerve: dorsal first web space (between thumb and index finger)

What are the three nerves of the hand and their associated motor functions? (Ch. 50)

Ulnar nerve: finger abduction, MCP flexion (starfish hand sign) Median nerve: thumb opposition to index and little finger (A-OK sign) Radial nerve: wrist extension, thumb abduction (hitch-hiker thumb sign)

What is the pathophysiology of ASA toxicity?(Ch. 149)

Unpredictable GI absorption(2-4-12 hours): Aspirin is hydrolyzed to salicylic acid and binds to albuin - Interferes with aerobic metabolism by uncoupling of mitochondrial oxidative phosphorylation = Metabolic acidosis, hypoglycemia and ketosis - At serum pH decreases more particles become unionized and cross the cell membrane and BBB = Cerebral edema, neuroglycopenia, death - Direct nephrotoxicity - Pulmonary edema = unknown mechnism - Tinnitus: Alterations in NMDA activity, decreased blood flow andincreased membrane permeability

List four potential sites for upper extremity aneurysms, and their associated underlying causes. (Ch 87)

Upper extremity aneurysms are a rare occurrence, unless there has been localised trauma Proximal subclavian artery aneurysms Thoracic outlet obstruction Trauma Atherosclerosis Axillary artery aneurysms Post-blunt trauma Prolonged use of crutches Anterior shoulder dislocation / humerus fracture Ulnar artery aneurysm Carpenters (hypothenar hammer syndrome) Radial artery aneurysm Post-cardiac angiography

What are the common causes of upper limb DVT? (Ch 88)

Upper limb DVT = thrombosis in the axillary vein "The deep veins of the upper extremity include the paired ulnar, radial and interosseous veins in the forearm, paired brachial veins of the upper arm, and axillary vein. The axillary vein originates at the lower border of the teres major muscle in continuity with the brachial veins. The basilic and cephalic veins, which are superficial veins, contribute to the axillary vein, though many anatomic variations occur. After passing the outer margin of the first rib, the axillary vein continues as the subclavian vein." - text from Uptodate - 2017 Causes swelling on the same side of an indwelling catheter / IVDU / trauma **use of D-dimer in ueDVT population is not adequately studied** Breakdown of causes: 50% post indewlling catheter Post-indwelling cardiac device placement Healthy, young throwing/working out athletes (see wisecrack #5) Trauma Hypercoagulable states with anatomic abnormalities Inherited or acquired subclavian vein stenosis

List 5 CXR findings of pleural effusion? (Ch 77) *Bonus points: How much fluid must be present to visualize the effusion on a plain AP/PA film?

Upright 1. Blunting of costophrenic angles on upright CXR 2. Hemidiaphragm obscured (larger effusion) 3. Upward concave meniscus (layers higher lateral than central) 4. Extension into major fissure 5. Total opacification of hemithorax Supine 1. Asymmetrical haziness or contrast difference on supine 2. apical capping 3. obliteration of the hemidiaphragm 4. widened minor fissure *Note: Need 250-500cc of fluid to be apparent on AP/PA CXR*

List 3 categories of complications of pelvic fractures (Ch 55):

Urologic More common in men, anterior pelvic fractures Check for blood at the meatus Retrograde urethrogram to check for urethral injury Don't forget bladder can be injured as well. Neurologic Worse with worsening instability (Tile A < B < C) Worse with more medial vertical sacral fractures (Denis I < II < III) Cauda equina possible Gynecologic Can have open pelvic fracture into vagina Look for blood at introitus Careful manual examination Gastrointestinal Open pelvic fracture into GI tract High risk of infection Careful digital internal examination

What do you do when the dix-hallpike maneuver is bilaterally positive? (Ch. 19)

Use the *ROLL TEST* (examines for horizontal canal variant of BPPV).

What is the Insall-Salvati Ratio used for (Ch 57): *Bonus points for knowing the measurement and ratio.

Used to diagnose abnormal high/low riding patella (Bonus: Lateral knee x-ray with knee 30 deg. Flexed Patellar tendon length / patellar bone length Normal ratio 0.8 - 1.2 (0.74 - 1.5 in other sources) Patella Baja = < 0.8 Patella Alta = >1.2)

List 6 causes of diarrhea in AIDS. Describe an appropriate initial work-up. What additional steps may be required? (Ch 94)

Usual bacterial suspects plus (don't forget primary HIV induced diarrhea, HAART induced, and salmonella / vibrio!): Cytomegalovirus Cyclospora Cryptosporidium Isospora Mycobacterium avium-intracellulare complex Giardia Workup: Stool examination & cultures C Diff toxins and salmonella PCR CMV and MAI if CD4+ count <200 Acid fast smear for Cryptosporidium, Cystoisospora, Isospora, and Cyclospora sigmoidoscopy

When should you perfrom a CT before LP in context of severe headache? (Ch. 20)

Usually CT first. LP can precede CT in meningitis if pt has normal neuro exam (GCS 15) and no papilledema.

Describe your management for Apthous Stomatitis (Ch. 70)

Usually self limiting condition, but make sure you don't miss systemic causes of oral ulcerations: Hand-foot-mouth / Herpetic stomatitis/ HIV / Syphilis / TB / Wegner's / Scleroderma / Lupus / to name a few Symptomatic care Saline and Hydrogen peroxide rinses Mix of Benzocaine and diphenhydramine gel Rx for steroid - antibiotic ointments (Kenalog or orabase) F/U with GP to ensure resolution

List complications of hemodialysis (Ch 97)

Vascular Access-Related Complications: 1. Occlusion 2. Bleeding 3. Infection Non-Vascular Access-Related Complications 1. Hypotension (see next question) 2. Bleeding (pericardial, pleural, GI, ICH, abdominal, retroperitoneal) 3. VTE 4. CAD 5. Neurologic Dysfunction (disequilibrium syndrome)

What is Brugada syndrome? What is the management? (Ch 79)

Ventricular dysrhythmia (sudden death, syncope) INHERITED sodium channel disorder males in young adulthood Dx: ***ecg findings may be transient and only elicited with pharm. stimulation*** saddleback or coved ST segment in V1-V3 similar to a RBBB Treatment: Syncope = ICD incidental = referral to cardiologist

Compare the expected lab work in acute viral hepatitis vs alcoholic hepatitis (Ch 90)

Viral: ALT > AST (both usually 10-100 x ULN), elevated bilirubin, elevated WBC, prolonged INR, fluid and electrolyte imbalances due to vomiting/diarrhea. ETOH: AST > ALT (values LESS than 10 times normal), elevated bilirubin, elevated WBC, prolonged INR, fluid and electrolyte imbalances due to vomiting/diarrhea.

What is Seidel's sign? (Ch 60)

Vitreal extrusion or Positive fluorescein flowing with Aqueous Humor As Rosen's puts it: "identification of rivulets of fluorescein tracking from the puncture (i.e., positive Seidel test result)" = GLOBE RUPTURE

List complications of AKI (Ch 97)

Volume overload - CHF, HTN Metabolic acidosis Hyperchloraemia accumulation of organic anions - PO4 decreased Alb -> decreased buffering impaired insulin action -> hyperglycaemia catcholamine resistance (bAR downregulation) increased iNOS Electrolytes - increased K+ and low Na+ Pulmonary oedema - low albumin -> decreased oncotic pressure + volume overload ALI - neutrophil activation and sequestration in the lung Uraemia (ALOC and pericarditis) Immune - decreased clearance of oxidant stress, tissue oedema, WCC dysfunction - increased risk of infection Haematological - decreased RBC synthesis and increased destruction of RBC -> anaemia, decreased EPO, platelet dysfunction secondary to uremic toxins, vWF -> bleeding GI - GI oedema -> compartment syndrome, decreased nutritional absorption, gut ischaemia -> peptic ulcer disease Pharmacology - increased Vd, decreased bioavailablity, albumin, decreased elimination -> under dosing or toxicity

Describe the treatment of anal fissures (Ch 96)

WASH Approach +/- Nitro. Ointment 0.4% bid/tid Nifedipine gel 0.2% bid with lidocaine gel 1.5% Surgical options (botox, anal dilation, excision). Most take 2-4 weeks to resolve.

Describe your management for Pericoronitis (Ch. 70)

Warm Saline irrigation +/- hydrogen peroxide rinses Antibiotics (strep viridans and anaerobes) Pen / Doxy / Metronidazole I&D abscess PRN *WATCH out for big red: Internal carotid is close to this area* Oral maxillofacial referral

Describe the ED management of rectal foreign bodies (Ch 96)

We need to remove them due to the risks of mucosal lacerations, perforation, obstruction, sepsis, peritonitis. Important things for us to know about: Fever, abd. peritonitis, rectal bleeding, duration of FB Should get an upright abd. Xray if it is radio-opaque If you suspect a perforation: try using water soluble rectal contrast to outline the foreign body Management: Ask the patient to assist with valsalva Use benzo's to help with rectal relaxation Suprapubic pressure can assist with removal Consider foley catheter to remove the suction - negative pressure proximally Some need deep procedural sedation or operative removal

What is epiglottis, how does it present, and list a few causes of this condition for bonus points (Ch 75)

What is it? Localized cellulitis of the supraglottic structures: based of the tongue, vallecula, aryepiglottic folds, arytenoids, lingual tonsils, epiglottis Presentation: Classically: Adult, male smoker with a "benign" URTI for the past few days who complains of dysphagia, odynophagia, and "sore throat/pharyngitis" with a "normal" oropharynx/tonsil exam Causes: H. influenzae type b Strep. Staph. Viruses Thermal injury

How do you perform the Dix-Hallpike maneuver? (Ch. 19)

When head is hanging over the edge of the bed, pt is asked about vertigo and observed for nystagmus. Only one head direction should be positive, indicating affected side.

When is WBC actually helpful in determining cause of abdo pain? (Ch. 27)

When it is low... suggesting immunosuppression

When are antibiotics indicated for appendicitis?(Ch 93)

When you make the diagnosis of appendicitis In patients with suspected appendicitis and severe sepsis or septic shock. "The choice of antibiotics should include broad-spectrum gram-negative and anaerobic coverage. For nonperforated appendicitis, we recommend ciprofloxacin, 400 mg IV, and metronidazole (Flagyl), 500 mg IV; or ceftriaxone, 1g IV, and metronidazole, 500 mg IV; or ampicillin-sulbactam, 3g IV monotherapy. For perforated appendicitis, we recommend broader spectrum antibiotics, such as piperacillin tazobactam, 3.375 to 4.5g IV, cefepime, 2 g IV, or imipenem cilastatin, 500 mg IV. Methicillin-resistant Staphylococcus aureus (MRSA) coverage is not typically needed to treat appendicitis but may be considered if the patient has previously known MRSA colonization " - rosen's 9th Ed.

List 10 conditions on the differential diagnosis of ischemic stroke. (Ch. 101)

acute/chronic subdural or epidural hematoma Brain tumour Brain abscess Air gas embolism Aortic dissection Carotid / cervical artery dissection Migraine Giant cell arteritis Polyarteritis nodosa Lupus / vasculitis Cerebral venous sinus thrombosis Hypoglycemia Wernicke's encephalopathy (ophthalmoplegia, ataxia, confusion) Post-seizure induced Todd's paralysis Bell's palsy Labyrinthitis Vestibular neuronitis Peripheral nerve palsy Demyelinating disease Meniere's disease

What is the treatment for bacterial otitis media? For those allergic to penicillin? For treatment failure? (CH. 72)

amoxicillin 80-90mg/kg/day Allergy: 1st gen ceph, azithro/ clarithro Failure: Amox/clav 90mh/kg/day

What causes oxygen-hemoglobin dissociation curve to shift to *right* (more O2 delivered to tissues)? (Ch. 14)

decreased pH increased temperature increased 2,3-BPG

What is FEIBA? (Ch. 7)

factor eight inhibitor bypassing activity

Decribe an orbital tripod fracture and its management: (Ch. 42)

orbital tripod fracture = fracture of the zygoma, maxilla, and lateral orbital wall, creating a mobile bony segment that is often depressed, causing facial asymmetry Management: surgical and ophtho consultation

List 3 complications of ocular trauma(Ch. 71)

post-traumatic corneal ulcers endophthalmitis sympathetic ophthalmia

How do you manage meconium? (Ch. 11)

tracheal suction ONLY if: -absent or depressed respiration (gasping) -poor muscle tone -HR <100

What is the most sensitive physial exam for assessing fracture-related nerve injury? (Ch. 49)

two-point discrimination

What is your intial care in hospital for snake bites?(Ch. 55)

vitals, IV, normal saline running in, CBC, UA, Creatinine, BUN, CK, Fibrin, INR/PTT, Electrolytes, ECG, type and screen***; consider hepatic studies as well. Minimal constitutes observation for up 6-8 hours. Moderate envenomation requires 4 FabAV (4-6 for pit viper) over 60 min, and monitor for progression to severe. Severe requires 4 FabAV (8-12 for pit viper) over 60 minutes then a maintenance of 2 vials every 3 hours or 4-6 if progressing to very severe. Pit viper envenomation may require initial doses of up to 18 vials, with maintenance up to 10 vials every 6 hours. Dont forget about tetanus and gram negative antibiotice(High risk wounds and patients)


Related study sets

ATI Dosage Calculation 2.0 Desired over Have-Injectable

View Set

Prep U Mastery Ch.66- Management of Patients with Neurologic Dysfunction

View Set

10B: It's said that peacock feathers are bad luck

View Set

Chapter 4- The Tissue Level of Organization (chapter questions)

View Set

Ch 5: The Working Cell (Dr. Kas)

View Set

PSY 2012 GENERAL PSYCHOLOGY QUIZ 14

View Set

НЕЙРОЛЕПТИКИ, ТРАНКВІЛІЗАТОРИ, СЕДАТИВНІ

View Set

Write the sentences in reported speech. 'I love you.' He told her that he <loved her>.

View Set

Macroeconomics chapter Exam 4 11,13 and 14

View Set

Describe the parts of a controlled experiment. Independent, dependent, controlled variables. Experimental vs. control groups

View Set